You are on page 1of 122

THPT CHUYÊN NGUYỄN QUANG DIÊU

√ a−b a−b
ab ≤ ln a−ln b ≤ 2 arcsin a−b
a+b

1 1
1 bb a2 +b2
  
b−a a+b 2
≤ e aa ≤ 2 ≤ 2

BẤT ĐẲNG THỨC

—————— QUA CÁC ĐỀ THI CHỌN ——————

ĐỘI HSG QUỐC GIA & HSG CẤP TỈNH

——————— NĂM HỌC 2017 - 2018 ———————

??
TRƯỜNG THPT CHUYÊN NGUYỄN QUANG DIÊU
—————————————

BẤT ĐẲNG THỨC

Qua các đề thi chọn đội HSGQG


&
HSG cấp Tỉnh

Năm học 2017 - 2018


??

Cao Lãnh - Đồng Tháp


Lời nói đầu
Tài liệu là lời giải câu bất đẳng thức (BĐT) trong các đề tuyển đội học sinh giỏi dự thi VMO và chọn
học sinh giỏi cấp Tỉnh cũng như các đề Olympic chuyên KHTN, GGTH,... trong năm học 2017 − 2018.

• Phần 1: Không dừng lại ở khâu trình bày lời giải, tài liệu cố gắng thể hiện quá trình hình thành
nên lời giải (chính vì vậy nên có nhiều lời giải có vẻ dài dòng) và đưa ra nhiều hướng suy nghĩ
khi tiếp cận một câu BĐT thông qua việc giải một bài toán bằng nhiều cách khác nhau (về phần
này người viết đã tham khảo nhiều nguồn bài giải khác nhau để có góc nhìn đa chiều, và khi đó
việc trích dẫn đúng nguồn là một điều mà người viết rất quan tâm).
Giải xong một bài toán, đưa ra nhận xét và tìm được các mối liên hệ của nó với các bài toán
khác là một việc nên làm với một người học Toán không chỉ riêng là BĐT. Nó cho người học thấy
được toàn cảnh bức tranh mà bài toán đó dệt nên, và cao hơn là thấy được tính "mới - cũ" và "
hay - dở" của mỗi đề toán. Về phần này, những nhận xét đều mang tính chủ quan. Bạn đọc có
thể đóng góp nếu thấy chưa đúng, chưa tốt.

• Phần 2: Phần bổ sung kiến thức. Ngoài việc nêu ra các phương pháp chứng minh BĐT thông
dụng, tài liệu còn trình bày thêm một số kỹ thuật, BĐT phụ,... khá đặc biệt trong chứng minh
BĐT mà nếu chưa gặp lần nào nhất định sẽ gây khó khăn cho người làm. Về cách đọc tài liệu
này, nhất là bạn đọc mới tiếp cận với BĐT thức thì nên đọc phần bổ sung kiến thức trước khi
đọc phần 1.

Người viết hy vọng qua những bài toán với những lời giải như vậy, bạn đọc hay cụ thể là những bạn
học sinh "say mê" BĐT hãy xem mỗi bài toán là một hành trình, hành trình đó ngắn hay dài tùy thuộc
vào mỗi bạn, nhưng luôn có một điểm chung là lời giải cuối hành trình luôn chứa đựng rất nhiều điều
thú vị.
Cuối cùng, người viết xin gửi lời xin lỗi bạn đọc về một số lỗi sai trong của tài liệu cùng tên được viết
cho năm học 2016 − 2017 chưa có dịp đính chính, các mục sai đó sẽ được đính chính trong phần cuối
của tài liệu này. Trong tài liệu này người viết vẫn mong nhận được những góp ý của bạn đọc cho tài
liệu không chỉ dừng lại ở những sai sót. Mọi góp ý bạn đọc có thể gửi về địa chỉ: anh110004@gmail.com.
Người viết chân thành cám ơn!

Cao Lãnh, Ngày 7 tháng 12 năm 2017

GV Nguyễn Tuấn Anh


Trang ii

GV: Nguyễn Tuấn Anh


Mục lục

Chương 1. BẤT ĐẲNG THỨC QUA CÁC ĐỀ THI OLYMPIC 1


1 Các bài toán BĐT qua các đề thi trong hè . . . . . . . . . . . . . . . . . . 1
2 Các bài toán BĐT qua các đề thi chọn đội HSGQG . . . . . . . . . . . . 9
3 Các bài toán BĐT qua các đề thi chọn HSG cấp tỉnh . . . . . . . . . . . 33

Chương 2. BỔ SUNG KIẾN THỨC 57


1 Một số phương pháp chứng minh BĐT thông dụng . . . . . . . . . . . . . 57
1.1 Kỹ thuật chọn điểm rơi . . . . . . . . . . . . . . . . . . . . . . . . 57
1.2 Phương pháp tiếp tuyến . . . . . . . . . . . . . . . . . . . . . . . . 60
1.3 Phương pháp pqr . . . . . . . . . . . . . . . . . . . . . . . . . . . . 64
1.4 Phương pháp dồn biến . . . . . . . . . . . . . . . . . . . . . . . . . 70
1.5 Phương pháp SOS . . . . . . . . . . . . . . . . . . . . . . . . . . . 75
2 Một số kỹ thuật, BĐT phụ đặc biệt trong chứng minh BĐT . . . . . . . 82
2.1 Một số phép đổi biến trong chứng minh BĐT . . . . . . . . . . . 82
2.2 Một số BĐT phụ thường dùng trong chứng minh BĐT . . . . . . 94
2.3 Bất đẳng thức Gerretsen . . . . . . . . . . . . . . . . . . . . . . . . 114

iii
Chương Một

BẤT ĐẲNG THỨC QUA CÁC ĐỀ


THI OLYMPIC

Nội dung chính


1 Các bài toán BĐT qua các đề thi trong hè . . . . . . . . . . . . . . 1

2 Các bài toán BĐT qua các đề thi chọn đội HSGQG . . . . . . . . 9

3 Các bài toán BĐT qua các đề thi chọn HSG cấp tỉnh . . . . . . . 33

1 Các bài toán BĐT qua các đề thi trong hè


Bài 1.1 (Olympic chuyên KHTN 2017).

Cho a, b, c là các số thực dương sao cho abc = 1. Chứng minh rằng:
 2  2  2
2+a 2+b 2+c
+ ca +a ≤ 1 + a + ca
1+a+b 1+b+c 1+c+a

LỜI GIẢI. (Mr Cooper - https://diendantoanhoc.net)


Áp dụng BĐT Cauchy ta có:

2
 (1 + 1 + a) ≤ (1 + b + a) (1 + ca + a)


ca(1 + 1 + b)2 ≤ ca (1 + c + b) (1 + ab + b) = (1 + b + c) (ca + a + 1)

 a(1 + 1 + c)2 ≤ a (1 + a + c) (1 + bc + c) = (1 + a + c) (a + 1 + ca)

Do vậy ta cần chứng minh:


1 1 1
h i
(1 + a + ac) + + ≤ 1 + a + ca
1+a+b 1+b+c 1+c+a

1 1 1
⇔ + + ≤1
1+a+b 1+b+c 1+c+a

1
Trang 2 CHƯƠNG 1. BẤT ĐẲNG THỨC QUA CÁC ĐỀ THI OLYMPIC

BĐT trên khá quen thuộc và có nhiều hướng giải


Hướng 1: Vì abc = 1 nên tồn tại x, y, z > 0 sao cho:

x2 y2 z2
a= ;b = ;c =
yz zx xy

Khi đó BĐT được viết lại là:

xyz xyz xyz


3 3
+ 3 3
+ ≤1
xyz + x + y xyz + y + z xyz + z 3 + x3

Ta có:
xyz xyz xyz
3 3
+ 3 3
+
xyz + x + y xyz + y + z xyz + z 3 + x3

xyz xyz xyz


≤ 2 2
+ 2 2
+ =1
xyz + x y + xy xyz + y z + yz xyz + z 2 x + zx2

nên bài toán được chứng minh. Đẳng thức xảy ra khi và chỉ khi a = b = c = 1.
Hướng 2: Vì abc = 1 nên tồn tại x, y, z > 0 sao cho:

x y z
a= ;b = ;c =
y z x

Khi đó BĐT được viết lại là:

yz zx xy
2
+ 2
+ ≤1
yz + zx + y zx + xy + z xy + yz + x2

Theo BĐT Cauchy ta có:

yz z 2 + xy + yz
yz + xz + y 2 z 2 + xy + yz ≥ yz(z + x + y)2 ⇔
 

(yz + xz + y 2 ) (x + y + z)2

Tương tự cho các số hạng còn lại, ta được:


yz zx xy
2
+ 2
+ ≤
yz + zx + y zx + xy + z xy + yz + x2

z 2 + xy + yz x2 + yz + xz y 2 + xz + xy
+ + =1
(x + y + z)2 (x + y + z)2 (x + y + z)2

Và bài toán được chứng minh.

Hướng 3: Vì abc = 1 nên tồn tại x, y, z sao cho a = x3 , b = y 3 , z = c3 thỏa xyz = 1. BĐT
được viết lại là:
1 1 1
3 3
+ 3 3
+ ≤1
1+x +y 1+y +z 1 + z 3 + x3

1. Các bài toán BĐT qua các đề thi trong hè GV: Nguyễn Tuấn Anh
CHƯƠNG 1. BẤT ĐẲNG THỨC QUA CÁC ĐỀ THI OLYMPIC Trang 3

Theo BĐT AM - GM ta có:


1 1 1
3 3
+ 3 3
+
1+x +y 1+y +z 1 + z 3 + x3

1 1 1
= 3 3
+ 3 3
+
xyz + x + y xyz + y + z xyz + z 3 + x3

1 1 1
≤ + + =1
xyz + xy (x + y) xyz + yz (y + z) xyz + zx (z + x)
Ta được điều phải chứng minh.
Hướng 4: (Võ Quốc Bá Cẩn) Ta đặt x2 = a, y 2 = b, z 2 = c khi đó BĐT cần chứng
minh là:
1 1 1
+ 2 + 2 ≤1
x2 + y + 1 y + z + 1 z + x2 + 1
2 2

với xyz = 1. Ta để ý rằng:


1 x2 + y 2 (x + y)2 + (x − y)2
1− 2 = 2 =
x + y2 + 1 x + y2 + 1 2 (x2 + y 2 + 1)
BĐT được viết lại là:
X (x + y)2 X (x − y)2
+ ≥4
x2 + y 2 + 1 x2 + y 2 + 1
Theo BĐT Cauchy dạng phân thức ta có:
P (x + y)2 4(x + y + z)2


 ≥
x2 + y 2 + 1 2 (x2 + y 2 + z 2 ) + 3


P (x − y)2 4(x − z)2








x2 + y 2 + 1 2 (x2 + y 2 + z 2 ) + 3
Do đó ta cần có:
4(x + y + z)2 4(x − z)2
+ ≥4
2 (x2 + y 2 + z 2 ) + 3 2 (x2 + y 2 + z 2 ) + 3

⇔ (x + y + z)2 + (x − z)2 ≥ 2 x2 + y 2 + z 2 + 3


Vì xyz = 1 nên xy + yz + zx ≥ 3. Vậy nên ta cần có:


2 2 2 2 2

(x + y + z) + (x − z) ≥ 2 x + y + z + xy + yz + zx

⇔ xy + yz ≥ zx + y 2 ⇔ x (x − y) + z (y − x) ≥ 0 ⇔ (x − z) (x − y) ≥ 0

BĐT cuối chỉ đúng khi x là số lớn nhất (hoặc số nhỏ nhất) trong 3 số x, y, z . Điều này
ta có thể giả sử ngay ban đầu vì tính đối xứng của BĐT. Do đó bài toán được chứng
minh.

GV: Nguyễn Tuấn Anh 1. Các bài toán BĐT qua các đề thi trong hè
Trang 4 CHƯƠNG 1. BẤT ĐẲNG THỨC QUA CÁC ĐỀ THI OLYMPIC

Hướng 5: BĐT tương đương với:


a+b b+c c+a
+ + ≥2
a+b+1 b+c+1 c+a+1

Áp dụng BĐT Cauchy ta có:


√ √ √ 2
a+b b+c c+a a+b+ b+c+ c+a
+ + ≥
a+b+1 b+c+1 c+a+1 2 (a + b + c) + 3

Pp P √ 
2 (a + b + c) + 2 (a + b) (a + c) Cauchy 2 (a + b + c) + 2 a+ bc
= ≥
2 (a + b + c) + 3 2 (a + b + c) + 3

√ √ √ 
4 (a + b + c) + 2 ab + bc + ca
=
2 (a + b + c) + 3
Do đó ta cần có: √ √ √ 
4 (a + b + c) + 2 ab + bc + ca
≥2
2 (a + b + c) + 3

√ √ √
⇔ ab + bc + ca ≥ 3
BĐT cuối là hiển nhiên vì abc = 1. Bài toán được chứng minh. 
Nhận xét:

• Bài toán có dạng tương đối lạ, và mấu chốt vẫn là đoạn sử dụng BĐT Cauchy ban
đầu.

• Kết quả quen thuộc trong bài giải từng là một câu trong Macau Mathematics
Olympiad 2000 và kết quả làm mạnh, là đề Moscow MO 1997 - lớp 11 như sau:
1 1 1 1 1 1
+ + ≤ + + ≤1
1+a+b 1+b+c 1+a+c 2+a 2+b 2+c

• Một số bài toán có dạng tương tự như kết quả được sử dụng trong bài chứng minh:

(GMB-IMAR 2005, Juniors, Problem 1) Với a, b, c là các số thực dương có


tích lớn hơn bằng 1 thì:
1 1 1
+ + ≤1
1+b+c 1+c+a 1+a+b

Kết quả tương tự:


(https://artofproblemsolving.com) Với a, b, c, d là các số thực dương có tích
không lớn hơn bằng 1 thì:
1 1 1 3
+ + ≤
1+b+c 1+c+a 1+a+b 2abc + 1

1. Các bài toán BĐT qua các đề thi trong hè GV: Nguyễn Tuấn Anh
CHƯƠNG 1. BẤT ĐẲNG THỨC QUA CÁC ĐỀ THI OLYMPIC Trang 5

Mở rộng cho 4 biến:


(Romania Junior Balkan Team Selection Tests 2013) Với a, b, c, d là các số
thực dương có tích lớn hơn bằng 1 thì:
1 1 1 1
+ + + ≤1
a+b+2 b+c+2 c+d+2 d+a+2

• Kỹ thuật được sử dụng trong hướng 4 là mạnh tay đối với bài toán, tuy nhiên nó
được trình bày ra với mục tiêu giới thiệu bạn đọc một kỹ thuật khá hay, thể hiện
sự tinh tế của người làm. Về phần kỹ thuật này mời bạn đọc tham khảo "Một kỹ
thuật nhỏ để sử dụng BĐT Cauchy Schwarz - Võ Quốc Bá Cẩn".

• Kết quả quen thuộc trên có thể được mở rộng như sau:
(Sử dụng phương pháp Cauchy Schwarz để chứng minh BĐT -
V.Q.B.Cẩn, T.Q.Anh) Với a, b, c là các số thực dương có tích bằng 1 và k > 0
thì:
1 1 1
k
+ k
+ ≤1
1+b+c 1+c+a 1 + a + bk

• Với cách làm trong hướng 5 ta có thể làm với bài toán mở rộng sau:
(Phân loại và phương pháp giải toán BĐT - Vasile Cirtoaje - V.Q.B.Cẩn,
T.Q.Anh) Với a, b, c là các số thực dương có tích bằng k 3 ≤ 1 thì:
1 1 1 3
+ + ≤
1+b+c 1+c+a 1+a+b 1 + 2k

Bài 1.2 (Olympic gặp gỡ Toán học 2017- Đồng Tháp, Lớp 11).

Cho ba số thực không âm x, y, z thỏa mãn điều kiện xy + yz + zx ≤ 1. Chứng minh rằng:

1 − xy − yz − zx ≤ (6 − 2 6)(1 − min{x, y, z})

LỜI GIẢI. Do vài trò đối xứng của x, y, z nên ta có thể giả sử x = min{x, y, z}. Khi đó
ta cần chứng minh:
√ 
1 − x (y + z) − yz ≤ 6 − 2 6 (1 − x)

Đến đây ta có ý tưởng sẽ đánh giá y, z để đưa BĐT trên về một ẩn là x. 1 Ta có:

y + z ≥ 2x và yz ≥ x2 ⇒ 1 − x (y + z) − yz ≤ 1 − x (2x) − x2

Vậy nên ta chỉ cần chứng minh:


√  √ 2
1 − x (2x) − x2 = −3x2 + 1 ≤ 6 − 2 6 (1 − x) ⇔ 3x + 6 − 3 ≥ 0
1

Và khi đó con số 6 − 2 6 sẽ có câu trả lời ở đâu sinh ra nó

GV: Nguyễn Tuấn Anh 1. Các bài toán BĐT qua các đề thi trong hè
Trang 6 CHƯƠNG 1. BẤT ĐẲNG THỨC QUA CÁC ĐỀ THI OLYMPIC


3− 6
Bài toán được chứng minh. Đẳng thức xảy ra khi và chỉ khi x = y = z = . 
3
Bài 1.3 (Olympic gặp gỡ Toán học 2017- Đồng Tháp, Lớp 10).

Cho ba số thực dương a, b, c thỏa mãn a2 + b2 + c2 = 3. Chứng minh rằng:


a b c 3
+ + ≥
b(a + c) c(b + a) a(c + b) 2

LỜI GIẢI.
Cách 1: Theo BĐT AM - GM ta có:
r
a b c 1
+ + ≥33
b(a + c) c(b + a) a(c + b) (a + b)(b + c)(c + a)

Bài toán sẽ được chứng minh nếu ta chứng minh được:

(a + b)(b + c)(c + a) ≤ 8

Cũng theo BĐT AM - GM ta lại có:


 3
2(a + b + c)
(a + b)(b + c)(c + a) ≤
3

mà a + b + c + 3 ≤ 2(a2 + b2 + c2 ) suy ra a + b + c ≤ 3. Do đó bài toán được chứng minh.


Đẳng thức xảy ra khi và chỉ khi a = b = c = 1.
Cách 2: Theo BĐT Cauchy ta có:
r !2
qa b
qc
+ +
a b c b c a
+ + ≥
b(a + c) c(b + a) a(c + b) 2(a + b + c)

Do đó ta cần chứng minh:


r r r !2 r r r !
a b c a b c b c a
+ + = + + +2 + + ≥ 3(a + b + c)
b c a b c a a b c

Ta thấy vế trái luôn lớn hơn hoặc bằng 9 (theo BĐT AM - GM) trong khi vế phải thì
bé hơn hoặc bằng 9 (vì a + b + c ≤ 3) nên BĐT trên hiển nhiên đúng.
Cách 3: Theo BĐT Cauchy ta có:
√ √ √ 2
a b c a+ b+ c
+ + ≥
b(a + c) c(b + a) a(c + b) 2(ab + bc + ca)

Nên ta cần chứng minh:


√ √ √ 2
3 a+ b+ c ≥ 9(ab + bc + ca)

1. Các bài toán BĐT qua các đề thi trong hè GV: Nguyễn Tuấn Anh
CHƯƠNG 1. BẤT ĐẲNG THỨC QUA CÁC ĐỀ THI OLYMPIC Trang 7

Theo BĐT Holder ta có:


√ √ √ 2 2
a+ b + c (a + b2 + c2 ) ≥ (a + b + c)3

nên ta cần có:


(a + b + c)3 ≥ 9(ab + bc + ca)
p
Theo giả thiết thì ta có: a + b + c = 3 + 2(ab + bc + ca). Nên BĐT cần chứng minh là:

[3 + 2 (ab + bc + ca)]3 ≥ 81(ab + bc + ca)2

⇔ 8(ab + bc + ca)3 − 45(ab + bc + ca)2 + 54 (ab + bc + ca) + 27 ≥ 0

⇔ [8 (ab + bc + ca) + 3] (ab + bc + ca − 3)2 ≥ 0

Bài 1.4 (HSG các trường chuyên khu vực Duyên Hải và ĐBBB - Lớp 10).

Với a, b, c là ba số thực dương thỏa mãn:


 
b+c c+a a+b 1 1 1
+ + =2 + +
a b c ab bc ca

Chứng minh rằng:


a2 + b2 + c2 + 3 ≥ 2(ab + bc + ca)

LỜI GIẢI. Từ giả thiết ta có:


b+c c+a a+b 1 1 1
 
+1+ +1+ +1=2 + + +3
a b c ab bc ca
11 1 2

⇔ (a + b + c) + + − =3
a b c abc

⇔ (a + b + c)(ab + bc + ac − 2) = 3abc

Theo BĐT AM - GM ta được:

(a + b + c)3
(a + b + c)(ab + bc + ac − 2) = 3abc ≤
9

(a + b + c)2
⇒ ab + bc + ac − 2 ≤ ⇔ 9(ab + bc + ca) − (a + b + c)2 ≤ 18
9

7(ab + bc + ca) + 5(a2 + b2 + c2 )


⇔ a2 + b 2 + c 2 + 3 ≥
6

GV: Nguyễn Tuấn Anh 1. Các bài toán BĐT qua các đề thi trong hè
Trang 8 CHƯƠNG 1. BẤT ĐẲNG THỨC QUA CÁC ĐỀ THI OLYMPIC

Vì a2 + b2 + c2 ≥ ab + bc + ca nên BĐT trên kéo theo:


7(ab + bc + ca) + 5(a2 + b2 + c2 )
a2 + b 2 + c 2 + 3 ≥ ≥ 2 (ab + bc + ca)
6

Bài toán được chứng minh. Đẳng thức xảy ra khi và chỉ khi a = b = c = 1. 

1. Các bài toán BĐT qua các đề thi trong hè GV: Nguyễn Tuấn Anh
CHƯƠNG 1. BẤT ĐẲNG THỨC QUA CÁC ĐỀ THI OLYMPIC Trang 9

2 Các bài toán BĐT qua các đề thi chọn đội HSGQG

Bài 1.5 (Tuyển đội HSG lớp 12 tỉnh Bà Rịa - Vũng Tàu).

Cho ba số thực dương a, b, c sao cho a2 + b2 + c2 + abc = 4. Chứng minh rằng


a b c
p +p +p ≥1
(b + 2)(c + 2) (c + 2)(a + 2) (a + 2)(b + 2)

LỜI GIẢI. Theo BĐT AM - GM ta có:

a 2a
p ≥
(b + 2)(c + 2) b+c+4

Tương tự cho hai số hạng còn lại. Vậy nên ta cần chứng minh:

2a 2b 2c
+ + ≥1
b+c+4 c+a+4 a+b+4

Theo BĐT Cauchy dạng phân thức ta có:

2a 2b 2c 2(a + b + c)2
+ + ≥
b+c+4 c+a+4 a+b+4 2 (ab + bc + ca) + 4 (a + b + c)

Vậy nên ta cần chứng minh:

2(a + b + c)2 ≥ 2 (ab + bc + ca) + 4 (a + b + c)

Theo giả thiết ta có:

2 (ab + bc + ca) = (a + b + c)2 − a2 − b2 − c2 = (a + b + c)2 − (4 − abc)

Nên BĐT trên được viết lại là:

(a + b + c)2 + 4 ≥ 4 (a + b + c) + abc

Cách 1: Vì a, b, c dương thỏa mãn a2 + b2 + c2 + abc = 4 nên tồn tại ba số thực dương
x, y, z sao cho:2
r r r
yz zx xy
a=2 ,b = 2 ,c = 2
(x + y) (x + z) (y + z) (y + x) (z + x) (z + y)

2
Lý do vì sao có phép đổi biến như vậy mời bạn đọc xem phần bổ sung kiến thức.

GV: Nguyễn Tuấn Anh 2. Các bài toán BĐT qua các đề thi chọn đội HSGQG
Trang 10 CHƯƠNG 1. BẤT ĐẲNG THỨC QUA CÁC ĐỀ THI OLYMPIC

BĐT cần chứng minh được viết lại là:


r r r r
yz zx xy 2xyz
+ + −1≥
(x + y) (x + z) (y + z) (y + x) (z + x) (z + y) (x + y) (y + z) (z + x)

p p p p √
⇔ xy (x + y) + yz (y + z) + zx (z + x) ≥ (x + y) (y + z) (z + x) + 2xyz

P√ p p
⇔2 xyz x (x + y) (x + z) ≥ 4xyz + 2 2xyz (x + y) (y + z) (z + x)
cyc

Pp √ p
⇔ x (x + y) (x + z) ≥ 2 xyz + 2 (x + y) (y + z) (z + x)
cyc

Hướng 1: (Sử dụng bổ đề quen thuộc) BĐT cần chứng minh là:
r

r r r
x+y y+z y+z z+x z+x x+y y+z z+x x+y
. + . + . ≥2+ 2 . .
z x x y y z x y z


r r r r
x y z x y z
⇔ + + ≥ 2+2 . .
y+z z+x x+y y+z z+x x+y

Không mất tính tổng quát ta giả sử z là số nhỏ nhất trong ba số x, y, z . Khi đó, ta có
kết quả sau: r r r
x y x+y
+ ≥2
y+z z+x x + y + 2z
Thật vậy, theo BĐT Holder ta có:
r r 2
x y
x2 (y + z) + y 2 (z + x) ≥ (x + y)3

+
y+z z+x

Vì z là số nhỏ nhất trong ba số nên ta được:

x2 (y + z) + y 2 (z + x) = xy (x + y − 2z) + z(x + y)2

(x + y)2 2 (x + y)2
≤ (x + y − 2z) + z(x + y) = (x + y + 2z)
4 4
Vậy nên ta có:
s s
(x + y)3 4(x + y)3
r r r
x y x+y
+ ≥ ≥ =2
y+z z+x x2 (y + z) + y 2 (z + x) (x + y)2 (x + y + 2z) x + y + 2z

Ta lại có:
xy 1 1 (x + y)2
= ≤ =
(y + z) (z + x) z (x + y + z)
+1
4z (x + y + z)
+ 1 (x + y + 2z)2
xy (x + y)2

2. Các bài toán BĐT qua các đề thi chọn đội HSGQG GV: Nguyễn Tuấn Anh
CHƯƠNG 1. BẤT ĐẲNG THỨC QUA CÁC ĐỀ THI OLYMPIC Trang 11

Bài toán được chứng minh nếu ta chứng minh được:


s
√ (x + y)2
r r
x+y z z
2 + ≥ 2+2 .
x + y + 2z x+y x + y (x + y + 2z)2

v v

r
u 1 z u z 1
⇔ 2u + ≥ 2 + 2u
ux + y .
t 2z x+y 2z
2
1+ t
1+
x+y x+y

z
Để dễ dàng, ta đặt t = . BĐT trên được viết lại là:
x+y
r
√ √
r
1 t
2 + t≥ 2+2
1 + 2t (2t + 1)2

r

r
4 t 4t t
⇔ +t+4 ≥2+ + 4 2
1 + 2t 1 + 2t (2t + 1)2 (2t + 1)2


(2t − 1) 2t2 − t − 2
r r
2t t 4t (1 − 2t)
⇔ 2
≥4 2
−4 = r
1 + 2t
r 
(2t + 1) (2t + 1) 2 2t t
(2t + 1) +
(2t + 1)2 1 + 2t

vậy nên ta cần có:


 4t
2 + t − 2t2 ≥ r r
2t t
2
+
1 + 2t
r r(2t + 1)
2t t 
⇔ 2
+ 2 + t − 2t2 ≥ 4t
(2t + 1) 1 + 2t

1
vì t ≤ nên
2 s r r r
2t t t t
2
≥ ; ≥
(2t + 1) 2 1 + 2t 2

Do đó ta cần có:
r
t 1
 
2 + t − 2t2 ≥ 2t ⇔ t t − (t − 2) 2t2 + 3t + 2 ≥ 0
 
2 2

BĐT trên đúng. Nên bài toán được chứng minh.


Hướng 2: (https://artofproblemsolving.com)
Nhắc lại ta cần chứng minh:
Xp √ p
x (x + y) (x + z) ≥ 2 xyz + 2 (x + y) (y + z) (z + x)
cyc

GV: Nguyễn Tuấn Anh 2. Các bài toán BĐT qua các đề thi chọn đội HSGQG
Trang 12 CHƯƠNG 1. BẤT ĐẲNG THỨC QUA CÁC ĐỀ THI OLYMPIC

Bình phương vế trái ta có:


!2
Pp
x (x + y) (x + z)
cyc

P p 
= x3 + x2 y + x2 z + xyz + 2 [x2 (x + y + z) + xyz] [y 2 (x + y + z) + xyz]
cyc

Theo BĐT Cauchy và Schur ta được:


P p 
x3 + x2 y + x2 z + xyz + 2 [x2 (x + y + z) + xyz] [y 2 (x + y + z) + xyz]
cyc

Cauchy 
x3 + x2 y + x2 z + xyz + 2 [xy (x + y + z) + xyz]
P

cyc


x3 + 3x2 y + x2 z + 2xy 2 + 5xyz
P
=
cyc

= x3 + y 3 + z 3 + 3 [xy (x + y) + yz (y + z) + zx (z + x)] + 15xyz

Schur
= (x + y + z)3 + 9xyz ≥ 4 (x + y + z) (xy + yz + zx)

Do đó ta cần chứng minh:


p √ p
2 (x + y + z) (xy + yz + zx) ≥ 2 xyz + 2 (x + y) (y + z) (z + x)

p
⇔ 4 (x + y + z) (xy + yz + zx) ≥ 4xyz + 2 (x + y) (y + z) (z + x) + 4 2 (x + y) (y + z) (z + x)

p
⇔ (x + y) (y + z) (z + x) ≥ 2 2xyz (x + y) (y + z) (z + x)

p √
⇔ (x + y) (y + z) (z + x) ≥ 2 2xyz

BĐT cuối hiển nhiên đúng nên bài toán được chứng minh.
Hướng 3: Nhắc lại ta cần chứng minh:
Pp √ p
x (x + y) (x + z) ≥ 2 xyz + 2 (x + y) (y + z) (z + x)
cyc
Pp √ p
⇔ x2 (x + y + z) + xyz ≥ 2 xyz + 2 (x + y) (y + z) (z + x)
cyc

Một BĐT quen thuộc khi ta xử lý BĐT chứa căn là:


p p q
a2 + b 2 + c2 + d 2 ≥ (a + c)2 + (b + d)2

2. Các bài toán BĐT qua các đề thi chọn đội HSGQG GV: Nguyễn Tuấn Anh
CHƯƠNG 1. BẤT ĐẲNG THỨC QUA CÁC ĐỀ THI OLYMPIC Trang 13

Áp dụng BĐT trên hai lần ta được:


q
(x + y)2 (x + y + z) + 4xyz +
Pp 2 p
x (x + y + z) + xyz ≥ z 2 (x + y + z) + xyz
cyc q q
2
≥ (x + y + z) (x + y + z) + 9xyz = (x + y + z)3 + 9xyz

Do đó ta cần chứng minh được rằng:


q
3 √ p
(x + y + z) + 9xyz ≥ 2 xyz + 2 (x + y) (y + z) (z + x)
⇔ (x + y + z)3 + 5xyz ≥ 4
p
2xyz (x + y) (y + z) (z + x) + 2 (x + y) (y + z) (z + x)
p
⇔ x3 + y 3 + z 3 + (x + y) (y + z) (z + x) + 5xyz ≥ 4 2xyz (x + y) (y + z) (z + x)

Theo BĐT AM - GM ta có:

x3 + y 3 + z 3 + (x + y) (y + z) (z + x) + 5xyz
p
≥ (x + y) (y + z) (z + x) + 8xyz ≥ 4 2xyz (x + y) (y + z) (z + x)

Bài toán được chứng minh.


Cách 2: (https://diendantoanhoc.net) Ta quay lại vấn đề đầu cách 1. Ta cần chứng
minh:

(a + b + c − 2)2 ≥ abc ⇔ a + b + c − 2 ≥ abc

Vì a2 + b2 + c2 + abc = 4 nên tồn tại tam giác nhọn ABC sao cho:

a = 2 cos A; b = 2 cos B; c = 2 cos C

BĐT cần chứng minh là:



cos A + cos B + cos C − 1 ≥ 2 cos A cos B cos C

Ta có:
R+r X p2 + r2 − 4R2
cos A + cos B + cos C = ; cos A cos B =
R 4R2
cyc

BĐT được viết lại là:


p2 ≤ 4R2 + 4Rr + 3r2

Đây chính là bất đẳng thức Gerretsen.


Cách 3: (https://diendantoanhoc.net) Lập luận như cách 2 ta cần chứng minh:

cos A + cos B + cos C − 1 ≥ 2 cos A cos B cos C
A B C
Vì cos A + cos B + cos C = 1 + 4 sin sin sin nên BĐT được viết lại là:
2 2 2
A B C √
4 sin sin sin ≥ 2 cos A cos B cos C ⇔ (1−cos A)(1−cos B)(1−cos C) ≥ cos A cos B cos C
2 2 2

GV: Nguyễn Tuấn Anh 2. Các bài toán BĐT qua các đề thi chọn đội HSGQG
Trang 14 CHƯƠNG 1. BẤT ĐẲNG THỨC QUA CÁC ĐỀ THI OLYMPIC

Ta có BĐT sau:3
3 3
 
3 + 2  27
(1 + cos A)(1 + cos B)(1 + cos C) ≤   =
3 8

nên ta cần chứng minh:


   27
1 − cos2 A 1 − cos2 B 1 − cos2 C ≥ cos A cos B cos C
8

cot A cot B cot C 8


⇔ ≤
sin A sin B sin C 27
Sử dụng công thức cộng đối với sin chú ý rằng sin(A + B) = sin C . Ta được:
cot A cot B cot C
= cot A cot B cot C (cot A + cot B) (cot B + cot C) (cot C + cot A)
sin A sin B sin C
Sử dụng BĐT AM - GM kết hợp với kết quả sau

cot A cot B + cot B cot C + cot C cot A = 1

ta được:

cot A cot B cot C (cot A + cot B) (cot B + cot C) (cot C + cot A)

[2 (cot A cot B + cot B cot C + cot C cot A)]3 8


≤ =
27 27
Do đó BĐT được chứng minh. 
Nhận xét:

• Hướng 1 trong cách 1, với bổ đề: x, y, z là ba số thực dương và z là số nhỏ nhất thì
r r r
x y x+y
+ ≥2
y+z z+x x + y + 2z

ta có một kết quả mạnh hơn (cách chứng minh giống như cách ta làm trong hướng
1) có thể chứng minh nhanh bài toán:
r r r r
x y z xyz
+ + ≥2 1+
y+z z+x x+y (x + y) (y + z) (z + x)

• Bổ đề trên còn có nhiều ứng dụng khác nữa mời bạn đọc xem bài "Ứng dụng một
bổ đề hay trong chứng minh BĐT - Ngô Trung Hiếu, Cao Minh Quang - THTT
469"
3 3
BĐT có được nhờ BĐT AM - GM cho 3 số kết hợp với kết quả quen thuộc cos A + cos B + cos C ≤
2

2. Các bài toán BĐT qua các đề thi chọn đội HSGQG GV: Nguyễn Tuấn Anh
CHƯƠNG 1. BẤT ĐẲNG THỨC QUA CÁC ĐỀ THI OLYMPIC Trang 15

• Đây là bài toán khá thú vị, và còn nhiều lời giải nữa, mời bạn đọc tìm đọc trong
bài viết "Xung quanh câu BĐT trong đề thi chọn HSG tỉnh Bà Rịa - Vũng Tàu".

Bài 1.6 (Lập đội tuyển dự thi quốc gia - Đăk Lăk).

Cho bất phương trình x2 − (a + b)x + ab ≤ 0 (1) trong đó a > b > 0. Gọi x1 , x2 , ..., xn là các
nghiệm của bất phương trình (1). Chứng minh rằng:

(x1 + x2 + ... + xn )2 4ab


2 2 2

n(x1 + x2 + ... + xn ) (a + b)2

LỜI GIẢI. Trước tiên ta có nhận xét các xi thỏa mãn (1) tương đương với b ≤ xi ≤ a.
BĐT cần chứng minh tương đương với:
q
(a + b)(x1 + x2 + ... + xn ) ≥ 2 abn(x21 + x22 + ... + x2n )

Vì xi thỏa mãn (1) nên ta có:

(a + b)(x1 + x2 + ... + xn ) ≥ (x21 + x22 + ... + x2n ) + nab

Theo BĐT AM - GM ta được:


q
(x21 + x22 + ... + x2n ) + nab ≥2 abn(x21 + x22 + ... + x2n )

Bài toán được chứng minh. 

Bài 1.7 (Chọn đội HSG dự thi Quốc Gia tỉnh Đồng Tháp).

Với a, b, c là các số thực dương. Chứng minh rằng:

a(b + c) b(c + a) c(a + b)


+ 2 + 2 ≥2
b2 + bc + c 2 c + ca + a 2 a + ab + b2

LỜI GIẢI.
Cách 1: (Algebraic inequalities old and new methods - Vasile Cirtoaje)
Theo BĐT Cauchy ta có:
a(b + c) b(c + a) c(a + b) (a + b + c)2
+ + ≥
b2 + bc + c2 c2 + ca + a2 a2 + ab + b2

P a b2 + bc + c2
b+c
Vậy nên ta cần chứng minh:
"   #
a b2 + bc + c2 b c2 + ca + a2 c a2 + ab + b2
(a + b + c)2 ≥ 2 + +
b+c a+c a+b

Vì 
a b2 + bc + c2
 
bc
=a b+c−
b+c b+c

GV: Nguyễn Tuấn Anh 2. Các bài toán BĐT qua các đề thi chọn đội HSGQG
Trang 16 CHƯƠNG 1. BẤT ĐẲNG THỨC QUA CÁC ĐỀ THI OLYMPIC

nên BĐT cần chứng minh được viết lại là:


 1 1 1

2 2 2
a + b + c + abc + + ≥ 2 (ab + bc + ca)
a+b b+c c+a
Theo BĐT Cauchy dạng phân thức, ta có:
 1 1 1
 9abc
abc + + ≥
a+b b+c c+a 2 (a + b + c)

Nên ta cần có:


9abc
a2 + b 2 + c 2 + ≥ 2 (ab + bc + ca)
2 (a + b + c)

⇔ a3 + b3 + c3 + 3abc ≥ ab (a + b) + bc (b + c) + ca (c + a)

BĐT cuối hiển nhiên đúng theo BĐT Schur bậc 3.


Cách 2: (Algebraic inequalities old and new methods - Vasile Cirtoaje) BĐT
cần chứng minh đương đương với:
   
P a(b + c) P a (b + c) (a + b + c)
(a + b + c) −2 ≥0⇔ − 2a ≥0
b2 + bc + c2 b2 + bc + c2

!
a ab + ac − b2 − c2
 
P P ab (a − b) − ca (c − a)
⇔ ≥0⇔ ≥0
b2 + bc + c2 b2 + bc + c2

1 1
P  
⇔ ab (a − b) 2 2
− 2 ≥0
b + bc + c c + ca + a2

P ab(a − b)2
⇔ (a + b + c) ≥0
(b2 + bc + c2 ) (c2 + ca + a2 )

Từ BĐT cuối, bài toán được chứng minh.


Cách 3: Theo BĐT AM - GM ta có:
1 2  1
2 2
b + bc + c2 + ab + bc + ca = (b + c)2 (a + b + c)2

b + bc + c (ab + bc + ca) ≤
4 4
Do vậy:
a (b + c) 4a (ab + bc + ca)

b2 + bc + c2 (b + c) (a + b + c)2
Tương tự cho các đánh giá còn lại. Vậy ta cần chứng minh:
 
ab + bc + ca a b c 1
2
+ + ≥
(a + b + c) b+c c+a a+b 2

Theo BĐT Cauchy dạng phân thức, ta có:


a b c (a + b + c)2
+ + ≥
b+c c+a a+b 2 (ab + bc + ca)

2. Các bài toán BĐT qua các đề thi chọn đội HSGQG GV: Nguyễn Tuấn Anh
CHƯƠNG 1. BẤT ĐẲNG THỨC QUA CÁC ĐỀ THI OLYMPIC Trang 17

Thay vào ta được điều phải chứng minh. 


Nhận xét: Cách 2 là ta cố gắng phân tích tổng bình phương. Ngoài cách tách như trên
ta còn có thể tách như sau:
a (b + c) ab + ac (b + c) [ab (a − b) + ac (a − c)]
− =
b2+ bc + c 2 ab + bc + ca (b2 + bc + c2 ) (ab + bc + ca)
Do đó:  
P a (b + c) P a (b + c) ab + ac
−2= −
b2 + bc + c2 2
b + bc + c 2 ab + bc + ca

P (b + c) [ab (a − b) + ac (a − c)]
=
(b2 + bc + c2 ) (ab + bc + ca)

  
P a−b ab (b + c) ba (c + a)
= . −
ab + bc + ca b2 + bc + c2 c2 + ca + a2

P ab(a − b)2
=
(b2 + bc + c2 ) (c2 + ca + a2 )
Bài 1.8 (THPT chuyên KHTN - Ngày 2).
Với a, b, c là các số thực thỏa mãn (a + b)(b + c)(c + a) 6= 0. Chứng minh rằng:

(a2 − b2 )(a2 − c2 ) (b2 − c2 )(b2 − a2 ) (c2 − a2 )(c2 − b2 )


+ + ≥0
(b + c)2 (c + a)2 (a + b)2

LỜI GIẢI. Để đơn giản, ta đặt x = a + b, y = b + c, z = c + a. Khi đó BĐT được viết lại
là:
xz (x − y) (z − y) yx (y − z) (x − z) zy (z − x) (y − x)
+ + ≥0
y2 z2 x2

yz (x − y) (x − z) zx (y − x) (y − z) xy (z − x) (z − y)
⇔ + + ≥0
x2 y2 z2
Không mất tính tổng quát ta giả sử x ≥ y ≥ z . Khi đó:
yz (x − y) (x − z)
≥0
x2
Do đó ta chỉ cần chứng minh:
zx (y − x) (y − z) xy (z − x) (z − y)
+ ≥0
y2 z2

x2 (y − z)2
 
zx (y − x) (y − z) xy (z − x) (z − y) z (y − x) y (z − x)
⇔ + = x (y − z) − = ≥0
y2 z2 y2 z2 z2
BĐT cuối hiển nhiên đúng nên bài toán được chứng minh. 
Nhận xét: BĐT cuối có thể được chứng minh bằng phép biến đổi tương đương đưa về
BĐT Schur.

GV: Nguyễn Tuấn Anh 2. Các bài toán BĐT qua các đề thi chọn đội HSGQG
Trang 18 CHƯƠNG 1. BẤT ĐẲNG THỨC QUA CÁC ĐỀ THI OLYMPIC

Bài 1.9 (THPT chuyên KHTN - Ngày 3).


Với a, b, c là các số thực dương. Chứng minh rằng:

a3 b3 c3 9 9
2 2
+ 2 2
+ 2 2
+ ≥
b − bc + c c − ca + a a − ab + b 2 (ab + bc + ca) 2

LỜI GIẢI. Trước tiên ta chứng minh:


a3 b3 c3
+ + ≥a+b+c
b2 − bc + c2 c2 − ca + a2 a2 − ab + b2
Thật vậy, theo BĐT Cauchy, ta cần chứng minh:
2 2
2 2
  2 2
  
a +b +c ≥ (a + b + c) a b − bc + c + b c2 − ca + a2 + c a2 − ab + b2

  
⇔ a4 + b4 + c4 + abc (a + b + c) ≥ ab a2 + b2 + bc b2 + c2 + ca c2 + a2
BĐT cuối là BĐT Schur bậc 4. Do đó ta cần chứng minh:
9 9
(a + b + c) + ≥
2 (ab + bc + ca) 2
Theo BĐT AM - GM, ta lại có:
s r
9 3 9(a + b + c)2 3 27 (ab + bc + ca) 9
(a + b + c) + ≥3 ≥ =
2 (ab + bc + ca) 8 (ab + bc + ca) 2 ab + bc + ca 2
Bài toán được chứng minh. Đẳng thức xảy ra khi và chỉ khi a = b = c = 1. 
Bài 1.10 (Chọn đội HSG Quốc Gia tỉnh Đăk Nông - Vòng 1).
Cho ∆ABC là tam giác nhọn. Đặt giá trị:
sin A sin B sin C
M= + +
sin B + sin C sin C + sin A sin A + sin B

Gọi [M ] là số nguyên không vượt quá M . Tính [M ].

LỜI GIẢI.
3
• Trước tiên ta chứng minh: M ≥ .
2
Theo BĐT Cauchy ta có:
sin A sin B sin C (sin A + sin B + sin C)2
+ + ≥
sin B + sin C sin C + sin A sin A + sin B 2 (sin A sin B + sin B sin C + sin C sin A)
Do đó ta cần chứng minh:
(sin A + sin B + sin C)2 3

2 (sin A sin B + sin B sin C + sin C sin A) 2

⇔ sin2 A + sin2 B + sin2 C ≥ sin A sin B + sin B sin C + sin C sin A

1
(sin A − sin B)2 + (sin B − sin C)2 + (sin C − sin A)2 ≥ 0


2

2. Các bài toán BĐT qua các đề thi chọn đội HSGQG GV: Nguyễn Tuấn Anh
CHƯƠNG 1. BẤT ĐẲNG THỨC QUA CÁC ĐỀ THI OLYMPIC Trang 19

BĐT được chứng minh.

• Ta chứng minh:

sin A sin B sin C


+ + <2
sin B + sin C sin C + sin A sin A + sin B

Ta có đánh giá
sin A 2 sin A

sin B + sin C sin A + sin B + sin C
và tương tự cho các đánh giá còn lại. Do đó:
sin A sin B sin C 2 (sin A + sin B + sin C)
+ + ≤ =2
sin B + sin C sin C + sin A sin A + sin B sin A + sin B + sin C

Tuy nhiên đẳng thức không thể xảy ra do đó đánh giá trên được chứng minh.

Từ hai đánh giá trên ta được [M ] = 1. 


Nhận xét: Bản chất bài toán trên là hai đánh giá quen thuộc của BĐT Nesbitt: Với
ba số thực dương a, b, c thì
a b c 3
• + + ≥ .
b+c c+a a+b 2
a b c
• + + < 2.
b+c c+a a+b
Dữ kiện ∆ABC nhọn chưa được sử dụng trừ công dụng là để các sin A, sin B, sin C là các
số dương.

Bài 1.11 (Chọn đội HSG Quốc Gia tỉnh Đăk Nông - Vòng 2).

Với a, b, c là các số thực dương thỏa mãn a2 + b2 + c2 + abc = 4. Chứng minh rằng:
√ √ √
a+b+c≥ a+ b+ c

Đẳng thức xảy ra khi nào?

LỜI GIẢI.
Cách 1: (Lekhanhsy - http://forum.mathscope.org/) BĐT cần chứng minh tương
đương với:
√ √ √ 
a2 + b2 + c2 + 2 (ab + bc + ca) ≥ a + b + c + 2 ab + bc + ca

√ 2 √ 2 √ 2
⇔ 1 − abc + ab + bc + ca + ab − 1 + bc − 1 + ( ca − 1) ≥ a + b + c

√ 2 √ 2 √ 2
⇔ (1 − a) (1 − b) (1 − c) + ab − 1 + bc − 1 + ( ca − 1) ≥ 0

GV: Nguyễn Tuấn Anh 2. Các bài toán BĐT qua các đề thi chọn đội HSGQG
Trang 20 CHƯƠNG 1. BẤT ĐẲNG THỨC QUA CÁC ĐỀ THI OLYMPIC

Không mất tính tổng quát ta giả sử a ≥ b ≥ c, từ đó ta được: 1 ≤ a < 2. Do vậy:


 √  √ 2
(1 − a) (1 − b) (1 − c) = (1 − a) (1 − b − c + bc) ≥ (1 − a) 1 − 2 bc + bc = (1 − a) bc − 1

Vì thế ta cần chứng minh:


√ 2 √ 2 √ 2
ab − 1 + (2 − a) bc − 1 + ca − 1 ≥0

Thế nhưng BĐT cuối là hiển nhiên nên bài toán được chứng minh. Đẳng thức xảy ra
khi và chỉ khi a = b = c = 1.
Cách 2: Áp dụng BĐT Cauchy ta có
√ √ p
a+ b≤ 2 (a + b)

Do đó ta cần chứng minh:


p √
a+b+c≥ 2 (a + b) + c
p
⇔ a2 + b2 + c2 + 2 (ab + bc + ca) ≥ 2 (a + b) + c + 2 2 (a + b) c
p
Theo BĐT AM - GM, ta có 2 2(a + b)c ≤ 2 + ac + bc nên ta chỉ cần chứng minh được:

4 − abc + 2 (ab + bc + ca) ≥ 2 (a + b) + c + ac + bc + 2


⇔ 2 − abc + 2ab + bc + ca − 2a − 2b − c ≥ 0
⇔ c (a + b − ab − 1) + 2 (1 + ab − a − b) ≥ 0
⇔ (a − 1) (b − 1) (2 − c) ≥ 0

BĐT cuối không hoàn toàn đúng. Nó chỉ đúng khi a, b cùng đồng thời bé hơn hoặc bằng
1 hoặc lớn hơn hoặc bằng 1. Vì BĐT ban đầu là đối xứng nên ta hoàn toàn có thể giả
sử ngay từ đầu điều đó. 4 Bài toán được chứng minh. Đẳng thức xảy ra khi và chỉ khi
a = b = c = 1. 

Bài 1.12 (Chọn đội HSG Quốc Gia tỉnh Hà Tĩnh, Ngày thứ nhất).

Với a, b, c là các số thực dương thỏa mãn a2 + b2 + c2 ≤ 3. Chứng minh rằng:

(a + b + c)(a + b + c − abc) ≥ 2(a2 b + b2 c + c2 a)

Đẳng thức xảy ra khi nào?

LỜI GIẢI. (Lekhanhsy - http://forum.mathscope.org/)

• Ta chứng minh
4(a + b + c)3
a2 b + b2 c + c2 a + abc ≤
27
4
Hiển nhiên 0 < a, b, c < 2

2. Các bài toán BĐT qua các đề thi chọn đội HSGQG GV: Nguyễn Tuấn Anh
CHƯƠNG 1. BẤT ĐẲNG THỨC QUA CÁC ĐỀ THI OLYMPIC Trang 21

Hướng 1: Không mất tính tổng quát ta giả sử a là số nhỏ nhất trong ba số a, b, c.
Khi đó tồn tại x, y ≥ 0 sao cho b = a + x, c = a + y . BĐT được viết lại là:

9 x2 − xy + y 2 a + (2x − y)2 (x + 4y) ≥ 0




nên BĐT trên đúng. Đẳng thức xảy ra khi và chỉ khi a = b = c = 1 hoặc a = 0, b =
1, c = 2 và các hoán vị của nó.

Hướng 2: Vì BĐT là BĐT hoán vị nên ta có thể giả sử b là ở giữa hai giá trị a, c.
Khi đó (a − b)(c − b) ≤ 0 ta được:

ac + b2 ≤ ab + cb ⇔ c2 a + b2 c ≤ abc + c2 b

Do vậy ta cần chứng minh:

2 2 4(a + b + c)3 2 4(a + b + c)3


a b + 2abc + c b ≤ ⇔ b(a + c) ≤
27 27
Theo BĐT AM -GM ta có:
4(a + b + c)3 1 [2b + (a + c) + (a + c)]3 1
= . ≥ .2b(a + c)2 = b(a + c)2
27 2 27 2
nên kết quả trên được chứng minh.

• Do đó ta chứng minh một kết quả mạnh hơn như sau:

8(a + b + c)3
(a + b + c) (a + b + c − abc) ≥ − 2abc
27

8(a + b + c)3
⇔ (a + b + c)2 − (a + b + c) abc ≥ − 2abc
27
Vì theo giả thiết ta được a + b + c ≤ 3 nên bài toán sẽ được chứng minh nếu ta
chứng minh được:

(a + b + c)3 8(a + b + c)3


− 3abc ≥ − 2abc
3 27

(a + b + c)3
⇔ ≥ abc
27
BĐT cuối đúng theo BĐT AM - GM. Bài toán được chứng minh. Đẳng thức xảy
ra khi và chỉ khi a = b = c = 1.


Nhận xét: BĐT phụ trong bài giải có hình dáng giống với bài toán mở rộng của Canada
- 1999:

GV: Nguyễn Tuấn Anh 2. Các bài toán BĐT qua các đề thi chọn đội HSGQG
Trang 22 CHƯƠNG 1. BẤT ĐẲNG THỨC QUA CÁC ĐỀ THI OLYMPIC

(Canada MO - 1999) Với ba số thực không âm a, b, c có tổng bằng 1. Chứng minh rằng:
4
a2 b + b 2 c + c 2 a ≤
27
(Canada MO - 1999 - Mở rộng) Với ba số thực không âm a, b, c. Chứng minh rằng:

2 2 2 4(a + b + c)3
a b + b c + c a + abc ≤
27
(Canada MO - 1999 - Mở rộng) Với ba số thực không âm a, b, c có tổng bằng 1. Chứng
minh rằng:
nn ∗
an b + b n c + c n a + an b n c n ≤ n+1 ; (n ∈ N )
(n + 1)

Bài 1.13 (Lập đội tuyển dự thi HSG Quốc Gia - Quảng Ninh, Ngày 1).

Với ba số thực dương a, b, c có tổng bằng 3. Chứng minh rằng:

a2 b2 c2 a2 + b2 + c2
+ + ≤√
2a + 1 2b + 1 2c + 1 a2 + b 2 + c 2 + 6

LỜI GIẢI.
Cách 1: Ta để ý:
a a a2
− =
2 2 (2a + 1) 2a + 1

Do đó bài toán được đưa về chứng minh BĐT sau:



a b c 2 a2 + b 2 + c 2
+ + +√ ≥3
2a + 1 2b + 1 2c + 1 a2 + b2 + c2 + 6

Theo BĐT Cauchy dạng phân thức ta có:

a b c 9
+ + ≥
2a + 1 2b + 1 2c + 1 2 (a2 + b2 + c2 ) + 3

Do đó ta cần có: 
9 2 a2 + b 2 + c 2
+ √ ≥3
2 (a2 + b2 + c2 ) + 3 a2 + b 2 + c 2 + 6
Hướng 1: Theo BĐT AM - GM ta chỉ cần chứng minh:
2 2 2

9 12 a + b + c
+ 2 ≥3
2 (a2 + b2+ c ) + 3 a + b2 + c2 + 15
2

2 
⇔ a2 + b 2 + c 2 ≥ 3 a2 + b 2 + c 2 ⇔ a2 + b 2 + c 2 ≥ 3

BĐT cuối là hiển nhiên do a + b + c = 3. Bài toán được chứng minh.

2. Các bài toán BĐT qua các đề thi chọn đội HSGQG GV: Nguyễn Tuấn Anh
CHƯƠNG 1. BẤT ĐẲNG THỨC QUA CÁC ĐỀ THI OLYMPIC Trang 23


Hướng 2: Ta đặt t = a2 + b2 + c2 + 6 với t ≥ 3 khi đó BĐT được viết lại là:
9 2t2 − 12
+ ≥3
2t2 − 9 t

⇔ 2t4 − 3t3 − 21t2 + 18t + 54 ≥ 0

⇔ (t − 3)2 t2 + 9t + 6 ≥ 0


BĐT cuối là hiển nhiên nên bài toán được chứng minh. Đẳng thức xảy ra khi và chỉ khi
a = b = c = 1.
Cách 2: (https://artofproblemsolving.com) Theo BĐT Cauchy ta có:
 2 2
b2 c2 a2 b2 c2
 
a 2 2 2

+ + ≤ a +b +c + +
2a + 1 2b + 1 2c + 1 (2a + 1)2 (2b + 1)2 (2c + 1)2

Dễ dàng đoán được điểm rơi của BĐT là a = b = c = 1 và BĐT là tách biến nên suy
nghĩ tự nhiên ta chọn phương pháp tiếp tuyến cho đánh giá:
a2 2 1 2 1
2
≤ (a − 1) + = a +
(2a + 1) 27 9 27 27

Do vậy:
 2 2
b2 c2
 
a 2(a + b + c) 3 1 2
≤ a2 + b 2 + c 2
a + b2 + c 2
 
+ + + =
2a + 1 2b + 1 2c + 1 27 27 3

Vậy nên ta cần chứng minh:


r
1 2 a2 + b 2 + c 2
(a + b2 + c2 ) ≤ √
3 a2 + b2 + c2 + 6

  2
⇔ a2 + b 2 + c 2 a2 + b 2 + c 2 + 6 ≤ 3 a2 + b 2 + c 2

2 2
⇔ a2 + b 2 + c 2 − 3 + a2 + b2 + c2 −9≥0

BĐT cuối hiển nhiên đúng vì:

a2 + b2 + c2 + 3 ≥ 2(a + b + c) = 6

Bài toán được chứng minh. Đẳng thức xảy ra khi và chỉ khi a = b = c = 1. 
Nhận xét: Một số bài toán tương tự:
(Võ Quốc Bá Cẩn) Với a, b, c là các số thực dương có tổng bằng 1 thì:

a2 b2 c2 1
+ + ≤
3a + 1 3b + 1 3c + 1 18 (ab + bc + ca)

GV: Nguyễn Tuấn Anh 2. Các bài toán BĐT qua các đề thi chọn đội HSGQG
Trang 24 CHƯƠNG 1. BẤT ĐẲNG THỨC QUA CÁC ĐỀ THI OLYMPIC

(Võ Quốc Bá Cẩn) Với a, b, c là các số thực dương có tổng bằng 1 thì:

a2 b2 c2 1
+ + ≤ p
9a + 1 9b + 1 9c + 1 12 3 (ab + bc + ca)

Bài 1.14 (Chọn đội dự thi HSGQG - Nam Định, ngày 2).

Xét ba số a, b, c ∈ [0; 1]. Tìm GTLN của biểu thức:

a b c
P = + + + (1 − a)(1 − b)(1 − c)
b+c+1 c+a+1 a+b+1

LỜI GIẢI. Ta sẽ chứng minh P ≤ 1 theo 2 cách sau:


Cách 1: (https://diendantoanhoc.net) BĐT cần chứng minh tương đương với:
a b c
f (a) = + + + (1 − a)(1 − b)(1 − c) − 1 ≤ 0
b+c+1 c+a+1 a+b+1
Ta có:
2b 2c
f 00 (a) = 3
+ ≥0
(c + a + 1) (a + b + 1)
Do vậy:
f (a) ≤ max {f (0) ; f (1)}
b c bc (bc − 1)
• f (0) = + + (1 − b) (1 − c) − 1 = ≤0
c+1 b+1 (b + 1) (c + 1)

1 b c b (b + 2) (b − 1) + c (c − 1) (c + 2)
• f (1) = + + −1= ≤0
b+c+1 c+2 b+2 (b + c + 1) (b + 2) (c + 2)
BĐT được chứng minh.
Cách 2: (https://diendantoanhoc.net) Không mất tính tổng quát ta giả sử a ≥ b ≥ c
khi đó:
a b c a b c 1−a
+ + ≤ + + =1−
b+c+1 c+a+1 a+b+1 b+c+1 b+c+1 b+c+1 b+c+1
Vậy ta cần chứng minh:
1−a
(1 − a) (1 − b) (1 − c) ≤
b+c+1

⇔ (1 − b) (1 − c) (b + c + 1) ≤ 1

Theo BĐT AM- GM ta lại có:


 3
1−b+1−c+b+c+1
(1 − b) (1 − c) (b + c + 1) ≤ =1
3

BĐT được chứng minh xong. 

2. Các bài toán BĐT qua các đề thi chọn đội HSGQG GV: Nguyễn Tuấn Anh
CHƯƠNG 1. BẤT ĐẲNG THỨC QUA CÁC ĐỀ THI OLYMPIC Trang 25

Nhận xét:

• Nếu thực hiện tương tự xét hàm cho biến b, c như đã làm cho a thì ta chỉ cần chứng
minh:
f (x, y, z) − 1 ≤ 0

với x, y, z là 0 hoặc 1.

• Kỹ thuật trên dựa vào tính chất của hàm số lồi, lõm. Mà đơn cử là hàm số bậc
nhất (cũng có tính chất tương tự vậy mặc dù không là hàm lồi hay lõm) và hàm
số bậc hai (Có thể tham khảo bài viết ” Một tính chất thú vị của tam thức bậc
hai và nhị thức bậc nhất - Võ Quốc Bá Cẩn ” THTT - số 444,).

• Thay vì tìm GTLN, bài toán chứng minh P ≤ 1 là đề thi chọn đội tuyển Nguyễn Du
(Đăk Lăk) vòng 2 năm học 2016 − 2017. Và đây là đề USA Mathematical Olympiad
1980.

Bài 1.15 (Chọn đội dự thi HSGQG - An Giang - Ngày 1).

Với hai số thực x, y thỏa mãn


p p
(x2 + 1)(1 − y) − 2 = x4 (1 − y)2 + 1 − y 2 + 2y + 2

Tìm GTNN của biểu thức xy − x.

LỜI GIẢI. Ta thấy biểu thức giả thiết khá đặc biệt, nếu ta đã quen với kỹ năng sử
dụng lượng liên hợp dể giải một phương trình thì không khó để xử lý giải thiết này. Cụ
thể: q
x4 (1 − y)2 + 1 −
p
(x2 + 1)(1 − y) − 2 = y 2 + 2y + 2

q q
⇔ x2 (1 − y) − (y + 1) = x4 (1 − y)2 +1− (y + 1)2 + 1 (1)

1 1
⇔ q =q
x2 (1 − y) + x4 (1 − y)2 + 1 (y + 1)2 + 1 + y + 1

q q
⇔ x2 (1 − y) + x4 (1 − y)2 +1= (y + 1)2 + 1 + y + 1 (2)

Từ (1) và (2) dẫn đến


x2 (1 − y) = y + 1

Điều này cho ta:


x2 − 1
x2 (1 − y) = y + 1 ⇔ y =
x2 + 1

GV: Nguyễn Tuấn Anh 2. Các bài toán BĐT qua các đề thi chọn đội HSGQG
Trang 26 CHƯƠNG 1. BẤT ĐẲNG THỨC QUA CÁC ĐỀ THI OLYMPIC

−2x
Khi đó biểu thức cần tìm GTNN được viết lại là: P =
x2 + 1
Ta thấy
−2x (x − 1)2
≥ −1 ⇔ 2 ≥0
x2 + 1 x +1
Đẳng thức xảy ra khi và chỉ khi x = 1 và khi đó y = 0. Do đó GTNN của P = xy − x là
−1. 
Nhận xét: Lời giải trên dù tìm ra được GTNN của P nhưng trong đoạn lập luận chỉ
ra rằng x2 (1 − y) = y + 1 là phép suy ra không phải tương đương từ giả thiết. Điều đó
có nghĩa có thể làm cho miền của x, y nới rộng thêm. Tuy nhiên mai mắn là điểm rơi
của bài toán không rơi vào phần nới rộng (nếu có) đó. Đây là điểm chú ý khi khai thác
một BĐT từ một giả thiết là một đẳng thức phức tạp.
Để chứng minh kết quả đó là tương đương với giả thiết bài toán, người viết mời bạn
đọc chứng minh. Để đến với kết quả x2 (1 − y) = y + 1 ta có thể xử dụng công cụ hàm số

f (t) = t2 + 1 − t, và khi đó kết quả là tương đương với điều kiện ban đầu.

Bài 1.16 (Chọn đội dự thi HSGQG - An Giang - Ngày 2).

Với a, b, c là các số thực thỏa mãn a2 + b2 + c2 = 1. Tìm GTLN, GTNN của biểu thức:

P = a + b + c + ab + bc + ca

LỜI GIẢI. Theo giả thiết ta có (a + b + c)2 = 1 + 2(ab + bc + ca)

• Tìm GTNN: Theo giả thiết ta có:

(a + b + c)2 − 1 (a + b + c + 1)2 − 2
P = (a + b + c) + = ≥ −1
2 2
Đẳng thức xảy ra khi a = −1, b = c = 0 và các hoán vị của nó, nên GTNN của P
là −1.

• Tìm GTLN: Theo giả thiết ta lại có:


p p √
1 + 2 (a2 + b2 + c2 )+ a2 + b2 + c2 =

P = 1 + 2 (ab + bc + ca)+ab+bc+ca ≤ 3+1
1 √
Đẳng thức xảy ra khi a = b = c = √ nên GTLN của P là 3 + 1.
3


Bài 1.17 (Chọn đội dự thi HSGQG - Phú Thọ - Ngày 1).

Cho n số thực dương a1 , a2 , . . . , an (n ≥ 2). Gọi a = min {a1 , a2 , ..., an }. Chứng minh rằng:

a1 a2 an (a1 − a)2 + (a2 − a)2 + · · · + (an − a)2


+ + ··· + ≤n+
a2 a3 a1 a2

2. Các bài toán BĐT qua các đề thi chọn đội HSGQG GV: Nguyễn Tuấn Anh
CHƯƠNG 1. BẤT ĐẲNG THỨC QUA CÁC ĐỀ THI OLYMPIC Trang 27

LỜI GIẢI. BĐT đã cho tương đương với:


a1 a2 an (a1 − a)2 + (a2 − a)2 + · · · + (an − a)2
+ + ··· + ≤n+
a2 a3 a1 a2

a1 − a2 a2 − a3 an − a1 (a1 − a)2 + (a2 − a)2 + · · · + (an − a)2


⇔ + + ... + ≤
a2 a3 a1 a2

a1 − a a − a2 an − a a − a1 (a1 − a)2 + (a2 − a)2 + · · · + (an − a)2


⇔ + + ... + + ≤
a2 a2 a1 a1 a2
   
1 1 a1 − a 1 1 a2 − a
⇔ (a1 − a) − − + (a2 − a) − − +
a2 a1 a2 a3 a2 a2

 
1 1 an − a
+ (an − a) − − ≤0
a1 an a2

a1 a2 − a2 a2 − a21 a2 + a1 a2 a a2 a2 − a3 a2 − a22 a3 + a2 a3 a
   
⇔ (a1 − a) + (a2 − a) +
a2 a1 a2 a2 a2 a3

an a2 − a1 a2 − a2n a1 + an a1 a
 
+ (an − a) ≤0
a2 an a1

Bài toán sẽ được chứng minh nếu ta chứng minh được:


ai a2 − ai+1 a2 − a2i ai+1 + ai ai+1 a (ai+1 − a) aai + ai+1 (ai − a)2
=− ≤0
a2 ai ai+1 a2 ai ai+1
(với i = 1, 2, ..., n với quy ước an+1 = a1 )
BĐT trên là hiển nhiên vì a là số nhỏ nhất. 

Bài 1.18 (Chọn đội dự thi HSGQG - Lâm Đồng).



Với ba số thực dương a, b, c thỏa mãn a3 + b2 + c = 1 + 2 3. Tìm GTNN của biểu thức:
1 1 1
P = + 2+ 3
a b c

LỜI GIẢI.
Cách 1: Ta sẽ chọn điểm rơi giả định là a = x; b = y; c = z khi đó:
3 a3

4

 + 4 ≥ 3


 a x x




 2 1 b 2
+ ≥
 b2 y 4 y 2






 1 + 3c ≥ 4


3 c 4 3 z z

GV: Nguyễn Tuấn Anh 2. Các bài toán BĐT qua các đề thi chọn đội HSGQG
Trang 28 CHƯƠNG 1. BẤT ĐẲNG THỨC QUA CÁC ĐỀ THI OLYMPIC

a3 b2
 
1 1 1 3c 4 2 4
⇒ + 2+ 3+ 4
+ 4
+ 4 ≥ 3
+ 2+ 3
a b c 3x y z 3x y z
Vậy ta cần chọn x, y, z sao cho:
 √
 x3 + y 2 + z = 2 3 + 1  √ √
4
z4 ⇔ x = 1; y = 3; z = 3
 3x4 = y 4 =
3

Khi đó √
1 1 1 4 3
P = + 2 + 3 ≥1+
a b c 9

4 3 √ √
Vậy GTNN của P là 1 + đạt được khi a = 1; b = 4 3; c = 3.
9
Cách 2: (Lekhanhsy - http://forum.mathscope.org/)
Theo BĐT AM - GM ta có:
1 1 1 1
3
+√ +√ ≥
z 27 27 z
Do đó
2 1 1 1 Cauchy 1 4 1 4
P+ √ ≥ + 2+ ≥ + 2 = + √
3 3 x y z x y +z x 1 + 2 3 − x3
Ta chứng minh
1 4 2
+ √ ≥1+ √
x 1+2 3−x 3 3
Thật vậy, điều đó tương đương với:
1 4 2
+ √ ≥1+ √
x 1 + 2 3 − x3 3

√ √ √  √ 
⇔ 1 + 2 3 − x3 + 4x 3 ≥ 3 + 2 −x4 + x + 2 3x

√ √ √  √ 
⇔6+ 3− 3x3 + 3 + 2 x4 − 3+8 x≥0

 √ √ √
⇔ (x − 1)2
  
3 + 2 x2 + 3+4 x+ 3+6 ≥0

4 √
4

Hay nói cách khác P ≥ 1 + √ . Đẳng thức xảy ra khi và chỉ khi x = 1, y = 3, z = 3.
3 3
4
Do đó GTNN của P là 1 + √ 
3 3
Nhận xét: Bài toán đơn thuần là kiểm tra kỹ thuật chọn điểm rơi trong BĐT AM
-GM. Trùng hợp đây là đề chọn đội của Tỉnh Bình Dương năm 2016.

2. Các bài toán BĐT qua các đề thi chọn đội HSGQG GV: Nguyễn Tuấn Anh
CHƯƠNG 1. BẤT ĐẲNG THỨC QUA CÁC ĐỀ THI OLYMPIC Trang 29

Bài 1.19 (Chọn đội dự thi HSGQG - Bắc Giang).

Với ba số thực dương a, b, c thỏa mãn ab + bc + ca = 3abc. Chứng minh rằng:


s s s
a2 + b2 b2 + c 2 c2 + a2 √ √ √ √
+ + + 3 ≤ 2( a + b + b + c + c + a)
a+b b+c c+a

LỜI GIẢI. Ta có:


r r
 a 2 + b2 a+b (a − b)2
− =

 "r #
a+b 2
 r


 a 2 + b2 a + b

 2 (a + b) +


 a+b 2

 r
(a − b)2
r
a + b 2ab





 = "r #
2 a+b
 r

 a + b 2ab

 2 (a + b) +
2 a+b

Do đó BĐT được viết lại là:


r
X (a − b)2 X (a − b)2 X 2ab
"r # +3≤ "r #+
a+b
r r
cyc a2 + b 2 a+b cyc a+b 2ab cyc
2 (a + b) + 2 (a + b) +
a+b 2 2 a+b

Hiển nhiên
(a − b)2 (a − b)2
"r r #≤ "r r #
a2 + b 2 a+b a+b 2ab
2 (a + b) + 2 (a + b) +
a+b 2 2 a+b

(tương tự cho các đánh giá còn lại) Vậy nên ta cần chứng minh:
r r r
X 2ab √ X 1 p X 1 √
≥3⇔ 2abc ≥3⇔ 2 (ab + bc + ca) ≥3 3
a+b ab + bc ab + bc
cyc cyc cyc

Theo BĐT AM - GM ta có:


p q p


 2 (ab + bc + ca) ≥ 3 3 (ab + bc) (bc + ca) (ca + ab)


r r

 P 1 1

 ≥36
ab + bc (ab + bc) (bc + ca) (ca + ab)

cyc

Do đó BĐT là hiển nhiển. Bài toán được chứng minh. 


Nhận xét:

• Biểu thức được thêm bớt như bên trên bản chất xuất phát từ ý tưởng phân tích
bình phương, điều này không khó nếu bạn đọc đã quen với phương pháp SOS.

GV: Nguyễn Tuấn Anh 2. Các bài toán BĐT qua các đề thi chọn đội HSGQG
Trang 30 CHƯƠNG 1. BẤT ĐẲNG THỨC QUA CÁC ĐỀ THI OLYMPIC

• Điểm mấu chốt của bài toán là hướng đến đánh giá:
r
X 2ab
≥3
a+b
cyc

đánh giá này có thể thu được dễ dàng hơn, nếu bạn đọc đã đọc qua bài giải HSG
lớp 10 KHTN năm 2016. 5

Với ba số thực dương a, b, c thỏa mãn a + b + c = abc. Chứng minh rằng:


√ √ √ √ √
a2 + b 2 + b2 + c 2 + c2 + a2 + 3 6 ≤ 8abc

Cụ thể,
√ √ p √
a2 + b2 + 2ab ≤ 2 (a2 + b2 + 2ab) = 2 (a + b)
r r
a2 + b 2 2ab p
⇒ + ≤ 2 (a + b)
a+b a+b

Bài 1.20 (Trường đông toán học - Đà Nẵng).

Với ba số thực dương a, b, c có tổng bằng bằng 3. Chứng minh rằng:


 
9 a b c
≥2 + + +3
abc b c a

LỜI GIẢI. (Ban tổ chức)


Không mất tính tổng quát ta giả sử a là số lớn nhất tỏng ba số. Trước hết ta biến đổi
biểu thức như sau:
 
9 a b c
≥2 + + +3
abc b c a

⇔ 6 a2 c + b2 a + c2 b + 9abc ≤ 27 = (a + b + c)3 = a3 + b3 + c3 + 3 (a + b) (b + c) (c + a)



⇔ a3 + b3 + c3 − 3abc + 3 a2 b − ab2 + b2 c − c2 b + c2 a − a2 c ≥ 0

1
(a + b + c) (a − b)2 + (b − c)2 + (c − a)2 + (a − b)3 + (b − c)3 + (c − a)3 ≥ 0
 

2
Ta thấy ba đại lượng a − b, b − c, c − a rất đặc biệt là tổng bằng 0. Do đó:

(a − b)3 + (b − c)3 + (c − a)3 = 3 (a − b) (b − c) (c − a)

Vậy nên ta cần chứng minh:

(a + b + c) (a − b)2 + (b − c)2 + (c − a)2 + 6 (a − b) (b − c) (c − a) ≥ 0


 

5
Trong "BĐT qua các đề chọn đội tuyển năm học 2016 - 2017 - Nguyễn Tuấn Anh", trang 10

2. Các bài toán BĐT qua các đề thi chọn đội HSGQG GV: Nguyễn Tuấn Anh
CHƯƠNG 1. BẤT ĐẲNG THỨC QUA CÁC ĐỀ THI OLYMPIC Trang 31

Ta lại có:
1 1
(a − b)2 + (b − c)2 ≥ (a − b + b − c)2 = (a − c)2
2 2

1 1 1
(a − b) (b − c) ≤ (a − b + b − c)2 = (a − c)2 ⇔ (a − b) (b − c) (c − a) ≥ (c − a)3
4 4 4

Do đó ta cần chứng minh:


 
3 (a + b + c) 3 3b + 6c
+ (c − a) (c − a)2 ≥ 0 ⇔ (c − a)2 ≥ 0
2 2 2

BĐT cuối là hiển nhiên nên bài toán được chứng minh. 
Nhận xét: Một lần nữa ta nhắc đến kỹ thuật 6 được sử dụng như trong hướng giải 4
Olympic chuyên KHTN.

Bài 1.21 (Chọn đội dự thi HSGQG - Cà Mau).

Với các số thực a, b thỏa mãn:

a2 + b2 = 5, a − b − 3 ≥ 0
1 1 2
Tìm GTNN và GTLN của biểu thức P = + − .
a b 5−a−b

LỜI GIẢI. Từ giả thiết ta có:

(a + b)2 = 2 a2 + b2 − (a − b)2 ≤ 10 − 9 = 1



a2 + b2 − (a − b)2 5−9
ab = ≤ = −2
2 2
• Ta tìm GTNN. Ta sẽ chứng minh: P ≥ −1. Thật vậy:
1 1 2
+ − +1
a b 5−a−b

5 (a + b) − 2ab − a2 + b2 − 2ab + 5ab − ab (a + b)
⇔ ≥0
ab (5 − a − b)

5 (a + b − 1) − ab (a + b − 1)
⇔ ≥0
ab (5 − a − b)

(5 − ab) (a + b − 1)
⇔ ≥0
ab (5 − a − b)
6
"Một kỹ thuật nhỏ để sử dụng BĐT Cauchy Schwarz - Võ Quốc Bá Cẩn".

GV: Nguyễn Tuấn Anh 2. Các bài toán BĐT qua các đề thi chọn đội HSGQG
Trang 32 CHƯƠNG 1. BẤT ĐẲNG THỨC QUA CÁC ĐỀ THI OLYMPIC

BĐT cuối là hiển nhiên vì (


ab ≤ −2 < 0
a+b≤1

Đẳng thức xảy ra khi và chỉ khi:



 a+b=1

 (
a=2
a2 + b 2 = 5 ⇔


 a−b−3≥0 b = −1

Do đó GTNN của P là −1.


1
• Ta tìm GTLN. Ta sẽ chứng minh P ≤ . Cụ thể:
6
1 1 2 1
+ − − ≤0
a b 5−a−b 6

6 a2 + b2 − 30 (a + b) + 29ab − ab (a + b)
⇔ ≥0
6ab (5 − a − b)

30 − 30 (a + b) + 30ab − ab (a + b + 1)
⇔ ≥0
6ab (5 − a − b)

−30 (a + b + 1) + 30 (ab + 2) − ab (a + b + 1)
⇔ ≥0
6ab (5 − a − b)

− (30 + ab) (a + b + 1) + 20 (ab + 2)


⇔ ≥0
6ab (5 − a − b)

BĐT là đúng vì: (


a+b+1≥0
ab + 2 ≤ 0

Đẳng thức xảy ra khi và chỉ khi





 a + b = −1
 (
 a2 + b 2 = 5

a=1



 ab = −2 b = −2

 a−b−3≥0

1
Do đó GTLN của P là .
6


2. Các bài toán BĐT qua các đề thi chọn đội HSGQG GV: Nguyễn Tuấn Anh
CHƯƠNG 1. BẤT ĐẲNG THỨC QUA CÁC ĐỀ THI OLYMPIC Trang 33

3 Các bài toán BĐT qua các đề thi chọn HSG cấp tỉnh
Bài 1.22 (Đề thi học sinh giỏi lớp 12 TP Hà Nội).

Cho hàm số f : R → R thỏa mãn điều kiện


1  π π
f (tan x) = sin 2x − cos 2x, ∀x ∈ − ;
2 2 2
Tìm giá trị lớn nhất và giá trị nhỏ nhất của biểu thức f (sin2 x)f (cos2 x), ∀x ∈ R
 π π
LỜI GIẢI. Với mỗi x ∈ − ; tồn tại t ∈ R sao cho t = tan x và ngược lại. Do đó ta
2 2
có thể viết lại giả thiết như sau:
1 2t 1 − t2 t2 + t − 1
f (t) = − = , ∀t ∈ R
2 1 + t2 1 + t2 1 + t2

Do đó biểu thức cần tìm GTNN, GTLN là:


sin4 x + sin2 x − 1 cos4 x + cos2 x − 1
P = .
1 + sin4 x 1 + cos4 x

Hướng 1: Sử dụng công thức hạ bậc ta được:



1
sin4 + sin2 − 1 cos4 + cos2 − 1 =
  

 cos2 4x − 34 cos 4x − 31

 64

 1 + sin4 x 1 + cos4 x = 1 cos2 4x + 14 cos 4x + 113



   

64
Vậy nên ta cần tìm GTLN, GTNN của biểu thức:
cos2 4x − 34 cos 4x − 31
P =
cos2 4x + 14 cos 4x + 113
Đặt u = cos 4x với −1 ≤ u ≤ 1 ta cần tìm GTLN, GTNN của hàm số:
u2 − 34u − 31
g(u) =
u2 + 14u + 113
Bằng cách khảo sát hàm số, ta được:
−1 1
≤ g(u) ≤
2 25
−1 1
hay nói cách khác ≤P ≤ . Đẳng thức có thể xảy ra từ đó có được kết luận cho
2 25
bài toán.
Hướng 2: (Võ Quốc Bá Cẩn) Từ đẳng thức quen thuộc sin2 x + cos2 = 1. Nên suy
nghĩ tự nhiên ta sẽ đặt a = sin2 x, b = cos2 x khi đó a + b = 1 và biểu thức trên được rút
gọn lại là:
u2 + 4u − 1
P =
u2 − 2u + 2

GV: Nguyễn Tuấn Anh 3. Các bài toán BĐT qua các đề thi chọn HSG cấp tỉnh
Trang 34 CHƯƠNG 1. BẤT ĐẲNG THỨC QUA CÁC ĐỀ THI OLYMPIC

 2
a+b 1
với o ≤ u = ab ≤ = . Bằng công cụ đạo hàm ta cũng tìm được GTLN, GTNN
2 4
như hướng 1. 
Nhận xét: Khi ta đoán được GTLN, GTNN hoặc có thể tìm được bằng công cụ đạo
hàm như trên, thì có thể chứng minh lại đơn giản như sau (Đơn cử ta lấy hướng 1 để
xử lý, hướng 2 bạn đọc hãy làm tương tự): 7

−1
• Ta chứng minh: P ≥ . Điều đó tương đương với:
2
cos2 4x − 34 cos 4x − 31 −1
2
≥ ⇔ 3cos2 4x − 54 cos 4x + 51 ≥ 0
cos 4x + 14 cos 4x + 113 2

⇔ 3 (cos x − 1) (cos x − 17) ≥ 0

1
• Ta cũng chứng minh P ≤ tương tự như trên:
25
cos2 4x − 34 cos 4x − 31 1
2
≤ ⇔ 24cos2 4x − 864 cos 4x − 888 ≤ 0
cos 4x + 14 cos 4x + 113 25

⇔ 24 (cos x + 1) (cos x − 37) ≤ 0

Bài 1.23 (Chọn HSG lớp 12 Vòng 1, Bảng A - Long An).

Với a, b, c là các số thực dương thỏa mãn a + b + c = 2017. Tìm giá trị lớn nhất của biểu
thức:
a b 4c
P = + +
a+1 b+1 c+1

LỜI GIẢI. Ta có:


a + 1 − 1 b + 1 − 1 4 (c + 1) − 4 1 1 2 2
 
P = + + =6− + + +
a+1 b+1 c+1 a+1 b+1 c+1 c+1

Theo BĐT Cauchy ta có:


1 1 2 2 42 16 4
+ + + ≥ = =
a+1 b+1 c+1 c+1 c+1 2017 + 3 505
a + b + 2 + 2.
2

Do đó:
4 3026
P ≤6− =
505 505
Đẳng thức xảy ra khi và chỉ khi a = b = 505, c = 1009. 
7
Đương nhiên bước làm này chỉ phù hợp cho thi cử, còn việc học và luyện tập hạn chế (trừ trường hợp đã
rành kỹ thuật) các bước như vậy, vì lời giải như vậy thiếu tự nhiên và ẩn sau nó là một bài suy luận dài trong
khi những bước suy luận đó mới là điều đáng học.

3. Các bài toán BĐT qua các đề thi chọn HSG cấp tỉnh GV: Nguyễn Tuấn Anh
CHƯƠNG 1. BẤT ĐẲNG THỨC QUA CÁC ĐỀ THI OLYMPIC Trang 35

Bài 1.24 (Chọn HSG lớp 12 Vòng 1, Bảng A - Long An).

Với a, b, c là các số thực dương. Tìm giá trị nhỏ nhất của biểu thức:
2 3
P = √ √
3
− √
a+ ab + abc a+b+c

LỜI GIẢI. Theo BĐT AM - GM ta có:


a + 4b √ a + 4b + 16c √
3
≥ 2 ab; ≥ 4 abc
2 3
nên ta được:
3 3
P ≥ −√
2 (a + b + c) a+b+c
Ta sẽ chứng:
3 3 −3
−√ ≥
2 (a + b + c) a+b+c 2

√ √ 2
3 − 6 a + b + c + 3 (a + b + c) a+b+c−1
⇔ ≥0⇔ ≥0
2 (a + b + c) 2 (a + b + c)
−3
 16 4 1

Do đó: P ≥ . Đẳng thức xảy ra khi và chỉ khi (a, b, c) = , , . Hay nói cách
2 21 21 21
−3
khác GTNN của P là . 
2
Nhận xét:

• Đây cũng là một câu trong đề thi chọn học sinh giỏi lớp 12 của Tỉnh Bình Dương.

• Bài toán tương tự: Đề thi chọn HSG cấp Tỉnh - Bến Tre năm học 2016 − 2017
Với ba số thực dương a, b, c. Tìm GTNN của biểu thức:
1344 2016
P = √ √
3
−√
a + ab + abc a+b+c

Bài 1.25 (Chọn HSG lớp 12 Vòng 1, Bảng B - Long An).

Với a, b, c là các số thực dương thỏa mãn a2 +b2 +c2 = 3. Tìm giá trị nhỏ nhất của biểu thức:
1 1 1
P =√ +√ +√
1 + 8a 3 1 + 8b 3 1 + 8c3

LỜI GIẢI.
Cách 1: Theo BĐT AM - GM
√ p 1 
1 + 8a3 = (1 + 2a) (1 − 2a + 4a2 ) ≤ (1 + 2a) + 1 − 2a + 4a2 = 1 + 2a2
2

1 1
⇒√ ≥
1 + 8a3 1 + 2a2

GV: Nguyễn Tuấn Anh 3. Các bài toán BĐT qua các đề thi chọn HSG cấp tỉnh
Trang 36 CHƯƠNG 1. BẤT ĐẲNG THỨC QUA CÁC ĐỀ THI OLYMPIC

Do đó:
1 1 1
P ≥ 2
+ 2
+
1 + 2a 1 + 2b 1 + 2c2
Theo BĐT Cauchy ta có:
1 1 1 9
P ≥ + + ≥ =1
1 + 2a2 1 + 2b2 1 + 2c2 3 + 2 (a2 + b2 + c2 )

Đẳng thức xảy ra khi và cỉ khi a = b = c = 1. Tức là GTNN của P là 1.


Cách 2: Từ ý tưởng xử lý của bài giải đề Tỉnh Lạng Sơn kết hợp với giả thiết bài toán,
ta cần chọn x, y cho đánh giá sau luôn đúng:
1
√ ≥ xa2 + y
1 + 8a3

và đẳng thức xảy ra khi a = 1. Khi đó ta nên chọn x, y thỏa mãn:


−2
 
1

 x+y = 
 x=

 3 
 9

 −4 = 2x  y=5

 

 
9 9

Kiểm tra lại đẳng thức trên mời bạn đọc kiểm tra bằng phép biến đổi tương đương.
Khi đó ta được:
2 2  15
P ≥− a + b2 + c 2 + =1
9 9
Đẳng thức xảy ra khi và chỉ khi a = b = c = 1. 

Bài 1.26 (HSG cấp tỉnh - Hải Dương).

Với a, b, c là ba số thực dương. Tìm giá trị nhỏ nhất của biểu thức
1 8
P = √ −q
2a + b + 8bc 2b2 + 2(a + c)2 + 5

LỜI GIẢI. Ta có:


1 8
P = √ −q
2a + b + 8bc 2b2 + 2(a + c)2 + 5

1 8
≥ −q
2a + b + 2c + b
2b2 + 2(a + c)2 + 5

1 8
≥ −q
2 (a + b + c)
(a + b + c)2 + 5

3. Các bài toán BĐT qua các đề thi chọn HSG cấp tỉnh GV: Nguyễn Tuấn Anh
CHƯƠNG 1. BẤT ĐẲNG THỨC QUA CÁC ĐỀ THI OLYMPIC Trang 37

Để đơn giản ta đặt t = a + b + c, t > 0 và khảo sát hàm số


1 8
f (t) = −√
2t 2
t +5
Bằng công cụ đạo hàm ta tìm được:

min f (t) = f (t0 )


t∈(0;+∞)
r
5
với t0 = √
3
. Do đó ta được:
4 4−1
r
5
P ≥ f (t0 ) ; t0 = √
3
4 4−1
Đẳng thức xảy ra khi và chỉ khi
r
1 5
b = 2a = 2c = √
2 3
4 4−1
Từ đó ta có câu trả lời cho bài toán. 

Bài 1.27 (HSG lớp 12 - Quảng Trị).


Với a, b, c là ba số thực dương có tích bằng 1. Tìm giá trị lớn nhất của biểu thức
 
a b c
P = (a + b + c) 6 − − −
b c a

LỜI GIẢI. Ta khai triển P như sau:


 
a b c
P = (a + b + c) 6 − − −
b c a


= 5 (a + b + c) − a3 c + b3 a + c3 b + a2 b2 + b2 c2 + c2 a2

 
= 5 (a + b + c) − a2 c (a + c) + b2 a (b + a) + c2 b (c + b)

 
= 6 (a + b + c) − a2 c (a + b + c) + b2 a (a + b + c) + c2 b (a + b + c)


= 6 (a + b + c) − (a + b + c) a2 c + b2 a + c2 b

Theo BĐT AM - GM ta có:



3
a2 c + a2 c + b2 a ≥ 3 a3 a2 b2 c2 = 3a

Do đó:
P ≤ 6 (a + b + c) − (a + b + c)2 = −(a + b + c − 3)2 + 9 ≤ 9

Đẳng thức xảy ra khi và chỉ khi a = b = c = 1. Do đó GTLN của P là 9. 

GV: Nguyễn Tuấn Anh 3. Các bài toán BĐT qua các đề thi chọn HSG cấp tỉnh
Trang 38 CHƯƠNG 1. BẤT ĐẲNG THỨC QUA CÁC ĐỀ THI OLYMPIC

Nhận xét:

• Các phép biến đổi trong bài toán hướng đến đánh giá:

P ≤ 6 (a + b + c) − (a + b + c)2

Đánh giá này có thể thu được trực tiếp từ bài toán dựa vào kết quả quen thuộc
sau:
a b c Cauchy (a + b + c)2
+ + ≥ =a+b+c
b c a a+b+c
• Bài toán mở rộng sau của bạn Lekhanhsy - http://forum.mathscope.org/
Với a, b, c là các số thực dương có tích bằng 1. Tìm GTLN của
 
p
2 2 2
a b c
P = 3 (a + b + c ) 6 − − −
b c a

GTLN của bài toán mở rộng có thể tìm dựa vào đánh giá:
 2
a b c 2 2 2

+ + ≥3 a +b +c
b c a

  
⇔ a4 c 2 + b 4 a2 + c 4 b 2 + 2 a2 b + b 2 c + c 2 a ≥ 3 a2 + b 2 + c 2

Theo BĐT AM - GM ta có:

a4 c2 + a2 b + a2 b ≥ 3a2

tương tự cho các đánh giá còn lại. Do vậy đánh giá trên là đúng. Khi đó:
 
p
2 2 2
a b c
P = 3 (a + b + c ) 6 − − −
b c a

 
p a b c p p 
=6 3 (a2 + b2 + c2 ) − + + 3 (a2 + b2 + c2 ) ≤ 6 3 (a2 + b2 + c2 ) − 3 a2 + b2 + c2
b c a

p 2
=− 3 (a2 + b2 + c2 ) − 3 +9≤9

a b c
• Liên quan đến đại lượng + + , ta có kết quả khá chặt sau:8
b c a

21 a2 + b2 + c2
 
a b c
2 + + +1≥
b c a (a + b + c)2
với BĐT này ta có thể tìm GTLN của P trong tình huống mở rộng, thông qua kết
quả 9
p
a b c 3 3 (a2 + b2 + c2 )
+ + ≥
b c a a+b+c
8
Võ Quốc Bá Cẩn
9
Vasile Cirtoaje

3. Các bài toán BĐT qua các đề thi chọn HSG cấp tỉnh GV: Nguyễn Tuấn Anh
CHƯƠNG 1. BẤT ĐẲNG THỨC QUA CÁC ĐỀ THI OLYMPIC Trang 39

• Ta cũng có thể sử dụng phép thế biến để tìm GTLN của P như sau: Vì a, b, c là
các số dương có tích bằng 1 nên tồn tại x, y, z dương sao cho:
x y z
a= ,b = ,c =
y z x

Khi đó:   
x y z yz zx xy
P = + + 6− 2 − 2 − 2
y z x x y z

 
− x2 z + y 2 x + z 2 y x3 y 3 + y 3 z 3 + z 3 x3 − 6x2 y 2 z 2
=
x3 y 3 z 3
Đến đây mọi việc trở nên trong sáng hơn 10 . Vì:
(
x3 y 3 + y 3 z 3 + z 3 x3 − 6x2 y 2 z 2 ≥ −3x2 y 2 z 2
x2 z + y 2 x + z 2 y ≥ 3xyz

nên P ≤ 9.

Bài 1.28 (HSG lớp 12 cấp Tỉnh - Tây Ninh).

Với a, b, c là các số thực dương thỏa mãn abc = 1. Chứng minh rằng:
1 1 1 √

4
+√
4
+√
4
≤ 3
a3 + 2b3 + 6 b3 + 2c3 + 6 c3 + 2a3 + 6

LỜI GIẢI.
Cách 1: Ta có:
b
a3 + 2b3 ≥ 3ab2 = 3
c
Do đó ta cần chứng minh:
√ √ √
4
c 4
a 4
b √

4
+ √
4
+ √
4
≤ 3
3b + 6c 3c + 6a 3a + 6b

Theo BĐT AM - GM ta lại có:


 √ √ √4

c a b 4 4
c 4
a b
+ + +1≥ √
4
√ +√ +√
3b + 6c 3c + 6a 3a + 6b 93
4
3b + 6c 4
3c + 6a 4
3a + 6b

Bài toán sẽ được chứng minh nếu ta chứng minh được rằng:
c a b 1
+ + ≤
3b + 6c 3c + 6a 3a + 6b 3

b c a
⇔ + + ≥1
b + 2c c + 2a a + 2b
10
Đây cũng là mục tiêu chính của phép thế biến.

GV: Nguyễn Tuấn Anh 3. Các bài toán BĐT qua các đề thi chọn HSG cấp tỉnh
Trang 40 CHƯƠNG 1. BẤT ĐẲNG THỨC QUA CÁC ĐỀ THI OLYMPIC

BĐT cuối đúng theo BĐT Cauchy, cụ thể:

b c a Cauchy (a + b + c)2
+ + ≥ =1
b + 2c c + 2a a + 2b a2 + b2 + c2 + 2 (ab + bc + ca)

Bài toán được chứng minh.


Cách 2: Vì vế trái là biểu thức chứa căn, nên suy nghĩ tự nhiên là ta sẽ xử lý căn thức.
Khi đó BĐT đưa về BĐT dạng phân thức bình thường ở đó ta có nhiều hướng tấn công
bài toán hơn, chí ít là dễ biến đổi hơn vì mất căn thức.
Theo BĐT AM - GM ta có:
 
4 1 1 1

4

4
+√
4 3
+√
4 3
93 a3 + 2b3 + 6 b + 2c3 + 6 c + 2a3 + 6

1 1 1
≤ + 3 + 3 +1
a3 + 2b + 6 b + 2c + 6 c + 2a3 + 6
3 3

nên ta cần chứng minh:


1 1 1 1
+ 3 + 3 ≤
a3 3 3 3
+ 2b + 6 b + 2c + 6 c + 2a + 6 3

a3 + 2b3 b3 + 2c3 c3 + 2a3


⇔ + + ≥1
a3 + 2b3 + 6 b3 + 2c3 + 6 c3 + 2a3 + 6

Theo BĐT Cauchy ta có:

a3 + 2b3 b3 + 2c3 c3 + 2a3


+ +
a3 + 2b3 + 6 b3 + 2c3 + 6 c3 + 2a3 + 6

2
9 a3 + b3 + c3

5 (a6 + b6 + c6 ) + 4 (a3 b3 + b3 c3 + c3 a3 ) + 18 (a3 + b3 + c3 )

Do vậy ta cần có:


2   
9 a3 + b 3 + c 3 ≥ 5 a6 + b6 + c6 + 4 a3 b3 + b3 c3 + c3 a3 + 18 a3 + b3 + c3

  
⇔ 4 a6 + b6 + c6 + 14 a3 b3 + b3 c3 + c3 a3 ≥ 18 a3 + b3 + c3

BĐT trên có thể chứng minh bằng công cụ đạo hàm theo một biến.11 Thế nhưng, vì
BĐT xuất hiện khá nhiều nhân tử có thể phân tích bình phương. Điều đó gợi cho ta
11
(Lekhanhsy - http://forum.mathscope.org/) Xét hàm số

7
f (a) = 2a6 + − 9a3 + 36 ln a
a3

Sử dụng công cụ đạo hàm chứng minh được f (a) ≥ 0, đẳng thức xảy ra khi a = 1.

3. Các bài toán BĐT qua các đề thi chọn HSG cấp tỉnh GV: Nguyễn Tuấn Anh
CHƯƠNG 1. BẤT ĐẲNG THỨC QUA CÁC ĐỀ THI OLYMPIC Trang 41

hướng đến phương pháp SOS mà chính xác hơn là ý tưởng phân tích bình phương.
BĐT tương đương với:
2 2 2
a3 − b 3 + b3 − c 3 + c 3 − a3 + 9 a3 b3 + b3 c3 + c3 a3 − abc a3 + b3 + c3
 
≥0

Ta lại có:


 a3 b3 + b3 c3 + c3 a3 − 3 = a3 b3 + b3 c3 + c3 a3 − 3a6 b6 c6
1

= (ab + bc + ca) b2 (a − c)2 + c2 (b − a)2 + a2 (c − b)2

  

3 3 3
 2 3 + b3 + c3 − 3abc



 abc a + b + c − 3 = abc a
 1
= abc (a + b + c) (a − b)2 + (b − c)2 + (c − a)2

  
2
Do vậy BĐT được viết lại là:
Xn
2 2 9 o
2
c2 (ab + bc + ca) − abc (a + b + c) (a − b)2 ≥ 0

a + ab + b +
2
cyc

Ta cần chứng minh: (tương tự cho hai biểu thức còn lại trước đại lượng bình phương)
2 9 2 
a2 + ab + b2 + c (ab + bc + ca) − abc (a + b + c) ≥ 0
2

⇔ 2a4 + 2b4 + 4a3 b + 4ab3 + 6a2 b2 + 9ac3 + 9bc3 ≥ 9abc (a + b)

⇔ a4 + b4 + (a + b)4 + 9c3 (a + b) ≥ 9abc (a + b)

Theo BĐT AM - GM ta có:


a4 b4 9c3 (a + b)
 q
≥ 10 (a + b)2
5
3. + 3. + 4.


 3 3 4

 2 2 2 2 2
 (a + b)4 + 2(a + b) 5 + 2(a + b) 5 + 2(a + b) 5 + 2(a + b) 5 + 2(a + b) 5 ≥ 6√


6

25 (a + b)


⇒ a4 + b4 + (a + b)4 + 9c3 (a + b) ≥ 6 25 (a + b)
6

Hiển nhiên

6
6 25 (a + b) > 9 (a + b) = 9abc (a + b)

nên bài toán được chứng minh. 


Nhận xét: Bài toán không quá phức tạp và các bước biến đổi luôn có một mục tiêu rõ
ràng. Qua cách 2, một lần nữa khẳng định BĐT không đơn thuần là các phép biến đổi,
trò chơi đánh đố mà ở đó ẩn chứa các phép suy luận ngược, phân tích,... một kỹ năng
không hề thừa cho việc học Toán cũng như trong cuộc sống.

GV: Nguyễn Tuấn Anh 3. Các bài toán BĐT qua các đề thi chọn HSG cấp tỉnh
Trang 42 CHƯƠNG 1. BẤT ĐẲNG THỨC QUA CÁC ĐỀ THI OLYMPIC

Bài 1.29 (Chọn HSG cấp Tỉnh - Phú Yên).

Với ba số thực dương a, b, c thỏa mãn a2 + b2 + c2 = 1. Tìm giá trị nhỏ nhất của biểu thức:
bc ca ab
P = + +
a b c

LỜI GIẢI.
Cách 1: Không khó để đoán được điểm rơi của bài toán. Ta sẽ chứng minh:
bc ca ab √
+ + ≥ 3
a b c
Để tận dụng được giả thiết và có hướng xử lý bài toán, ta tiến hành thuần nhất BĐT:
 2
bc ca ab 2 2 2

+ + ≥3 a +b +c
a b c

BĐT trên tương đương với:


2
2
(ab) + (bc)2 + (ca)2
 
≥ 3a2 b2 c2 a2 + b2 + c2
⇔ (ab − bc)2 + (bc − ca)2 + (ca − ab)2 ≥ 0

√ 1
Do vậy P ≥ 3. Đẳng thức xảy ra khi và chỉ khi a = b = c = √ . Tức GTNN của P là
√ 3
3.
Cách 2: Áp dụng BĐT AM -GM ta có:
 2  
bc ca ab bc ca bc ab ca ab
= 3 a2 + b 2 + c 2 = 3

+ + ≥3 . + . + .
a b c a b a c b c

Từ đó ta có đáp số cho bài toán. 

Bài 1.30 (HSG cấp tỉnh - Thái Nguyên).

Với x, y, z là các số thực dương. Tìm giá trị nhỏ nhất của biểu thức:
 
7 3 9
P = max x, y, z, + 2 + 3
x y z

LỜI GIẢI. Từ giả thiết ta có:


7 3 9
P ≥ x; P ≥ y; P ≥ z; P ≥ + 2+ 3
x y z

Giả sử GTNN đạt được khi x = a, y = b, z = c. Khi đó ta sẽ dùng đánh giá sau để tìm
GTNN 12
7 6 27 7x 6y 27z 7
 3 9
+ + + 1 P ≥ + + + + +
a2 b3 c4 a2 b3 c4 x y2 z3
12
Bạn đọc hãy suy nghĩ tại sao lại chọn cách đánh giá này?

3. Các bài toán BĐT qua các đề thi chọn HSG cấp tỉnh GV: Nguyễn Tuấn Anh
CHƯƠNG 1. BẤT ĐẲNG THỨC QUA CÁC ĐỀ THI OLYMPIC Trang 43

Theo BĐT AM - GM ta có:


7 6 27
 7x 6y 27z 7 3 9 14 9 36
+ + +1 P ≥ + 3 + 4 + + 2+ 3 ≥ + +
a2 b3 c4 a 2 b c x y z a b c
Và ta cần chọn a, b, c sao cho đẳng thức có thể xảy ra. Theo các lập luận trên thì đẳng
thức xảy ra khi và chỉ khi:
14 9 36
7 3 9 + 2+ 3
a=b=c= + 2 + 3 = a b c ⇔a=b=c=3
a b c 7 6 27
+ + 4 +1
a2 b 3 c
Trình bày lại lời giải xin dành cho bạn đọc. 

Bài 1.31 (HSG cấp tỉnh lớp 11 - Vĩnh Long).

Với a, b, c là độ dài ba cạnh của tam giác, p là nữa của vi của tam giác đó. Chứng minh
rằng:
1 1 1 18
+ + ≥
p−a p−b p−c a+b+c

LỜI GIẢI.
Cách 1: Theo BĐT Cauchy dạng phân thức ta có:
1 1 1 9 18
+ + ≥ =
p−a p−b p−c 3p − (a + b + c) a+b+c

Cách 2: Ta có:
1 1 1 r + 4R
+ + =
p−a p−b p−c pr
nên BĐT cần chứng minh được viết lại là:
r + 4R
≥ 9 ⇔ R ≥ 2r
r
BĐT trên hiển nhiên nên bài toán được chứng minh. 

Bài 1.32 (Quảng Ninh).

Với x, y là các số thực khác 0 thỏa mãn 2 (x4 + y 4 ) = x4 y 4 . Tìm GTNN của biểu thức:

x2 y2 1
P = 2
+ 2
+p
y +1 x +1 x4 + y 4 + 1

LỜI GIẢI. Theo giả thiết ta có:

4x2 y 2 ≤ 2 x4 + y 4 = x4 y 4 ⇒ x2 y 2 ≥ 4


Theo BĐT Cauchy và BĐT AM - GM ta có:


2 2
2
 2 2
x2 y2 x +y x2 + y 2 x2 + y 2 x2 y 2 2
+ ≥ 2 2 ≥ = = +
y 2 + 1 x2 + 1 2x y + x2 + y 2 3x2 y 2
p
2x2 y 2 + 2 (x4 + y 4 ) 6 3

GV: Nguyễn Tuấn Anh 3. Các bài toán BĐT qua các đề thi chọn HSG cấp tỉnh
Trang 44 CHƯƠNG 1. BẤT ĐẲNG THỨC QUA CÁC ĐỀ THI OLYMPIC

Do đó:
x2 y 2 2 1
P ≥ f x2 y 2 =

+ +r
6 3 x4 y 4
+1
2
Bằng cách khảo sát hàm số ta được:
x2 y 2 2 1 5
P ≥ f x2 y 2 =

+ +r ≥ f (4) =
6 3 x4 y 4 3
+1
2
5
Hay nói cách khác P ≥ . Đẳng thức xảy ra khi và chỉ khi
3

x2 = y 2 ; x2 y 2 = 4 ⇔ x 2 = y 2 = 2

5
vậy nên GTNN của P là . 
3

3. Các bài toán BĐT qua các đề thi chọn HSG cấp tỉnh GV: Nguyễn Tuấn Anh
CHƯƠNG 1. BẤT ĐẲNG THỨC QUA CÁC ĐỀ THI OLYMPIC Trang 45

Các câu BĐT tiếp theo trong mục này, người viết chưa rõ nguồn là đề
chọn HSG dự thi VMO hay là đề thi HSG cấp Tỉnh. Nguồn được gửi từ
một người bạn qua facebook.

Bài 1.33 (TP.HCM).

Với các số thực dương a, b, c có tổng bằng 3.


a) Tìm GTNN của biểu thức:
12
P = abc +
ab + bc + ca
b) Chứng minh số nguyên k nhỏ nhất sao cho

k k
abc + ≥1+
ab + bc + ca 3

luôn đúng với mọi a, b, c thỏa điều kiện trên là k = 10.

LỜI GIẢI.
a) Không khó để đoán GTNN của P là 5. Theo BĐT Schur bậc 3 ta có:

(a + b + c)3 + 9abc ≥ 4 (a + b + c) (ab + bc + ca)


⇔ 9 + 3abc ≥ 4 (ab + bc + ca)

12 16
⇔ ≥
ab + bc + ca 3 + abc
Do đó ta cần chứng minh:
16
+ abc ≥ 5
3 + abc
Theo BĐT AM - GM ta lại có:
16
+ (3 + abc) ≥ 8
3 + abc
Nên P ≥ 5, đẳng thức xảy ra khi và chỉ khi a = b = c = 1. Nên GTNN của P là 5.
b) Xét a = 3 − 2x, b = x, c = x. Khi đó:
k k
P = x2 (3 − 2x) + 2
≥1+
2 (3 − 2x) x + x 3

 
1 1
⇔ x2 (3 − 2x) − 1 ≥k −
3 2 (3 − 2x) x + x2

(x − 1)2
⇔ (x − 1)2 (−2x − 1) ≥ k.
3 (x − 2) x

BĐT cuối luôn đúng khi và chỉ khi:

3 (−2x − 1) (x − 2) x ≤ k

GV: Nguyễn Tuấn Anh 3. Các bài toán BĐT qua các đề thi chọn HSG cấp tỉnh
Trang 46 CHƯƠNG 1. BẤT ĐẲNG THỨC QUA CÁC ĐỀ THI OLYMPIC

Xét hàm số:13

f (x) = 3 (−2x − 1) (x − 2) x

3
với 0 < x < . Bằng cách khảo sát hàm số ta được:
2
r
7
7
9 3
max ! f (x) = +
3 2 2
x∈ 0;
2
r
7
1 3
đạt được khi và chỉ khi x = + . BĐT trên luôn đúng khi và chỉ khi:
2 6
r
7
7 √
9 3 27 + 7 21
k≥ + =
2 2 6

27 + 7 21
Vậy ta đã được điều kiện cần cho k để BĐT đề bài đúng là k ≥ . Để trả lời
√ 6
27 + 7 21
cho bài toán, ta sẽ chứng minh k ≥ cũng là điều kiện đủ để BĐT đề bài luôn
6
đúng. √
27 + 7 21
Ta đặt k0 = , khi đó nếu BĐT đúng với k0 thì BĐT sẽ đúng với các k khác
6
thỏa k ≥ k0 . Do đó ta cần chứng minh:

k0 k0
abc + ≥1+
ab + bc + ca 3

Hiển nhiên với k0 ta không thể sử dụng cách đánh giá như với k = 12. Vì tổng các biến
là hằng số và điểm rơi của bài toán là a = b = c = 1 nên suy nghĩ tự nhiên ta sẽ sử dụng
phương pháp dồn biến. Cụ thể ta cần có:
   2
k0 b+c b+c b+c k0
f (a, b, c) = abc + ≥f a, , =a + 2
ab + bc + ca 2 2 2

b+c
a (b + c) +
2

13
Thực chất thì điều kiện cần và đủ để BĐT

2
2 (x − 1)
(x − 1) (−2x − 1) ≥ k.
3 (x − 2) x

3
luôn đúng là 3 (−2x − 1) (x − 2) x ≤ k với 0 < x < , x 6= 1. Nhưng y = 3 (−2x − 1) (x − 2) x là hàm liên tục
2
3
nên ta xét với 0 < x < .
2

3. Các bài toán BĐT qua các đề thi chọn HSG cấp tỉnh GV: Nguyễn Tuấn Anh
CHƯƠNG 1. BẤT ĐẲNG THỨC QUA CÁC ĐỀ THI OLYMPIC Trang 47

BĐT trên tương đương với:


 

−a(b − c)2 1 1
 
+ k0  − 2  ≥ 0
 
4  ab + bc + ca

b+c 
a (b + c) +
2

 

(b − c)2  a(b − c)2


 
k0 
⇔  " 2 
# ≥
4 


b+c  4
(ab + bc + ca) a (b + c) +
2

Ta thấy rằng:
"  2 #
b+c
(ab + bc + ca) a (b + c) +
2

(ab + bc + ca) (b + c) (b + c + 4a) 3 (b + c) (b + c + 4a)


= ≤
4 4

(3b + 3c) (b + c + 4a) (4a + 4b + 4c)2


= ≤ =9
4 16
Nên ta được:
k0 (b − c)2 k0 (b − c)2
4
"  2 # ≥ 36
b+c
(ab + bc + ca) a (b + c) +
2

Do đó ta cần có:
k0 (b − c)2 a(b − c)2

36 4
BĐT trên không hoàn toán đúng, tuy nhiên nếu ta sắp thứ tự biến trước khi dồn biến
tức giả sử c ≥ b ≥ a thì BĐT trên hiển nhiên vì
k0 1 a
> ≥
36 4 4
Vậy ta được:
 
b+c b+c 3
 
f (a, b, c) ≥ f a, , = f (3 − 2x, x, x) ; 1 ≤ x <
2 2 2

Theo cách tìm điều kiện cần thì ta đã có:


k0
f (3 − 2x, x, x) ≥ 1 +
3
Bài toán được chứng minh. 

GV: Nguyễn Tuấn Anh 3. Các bài toán BĐT qua các đề thi chọn HSG cấp tỉnh
Trang 48 CHƯƠNG 1. BẤT ĐẲNG THỨC QUA CÁC ĐỀ THI OLYMPIC

Bài 1.34 (Lạng Sơn).

Với ba số thực dương a, b, c có tổng bằng 9. Tìm GTNN của biểu thức:

a3 b3 c3
P = + +
a2 + ab + b2 b2 + bc + c2 c2 + ca + a2

LỜI GIẢI. Ta có:


a3 2a − b

a2 + ab + b2 3
⇔ a − 2a b − 2ab2 + a2 b + ab2 + b3 ≥ 0
3 2

⇔ (a − b)2 (a + b) ≥ 0

Tương tự cho các đánh giá còn lại. Do đó:


a3 b3 c3 a+b+c
+ + ≥ =3
a2 + ab + b2 b2 + bc + c2 c2 + ca + a2 3
Đẳng thức xảy ra khi và chỉ a = b = c = 3. Nên GTNN của P là 3. 
Nhận xét: Câu hỏi được đặt ra cho bài giải trên là tại sao có đánh giá ban đầu? Đánh
giá này có nhiều hướng để chọn ra nó, cụ thể:

• Sử dụng hệ số bất định: Ta cần tìm x, y sao cho đánh giá sau luôn đúng:

a3
≥ xa + yb
a2 + ab + b2
Đương nhiên bằng phép biến đổi tương đương, ta có thể chọn được x, y cho bài
toán. Ta để ý cách làm sau: dễ dàng đoán được GTNN của bài toán đặt được khi
a = b = c = 1 nên ta được:
1
x+y =
3
Đánh giá trên được viết lại là:
a3
1  1
1  b
≥ − y a + yb ⇔  2 ≥ 3 − y + y a
a2 + ab + b2 3 b b
1+ +
a a

b
Ta đặt t = . Khi đó:
a
1 1 2 3
⇔ −3yt3 − t2 − t + 3y + 2 ≥ 0

2
≥ + y (t − 1) ⇔ 3 ≥ t + t + 1 + y t − 1
1+t+t 3
Ta biết BĐT cuối có điểm rơi là t = 1 (ứng với a = b) và để nó luôn đúng, ta cần
chọn y sao cho t = 1 là nghiệm kép, tức cần chọn y sao cho:
−1
−9y − 2 − 1 = 0 ⇔ y =
3
2
Từ đó ta được x = .
3

3. Các bài toán BĐT qua các đề thi chọn HSG cấp tỉnh GV: Nguyễn Tuấn Anh
CHƯƠNG 1. BẤT ĐẲNG THỨC QUA CÁC ĐỀ THI OLYMPIC Trang 49

• Sử dụng phương pháp tiếp tuyến: Như phần trên ta cần chọn y sao cho có
đánh giá sau:
1 1
 
2
≥ yt + −y
1+t+t 3
ta liên tưởng đến phương pháp tiếp tuyến. Phương trình tiếp tại điểm rơi t = 1 là
1 2
y = − t + . Do đó không khó để kiểm chứng lại:
3 3
1 1 2
2
≥− t+
1+t+t 3 3

Bài 1.35 (Bình Dương).

Tìm tất cả các số thực k sao cho BĐT sau đúng với mọi số thực a, b, c:

(a + b + c)2
+ k. max (a − b)2 ; (b − c)2 ; (c − a)2 ≤ a2 + b2 + c2

ab + bc + ca ≤
3

LỜI GIẢI.
Cách 1: 14

• Điều kiện cần: Chọn a = 3, b = 2, c = 1. BĐT đã cho trở thành:

1 1
11 ≤ 12 + 4k ≤ 14 ⇔ ≥k≥−
2 4

1 1
• Điều kiện đủ: Với mỗi k thỏa mãn ≥ k ≥ − ta sẽ chứng minh BĐT đề bài luôn
2 4
đúng.
Không mất tính tổng quát ta giả sử a ≥ b ≥ c. BĐT được viết lại là:

(a + b + c)2
ab + bc + ca ≤ + k(c − a)2 ≤ a2 + b2 + c2
3

⇔ 3 (ab + bc + ca) ≤ (a + b + c)2 + 3k(c − a)2 ≤ 3 a2 + b2 + c2




(
(a + b + c)2 + 3k(c − a)2 ≥ 3 (ab + bc + ca)

(a + b + c)2 + 3k(c − a)2 ≤ 3 a2 + b2 + c2


(
(b − c)2 + (6k + 1) (c − a)2 + (a − b)2 ≥ 0

(b − c)2 + (1 − 3k) (c − a)2 + (a − b)2 ≥ 0

Ta lại có:
2 2(a − c)2
(b − c) + (a − b) ≥
2
14
Đay cũng là đề thi chọn đội tuyển THPT chuyên Đại học Vinh ngày thứ 1 năm 2016 - 2017.

GV: Nguyễn Tuấn Anh 3. Các bài toán BĐT qua các đề thi chọn HSG cấp tỉnh
Trang 50 CHƯƠNG 1. BẤT ĐẲNG THỨC QUA CÁC ĐỀ THI OLYMPIC

Nên ta chỉ cần chứng minh:



3


 6k + (c − a)2 ≥ 0

 2

 3 − 3k (c − a)2 ≥ 0

  

2
1 1
Hai BĐT cuối luôn đúng vì ≥k≥− .
2 4
1 1
Vậy ≥ k ≥ − là các giá trị cần tìm.
2 4
Cách 2: (diendantoanhoc.net)

• Điều kiện cần: Chọn a = 3, b = 2, c = 1. BĐT đã cho trở thành:

1 1
11 ≤ 12 + 4k ≤ 14 ⇔ ≥k≥−
2 4

1 1
• Điều kiện đủ: Với mỗi k thỏa mãn ≥ k ≥ − ta sẽ chứng minh BĐT đề bài luôn
2 4
đúng.
Không mất tính tổng quát ta giả sử a ≥ b ≥ c. BĐT được viết lại là:

(a + b + c)2
ab + bc + ca ≤ + k(c − a)2 ≤ a2 + b2 + c2
3
(
a=c+x
Đặt ; (x ≥ y ≥ 0) BĐT trên được viết lại là:
b=c+y


 y 2 − xy + (3k + 1) x2 ≥ 0

 y 2 − xy + 1 − 3 k x2 ≥ 0

  
2

1 2 1 2
 
2

 y − xy + x + 3 k + x ≥0

 4 4

 y 2 − xy + 1 x2 + 3 (1 − 2k) x2 ≥ 0



4 4

1 2

1 2
  

 y− x +3 k+ x ≥0

 2 4

 2
 y − 1 x + 3 (1 − 2k) x2 ≥ 0

 
2 4
1 1
Hai BĐT cuối luôn đúng vì ≥k≥−
2 4

3. Các bài toán BĐT qua các đề thi chọn HSG cấp tỉnh GV: Nguyễn Tuấn Anh
CHƯƠNG 1. BẤT ĐẲNG THỨC QUA CÁC ĐỀ THI OLYMPIC Trang 51

1 1
Vậy ≥ k ≥ − là các giá trị cần tìm. 
2 4
Bài 1.36 (Kon Tum).

Với ba số thực dương a, b, c thỏa mãn a2 b2 + b2 c2 + c2 a2 ≥ a2 b2 c2 . Chứng minh rằng:



a2 b 2 b 2 c2 c 2 a2 3
3 2 2
+ 3 2 2
+ 3 2 2

c (a + b ) a (b + c ) b (c + a ) 2

LỜI GIẢI. Từ giả thiết ta có:


1 1 1
+ + ≥1
a2 b2 c2

và BĐT được viết lại là:


1 1 1
1 c3 +1 a3 +1 b3  ≥1
1 1 1
+ + +
a2 b2 b2 c2 c2 a2

Để thuận tiện, ta đặt:


1 1 1
x = ;y = ;z =
a b c
Khi đó ta cần chứng minh:
x3 y3 z3
+ + ≥1
y 2 + z 2 z 2 + x2 x2 + y 2

Liên kết giả thiết ta sẽ chứng minh một BĐT (đồng bậc) mạnh hơn:

x3 y3 z3 3p 2
+ + ≥ x + y2 + z2
y 2 + z 2 z 2 + x2 x2 + y 2 2

Hướng 1: Theo BĐT Cauchy ta có:


2
x3 y3 z3 x2 + y 2 + z 2
+ + ≥
y 2 + z 2 z 2 + x2 x2 + y 2 x (y 2 + z 2 ) + y (z 2 + x2 ) + z (x2 + y 2 )

Do đó ta cần chứng minh:


3 2
4 x2 + y 2 + z 2 ≥ 3 x y 2 + z 2 + y z 2 + x2 + z x 2 + y 2
  

BĐT là đồng bậc (ta quên điều kiện bài toán đi) ta chuẩn hóa x2 + y 2 + z 2 = 3. Khi đó
ta cần chứng minh:
x 3 − x2 + y 3 − y 2 + z 3 − z 2 ≤ 6
  

Mà điều trên là hiển nhiên vì ta có:

x3 + 2 ≥ 3x

GV: Nguyễn Tuấn Anh 3. Các bài toán BĐT qua các đề thi chọn HSG cấp tỉnh
Trang 52 CHƯƠNG 1. BẤT ĐẲNG THỨC QUA CÁC ĐỀ THI OLYMPIC

1
Bài toán được chứng minh. Đẳng thức xảy ra khi và chỉ khi a = b = c = √ .
3
Hướng 2: Ta nhắc lại BĐT cần chứng minh:

x3 y3 z3 3p 2
+ + ≥ x + y2 + z2
y 2 + z 2 z 2 + x2 x2 + y 2 2

Vì BĐT là thuần nhất nên ta chuẩn hóa (quên điều kiện bài toán đi) x2 + y 2 + z 2 = 3.
Khi đó BĐT cần chứng minh là:

x3 y3 z3 3
2
+ 2
+ 2

3−x 3−y 3−z 2

Ta có:15

x3 1
≥ x2 − ⇔ 2x3 ≥ 3 − x2 2x2 − 1 ⇔ 2x3 + 2x4 + 3 ≥ 7x2
 
3−x 2 2

BĐT cuối là đúng theo BĐT AM - GM 7 số. Do đó ta được:

x3 y3 z3 3 3
2
+ 2
+ 2
≥ x2 + y 2 + z 2 − =
3−x 3−y 3−z 2 2

Bài toán được chứng minh. 

Bài 1.37 (Quảng Nam).

Với ba số thực không âm a, b, c có tổng 3. Tìm GTNN, GTLN của biểu thức:
√ √ √
P = a2 + a + 4 + b 2 + b + 4 + c 2 + c + 4

LỜI GIẢI.

• Ta tìm GTNN bằng phương pháp tiếp tuyến, cụ thể ta có:


√ √ √
p 6 √ 6 3 6
a2 +a+4≥ (a − 1) + 6 = a+
4 4 4

Thật vậy, BĐT trên tương đương với:

16 a2 + a + 4 ≥ 6(a + 3)2 ⇔ 10(a − 1)2 ≥ 0




Khi đó: √ √
6 9 6 √
P ≥ (a + b + c) + =3 6
4 4

Đẳng thức xảy ra khi và chỉ khi a = b = c = 1. Do đó GTNN của P là 3 6.
15
Để có đánh giá này mời bạn đọc xem phần nhận xét trong lời giải đề HSG của Tỉnh Lạng Sơn.

3. Các bài toán BĐT qua các đề thi chọn HSG cấp tỉnh GV: Nguyễn Tuấn Anh
CHƯƠNG 1. BẤT ĐẲNG THỨC QUA CÁC ĐỀ THI OLYMPIC Trang 53

• Ta tìm GTLN:
Không khó để dự đoán GTLN đặt khi a = b = 0; c = 3 và các hoán vị của nó. Sử
dụng phương pháp hệ số bất định, ta cần chọn x, y cho đánh giá:
p
a2 + a + 4 ≤ xa + y

đúng với mọi a và đẳng thức xảy ra khi a = 0, 3. Tức cần chọn x, y sao cho:

 x= 2
(
y=2
⇔ 3
4 = 3x + 2  y=2

Kiểm tra lại đánh giá trên bằng phép biến đổi tương đương:16
p 2
a2 + a + 4 ≤ a + 2 ⇔ a2 − 3a ≤ 0 ⇔ a (a − 3) ≤ 0
3
Điều này là hiển nhiên vì a, b, c là các số dương và a + b + c = 3.
Từ đó ta được:
2
P ≤ (a + b + c) + 6 = 8
3
Đẳng thức xảy ra khi và chỉ khi a = b = 0, c = 3 và các hoán vị của nó.


Nhận xét: Bạn đọc hãy áp dụng kỹ thuật trên cho bài toán tương tự sau: Trại hè Hùng
Vương lần XII Bắc Giang 2016, Lớp 11
Với ba số thực không âm x, y, z có tổng bằng 3. Tìm GTNN và GTLN của biểu thức sau:
r r r
2 2 2
P = x3 − x2 + 4 + y 3 − y 2 + 4 + z 3 − z 2 + 4
3 3 3

Bài 1.38 (Ninh Bình).

Với ba số thực dương a, b, c thỏa mãn 3bc + 4ca + 5ab ≤ 6abc. Tìm GTLN của biểu thức:
3a + 2b + c
P =
(a + b) (b + c) (c + a)

LỜI GIẢI. Ta viết lại giả thiết:


3 4 5
3bc + 4ca + 5ab ≤ 6abc ⇔ + + ≤6
a b c
và biểu thức bài toán:
3a + 2b + c (a + c) + 2 (a + b) 1 2
= = +
(a + b) (b + c) (c + a) (a + b) (b + c) (c + a) (a + b) (b + c) (b + c) (c + a)
16
Cách ta chọn đánh giá trên chỉ là điều kiện cần chưa là điệu kiện đủ.

GV: Nguyễn Tuấn Anh 3. Các bài toán BĐT qua các đề thi chọn HSG cấp tỉnh
Trang 54 CHƯƠNG 1. BẤT ĐẲNG THỨC QUA CÁC ĐỀ THI OLYMPIC

Do đó một suy nghĩ tự nhiên là ta cần có đánh giá:


1 2 3 4 5
 2
+ ≤x + +
(a + b) (b + c) (b + c) (c + a) a b c

Có thể đoán được GTLN của P đạt được khi a = b = c = 2 nên ta sẽ chứng minh:
1 2 1 3 4 5 2
 
+ ≤ + +
(a + b) (b + c) (b + c) (c + a) 192 a b c

BĐT này có nhiều hướng để chứng minh, vì dạng BĐT chứa phân thức ta nghĩ ngay
đến BĐT Cauhy dạng phân thức. Cụ thể Theo BĐT Cauchy, ta có:
3 4 5 1 1
1 1 1 1 4 8 12
+ + ≥ + +2 + +3 + ≥ + +
a b c a b a c b c a+b a+c b+c
Do vậy ta cần chứng minh:
1 2 1 4 8 12
 2 1 1
 2 3
2
+ ≤ + + = + +
(a + b) (b + c) (b + c) (c + a) 192 a + b a + c b + c 12 a + b a + c b + c

Thật vậy, BĐT trên tương đương với:


12 24
 1 2 3
2
+ ≤ + +
(a + b) (b + c) (b + c) (c + a) a+b a+c b+c

1 4 9 4 12 6
= + + + + +
(a + b)2 (a + c)2 (b + c)2 (a + b) (a + c) (b + c) (a + c) (a + b) (b + c)

1 4 9 4 12 6
⇔ 2
+ 2
+ 2
≥− + +
(a + b) (a + c) (b + c) (a + b) (a + c) (b + c) (a + c) (a + b) (b + c)

 1 2 3
2
⇔ + − ≥0
a+b a+c b+c
3
Vậy ta được: P ≤ , đẳng thức xảy ra khi và chỉ khi a = b = c = 2. Do đó GTLN của
16
3
P là . 
16
Bài 1.39 (Hậu Giang).

Với ba số thực dương a, b, c thỏa mãn ab + bc + ca = 2, a2 + b2 + c2 = 4. Tìm GTLN của biểu


thức P = abc.

LỜI GIẢI. Từ giả thiết ta thấy rằng không thể xảy ra trường hợp a = b = c, hơn nữa

ta cũng có a + b + c = 2 2.
Với mỗi a, b, c thỏa mãn giả thiết thì a, b, c là ba nghiệm dương (có thể có nghiệm kép)
của phương trình:

X 3 − 2 2X 2 + 2X − abc = 0

3. Các bài toán BĐT qua các đề thi chọn HSG cấp tỉnh GV: Nguyễn Tuấn Anh
CHƯƠNG 1. BẤT ĐẲNG THỨC QUA CÁC ĐỀ THI OLYMPIC Trang 55

Ta xét hàm số

f (X) = X 3 − 2 2X 2 + 2X − abc

Ta có: √ 
√ X= 2
f 0 (X) = 3X 2 − 4 2X + 2; f 0 (X) = 0 ⇔  √
2
X=
3

Để phương trình X 3 − 2 2X 2 + 2X − abc = 0 có ba nghiệm dương (có thể nghiệm kép)
thì điều kiện cần là:  √ 
2 √
≥0

f
8 2
3 ⇒ 0 ≤ abc ≤
√  27
2 ≤0

 f

Hay nói cách khác √


8 2
P = abc ≤
27
√ √
2 4 2
Đẳng thức xảy ra khi và chỉ khi a = b = ;c = và các hoán vị của nó. Vậy nên
√ 3 3
8 2
GTLN của P là 
27
Nhận xét:

• Với cách làm như trên, ta thấy giả thiết có thể thay đổi ab+bc+ca = m, a2 +b2 +c2 = n
với m, n hợp lý.

• Bài toán tổng quát:

(Võ Quốc Bá Cẩn) Cho ba số thực a, b, c thỏa mãn a + b + c = 3k, a2 + b2 + c2 =


3k 2 + 6t2 (với k bất kỳ và t ≥ 0 là hai số cho trước). Khi đó

(k − 2t) (k + t)2 ≤ abc ≤ (k + 2t) (k − t)2

Đẳng thức xảy ra khi và chỉ khi

(a, b, c) ∈ {(k − 2t, k + t, k + t) ; (k + 2t, k − t, k − t)}

Bài 1.40 (Hải Phòng).

Với a, b, c là các số thực thuộc đoạn [0; 1] thỏa mãn a + b + c = 2. Chứng minh rằng:
2 2 4 2
a + b 2 + c 2 ≤ a3 + b 3 + c 3 ≤ a + b2 + c2 − 3abc
 
3 3

LỜI GIẢI. Trước hết ta làm đồng bậc BĐT lại như sau:
 
(a + b + c) a2 + b2 + c2 2 (a + b + c) a2 + b2 + c2
≤ a3 + b 3 + c 3 ≤ − 3abc
3 3

GV: Nguyễn Tuấn Anh 3. Các bài toán BĐT qua các đề thi chọn HSG cấp tỉnh
Trang 56 CHƯƠNG 1. BẤT ĐẲNG THỨC QUA CÁC ĐỀ THI OLYMPIC

• Ta chứng minh: 
(a + b + c) a2 + b2 + c2
≤ a3 + b3 + c3

3
BĐT trên tương đương với:

2 a3 + b3 + c3 ≥ ab (a + b) + bc (b + c) + ca (c + a)


BĐT trên là hiển nhiên theo BĐT AM - GM.

• Ta chứng minh:

3 3
2 (a + b + c) a2 + b2 + c2
3
a +b +c ≤ − 3abc
3
BĐT tương đương với:

a3 + b3 + c3 ≤ 2 [ab (a + b) + bc (b + c) + ca (c + a)] − 9abc

⇔ a3 + b3 + c3 − 3abc ≤ 2 [ab (a + b) + bc (b + c) + ca (c + a) − 6abc]

1
(a + b + c) (a − b)2 + (b − c)2 + (c − a)2 ≤ 2 c(a − b)2 + a(b − c)2 + b(c − a)2
   

2

⇔ (2c − 1) (a − b)2 + (2a − 1) (b − c)2 + (2b − 1) (c − a)2 ≥ 0

Không mất tính tổng quát ta giả sử a ≥ b ≥ c. Khi đó:

(c − a)2 ≥ (a − b)2 + (b − c)2 ⇔ 2 (a − b) (b − c) ≥ 0

Từ đó ta được:

(2a − 1) (b − c)2 + (2b − 1) (c − a)2 + (2c − 1) (a − b)2


≥ (2a + 2b − 2) (b − c)2 + (2b + 2c − 2) (a − b)2

Vì a, b, c là các số thực thuộc đoạn [0; 1] và có tổng bằng 2 nên nếu b + c < 1 thì
a = 2 − b − c > 1 mâu thuẩn giả thiết. Do đó b + c ≥ 1 và hiển nhiên a + b ≥ 1.
Vậy nên BĐT cuối là đúng. Bài toán được chứng minh.


Nhận xét: Để có những bước tách ghép hợp lý như vậy mời bạn đọc xem phần bổ sung
kiến thức của phương pháp SOS (đối với bạn đọc chưa biết phương pháp). Đánh giá
cuối thực chất là một tiêu chuẩn của phương pháp SOS. Ngoài cách chứng minh bằng
phương pháp SOS, bài toán còn có thể chứng minh bằng phương pháp dồn biến. Mời
bạn đọc làm thử.

3. Các bài toán BĐT qua các đề thi chọn HSG cấp tỉnh GV: Nguyễn Tuấn Anh
Chương Hai

BỔ SUNG KIẾN THỨC

1 Một số phương pháp chứng minh BĐT thông dụng


Ngoài những kỹ thuật cơ bản (sử dụng BĐT cổ điển, biến đổi tương đương,...) ta còn
một số kỹ thuật, phương pháp mà tính hiệu quả của nó đã được khẳng định qua rất
nhiều bài viết với những lời giải đẹp. Tài liệu sẽ điểm lại một số kỹ thuật đó. Trước hết
là kỹ thuật cũng là yêu cầu đầu tiên khi ta học về BĐT.

1.1 Kỹ thuật chọn điểm rơi

Trong chứng minh BĐT việc đảm bảo dấu bằng luôn xảy ra qua các bước đánh giá là
vô cùng quan trọng. Bởi nếu trong các bước đánh giá mà mỗi bước có dấu bằng xảy
ra với biến x xảy ra khác nhau thì qua các bước đánh giá đó ta chỉ thu được BĐT mà
đẳng thức không thể xảy ra, có nghĩa ta đã làm yếu BĐT đi và việc không chứng minh
được là điều tất yếu. Ví dụ đơn giản như sau:

Ví dụ 1.
Với mọi số thực x. Chứng minh rằng:
1 5
x2 + 2 + ≥
x2 +2 2

LỜI GIẢI. Nếu ta vận dụng BĐT AM - GM như sau:


r
2 1 x2 + 2
x +2+ 2 ≥2 =2
x +2 x2 + 2
Dấu bằng không thể xảy ra vì:
1
x2 + 2 = ⇔ x2 + 2 = 1 (V N )
x2 +2
vậy nên ta chỉ được: r
1 x2 + 2
x2 + 2 + 2 >2 =2
x +2 x2 + 2

57
Trang 58 CHƯƠNG 2. BỔ SUNG KIẾN THỨC

BĐT đã yếu đi qua bước đánh giá trên đó là lý do không thể chứng minh được bài toán.
Vậy làm sau để có được một đánh giá hợp lý? Trước tiên ta phải đoán nếu đẳng thức
xảy ra thì x =? Sau đó mỗi bước đánh giá ta luôn nhớ phải đảm bảo đẳng thức sẽ xảy
ra. Cụ thể với bài toán trên không khó để đoán đẳng thức xảy ra khi x = 0 do đó ta
đánh giá khéo léo hơn như sau:
 s
1 x2 +2 1 3 x2 + 2 x2 + 2 6 5
x2 + 2 + 2
= + 2 + ≥2 2
+ =
x +2 4 x +2 4 4 (x + 2) 4 2

Tóm lại khi làm việc với BĐT cái nhìn đầu tiên là hãy dự đoán dấu bằng của BĐT sẽ
xảy ra khi nào.
Nếu bước dự đoán không dễ dàng hay nói cách khác điểm rơi là số rất xấu không thể
đoán được thì thế nào?
Đối với mỗi dạng có giả thiết khác nhau sẽ có cách đánh giá khác nhau, hai ví dụ sau
đây hy vọng bạn đọc nắm được ý tưởng khi gặp các BĐT mà dấu bằng khó đoán.
Ví dụ 2.
Với ba số thực dương a, b, c thỏa mãn a + 2b + 3c = 4. Tìm GTNN của biểu thức sau:

P = a2 + b 2 + c 3

LỜI GIẢI. Không khó để nhận ra ta cần một đánh giá dạng:
a2 + b2 + c3 ≥ k (a + 2b + 3c)

Và bước đánh giá đó là dùng BĐT AM- GM. Tuy nhiên rất khó để có thể biết GTNN
của P xảy ra khi a, b, c có giá trị là bao nhiêu để áp dụng AM- GM hợp lý. Ta giả sử khi
P đạt GTNN thì a = x, b = y, c = z và sẽ tiến hành như sau:


 a2 + x2 ≥ 2xa






⇒ a2 + b2 + c3 + x2 + y 2 + 2z 3 ≥ 2xa + 2yb + 3z 2 c

b3 + y 2 ≥ 2yb







c3 + 2z 3 ≥ 3z 2 c

Vậy là ta cần chọn x, y, z sao cho:


 2x 1   16
 = y = 2x  y=
2y 2 25

 
 


 
 


 
 

  

2x 1
 √ 
4
= ⇔ z= 2x ⇔ z=
2 5
 3z 3

 
 

 
 

  
  √ 
 x= 8
  
5x + 3 2x = 4
  
x + 2y + 3z = 4

25

1. Một số phương pháp chứng minh BĐT thông dụng GV: Nguyễn Tuấn Anh
CHƯƠNG 2. BỔ SUNG KIẾN THỨC Trang 59

Nhưng khi trình bày lời giải ta có thể làm ngắn ngọn như sau:
Ta có:
  8 2 16
2
a + ≥ a





 25 25




 2 16 192 128
16 32
 
2 2 3
b2 + ≥ b ⇒a +b +c ≥ (a + 2b + 3c) − =

 25 25 25 125 125




  3
 c3 + 2 4 ≥ 48 c



5 25
8 16 4 128
Đẳng thức xảy ra khi và chỉ khi a = ; b = ; c = . Do đó GTNN của P là 
25 25 5 125
Ví dụ 3.

Với a, b, c là các số thực thay đổi sao cho ab + bc + ca = 1. Tìm GTNN của biểu thức sau:

P = a2 + b2 + 3c2

LỜI GIẢI. Từ giả thiết bài toán gợi cho ta chọn cách đánh giá:

a2 + b2 + 3c2 ≥ k (ab + bc + ca)

Tương tự như ví dụ 2, ta không đoán được điểm rơi khi GTNN đạt được. Ta giả sử khi
đó a = x, b = y, c = z ta có đánh giá sau:

(ay)2 + (bx)2

xyab ≤





 2




 2 2
(bz) + (cy)
yzbc ≤


 2



2 2

 zxac ≤ (az) + (cx)



2

Vấn đề còn lại là chon x, y, z thích hợp sao cho có thể tận dụng được ab + bc + ca = 1.
Do đó ta điều chỉnh lại như sau:

z(ay)2 + z(bx)2

xyzab ≤





 2




x(bz)2 + x(cy)2

xyzbc ≤


 2



2 2

 xyzac ≤ y(az) + y(cx)



2

GV: Nguyễn Tuấn Anh 1. Một số phương pháp chứng minh BĐT thông dụng
Trang 60 CHƯƠNG 2. BỔ SUNG KIẾN THỨC

Vậy nên:

[yz (y + z)] a2 + [zx (z + x)] b2 + [xy (x + y)] c2 ≥ 2xyz (ab + bc + ca)

Ta sẽ chọn x, y, z sao cho:



 y (y + z) = x (z + x)  r

  4 3
x=y=

 

11

 

 
3z (y + z) = x (x + y) ⇔
  √ r
33 − 3 1

 

  4

 z=
 .
 2 297
xy + yz + zx = 1

Phần trình bày lại giải dành cho bạn đọc. 


Để kết lại phần này mời bạn đọc đến với một số bài toán sau để củng cố phương pháp

Bài 2.1 (Tp. HCM - Ngày thứ 1 - Năm 2016).

Với a, b, c là các số thực có tổng bằng 0. Tìm GTNN của biểu thức:
2 2 2 √
P = a2 + 1 + b2 + 1 + c2 + 1 + 6 6abc

Bài 2.2 (Đề chọn đội tuyển Bình Dương 2016).



Cho x, y, z là các số thực dương thỏa mãn x3 + y 2 + z = 2 3 + 1. Tìm giá trị nhỏ nhất của:
1 1 1
P = + 2+ 3
x y z

Bài 2.3 (Đề thi chọn đội tuyển Tỉnh Thái Nguyên - Năm 2016).

Với ba số thực dương a, b, c thỏa mãn ac ≥ 12; bc ≥ 8. Tìm GTNN của biểu thức sau:
 
1 1 1 8
P =a+b+c+2 + + +
ab bc ca abc

1.2 Phương pháp tiếp tuyến

Ý tưởng của phương pháp có thể giải thích như sau:

• Giải tích: Trong chương trình toán THPT ta thường xét đến đạo hàm cấp 1, 2 do
đó nếu f 00 (x) không đổi dấu thì trong khai triển Taylor

f 0 (x0 ) f 00 (x0 ) f (n) (x0 ) f (n+1) (a)


f (x) = f (x0 )+ (x − x0 )+ (x − x0 )2 +...+ (x − x0 )n + (x − x0 )n+1
1! 2! n! (n + 1)!

1. Một số phương pháp chứng minh BĐT thông dụng GV: Nguyễn Tuấn Anh
CHƯƠNG 2. BỔ SUNG KIẾN THỨC Trang 61

ta được:
(
Nếu f 00 (x) ≥ 0; ∀x ∈ [a; b] thì f (x) ≥ f (x0 ) + f 0 (x0 ) (x − x0 ) ; ∀x0 ∈ [a; b]
Nếu f 00 (x) ≤ 0; ∀x ∈ [a; b] thì f (x) ≤ f (x0 ) + f 0 (x0 ) (x − x0 ) ; ∀x0 ∈ [a; b]

(Chứng minh đơn giản bằng cách khai triển đến cấp 2.)

• Hình học: Ta biết rằng tiếp tuyến của đồ thị hàm số tại một điểm (trừ điểm uốn)
luôn nằm trên hoặc nằm dưới đồ thị hàm số trong một khoảng lồi - lõm tức là

f (x) ≥ f 0 (x0 ) (x − x0 ) + f (x0 ) ; ∀x ∈ (a; b)


Hoặc là
f (x) ≤ f 0 (x0 ) (x − x0 ) + f (x0 ) ; ∀x ∈ (a; b)

(Trong đó (a; b) là một khoảng mà hàm số lồi hoặc lõm)

Chính vì vậy ta thường chọn nó làm một đánh giá phụ để chứng minh BĐT.

Ví dụ 4 (BĐT Nesbitt).

Với ba số thực dương a, b, c có tổng bằng 3. Chứng minh rằng:


a b c 3
+ + ≥
3−a 3−b 3−c 2

LỜI GIẢI. Trước tiên ta nhận xét được đẳng thức xảy ra khi và chỉ khi a = b = c = 1.
x
Ta lại thấy tiếp tuyến của hàm số f (x) = luôn nằm phía dưới đồ thị với mọi
3−x
0 < x < 3.
(Bạn đọc hãy vẽ hình để thấy được điều đó)
Trên cơ sở đó ta mạnh dạn đưa ra đánh giá phụ:
a 3 1
≥ (a − 1) +
3−a 4 2

mà việc kiểm chứng rất dễ dàng bằng phép biến đổi tương đương.
Làm tương tự cho các biến b, c cộng lại ta được điều phải chứng minh. 
Một bài nữa để bạn đọc tập dợt

Ví dụ 5 (Baltic Way 2011).

Với bốn số thực a, b, c, d không âm có tổng bằng 4. Chứng minh rằng:


a b c d 4
+ + + ≤
a3 + 8 b 3 + 8 c 3 + 8 d 3 + 8 9
a 6(a − 1) 1
LỜI GIẢI. Ta sử dụng đánh giá: ≤ + 
a3 +8 92 9

GV: Nguyễn Tuấn Anh 1. Một số phương pháp chứng minh BĐT thông dụng
Trang 62 CHƯƠNG 2. BỔ SUNG KIẾN THỨC

Điểm mạnh của phương pháp tiếp tuyến là sử dụng được cho các BĐT có điều kiện
tổng là hằng số và biểu thức tách biến. Nếu điều kiện khác đi hoặc biểu thức không có
dạng tách biến có thể sử dụng được phương pháp không? Hai ví dụ sẽ trả lời cho câu
hỏi đó.

Ví dụ 6 (All-Russian Olympiad 2002).

Với ba số thực dương a, b, c có tổng bằng 3. Chứng minh rằng:


√ √ √
a + b + c ≥ ab + bc + ca

LỜI GIẢI. BĐT không ở dạng tách biến không thể áp dụng phương pháp tiếp tuyến.
Tuy nhiên nếu ta điều chỉnh lại biểu thức như sau:
√ √ √
a2 + 2 a + b2 + 2 b + c2 + 2 c ≥ 2 (ab + bc + ca) + a2 + b2 + c2 = 9

Đến đây vấn đề trở nên đơn giản rất nhiều bằng cách sử dụng đánh giá sau:

a2 + 2 a ≥ 3 (a − 1) + 3 = 3a

Ta có được lời giải cho bài toán. 

Ví dụ 7 (Toán học tuổi trẻ).

Với các số thực dương thỏa mãn ab + bc + ca = 3. Tìm GTNN của biểu thức sau:

P = 5 a3 + b 3 + c 3 + 2 a2 b + b 2 c + c 2 a
 

LỜI GIẢI. Xét hàm số:


f (a) = 5a3 + 2a2 b

Ta dự đoán cho đánh giá:

f (a) ≥ f 0 (1) (a − 1) + f (1)

⇔ 5a3 + 2a2 b ≥ (15 + 4b) (a − 1) + (5 + 2b)

⇔ (a − 1)2 (5a + 2b + 10) ≥ 0

Vậy nên:

P ≥ (15 + 4b) (a − 1) + (5 + 2b) + (15 + 4c) (b − 1) + (5 + 2c) + (15 + 4a) (c − 1) + (5 + 2a)

⇔ P ≥ 4 (ab + bc + ca) + 13 (a + b + c) − 30 ≥ 21

1. Một số phương pháp chứng minh BĐT thông dụng GV: Nguyễn Tuấn Anh
CHƯƠNG 2. BỔ SUNG KIẾN THỨC Trang 63

Đẳng thức xảy ra khi và chỉ khi a = b = c = 1 nên GTNN của P là 21. 
Nhận xét: Ví dụ này muốn nói lên nếu biểu thức cần chứng minh không ở dạng tách
biến ta vẫn có thể dùng phương pháp tiếp tuyến bằng cách xét hàm và xem các biến
còn lại là tham số.
Để hiểu hơn về phương pháp tiếp tuyến cũng như khẳng định lại tính hiệu quả của nó
mời bạn đọc đến với các bài toán sau:

Bài 2.4 (Đề thi chọn đội tuyển tỉnh Thanh Hóa - Vòng 2 - Năm 2016).

1 1 1
Với ba số thực dương x, y, z thỏa mãn x + y + z = + + . Chứng minh rằng:
x y z
1 1 1 3
2 + 2 + 2 ≤
(2xy + yz + zx) (2yz + zx + xy) (2zx + xy + yz) 16x y 2 z 2
2

Bài 2.5 (Đề thi chọn đội tuyển Tỉnh Hà Nam - Năm 2016).

Với ba số thực không âm a, b, c và không có hai số nào đồng thời bằng 0. Tìm GTNN của
biểu thức sau: r r r
a b c
P = + +
b+c c+a a+b

Bài 2.6 (Đề thi chọn HSG Tỉnh Ninh Bình - Năm 2016).

Với ba số thực dương a, b, c, có tổng bằng 3. Chứng minh rằng:


 
1 1 1 1 1 1
√ √ +√ √ +√ √ ≥4 + +
a+ b b+ c c+ a a+7 b+7 c+7

Bài 2.7 (Đề thi HSG cấp Tỉnh - Phú Yên - Năm 2016).

Với bốn số thực dương x, a, b, c thỏa mãn x2 = a2 + b2 + c2 . Chứng minh rằng:


a b c 3
+ + ≤√
x + 2a x + 2b x + 2c 3+2

Bài 2.8 (Đề thi HSG cấp Tỉnh Quảng Ngãi - Năm 2016).
 
1
Với ba số thực x, y, z ∈ ; 1 . Tìm GTNN của biểu thức:
2
6
P = x5 y + xy 5 + − 3 (x + y) − 2016z
x2 + y2

GV: Nguyễn Tuấn Anh 1. Một số phương pháp chứng minh BĐT thông dụng
Trang 64 CHƯƠNG 2. BỔ SUNG KIẾN THỨC

Bài 2.9 (Trại hè Hùng Vương lần XII Bắc Giang 2016, Lớp 11).

Với ba số thực không âm x, y, z có tổng bằng 3. Tìm GTNN và GTLN của biểu thức sau:
r r r
2 2 2
P = x3 − x2 + 4 + y 3 − y 2 + 4 + z 3 − z 2 + 4
3 3 3

1.3 Phương pháp pqr

Các BĐT đối xứng ba biến x, y, z đều có thể quy về BĐT với các biến p = x + y + z, q =
xy + yz + zx, r = xyz , như vậy thay vì chứng minh BĐT với ba biến x, y, z không có mối
quan hệ với nhau ta quy về chứng minh BĐT với ba biến mới p, q, r có mối quan hệ mật
thiết với nhau đó chính là ý tưởng chính của phương pháp.
Để thuận lợi cho việc áp dụng phương pháp ta cần nhớ một số phân tích cơ bản và một
số đánh giá đơn giản như sau:1

Một số phân tích cơ bản

• a2 + b2 + c2 = p2 − 2q

• a3 + b3 + c3 = p3 − 3pq + 3r

• a4 + b4 + c4 = p4 − 4p2 q + 2q 2 + 4pr

• a2 b2 + b2 c2 + c2 a2 = q 2 − 2pr

• ab (a + b) + bc (b + c) + ca (c + a) = pq − 3r

• (a + b) (b + c) (c + a) = pq − r

• (a + b) (a + c) + (b + c) (b + a) + (c + a) (c + b) = p2 + q

  
• ab a2 + b2 + bc b2 + c2 + ca c2 + a2 = p2 q − 2q 2 − pr

• a3 b3 + b3 c3 + c3 a3 = q 3 − 3pqr + 3r2

• a4 b4 + b4 c4 + c4 a4 = q 4 − 4pq 2 r + 2p2 r2 + 4qr2


1
Tham khảo ” BĐT schur và phương pháp biến đổi p, q, r - Võ Thành Văn ”.

1. Một số phương pháp chứng minh BĐT thông dụng GV: Nguyễn Tuấn Anh
CHƯƠNG 2. BỔ SUNG KIẾN THỨC Trang 65

Một số đánh giá đơn giản

• p2 ≥ 3q

• p3 ≥ 27r

• q 2 ≥ 3pr

• pq ≥ 9r

• 2p3 + 9r ≥ 7pq

• p2 q + 3pr ≥ 4q 2

• p4 + 3q 2 ≥ 4p2 q

• p4 + 4q 2 + 6pr ≥ 5p2 q

• p3 + 9r ≥ 4pq

• q 3 + 9r2 ≥ 4pqr

• p3 r + q 3 ≥ 6pqr
( )
p 4q − p2
• r ≥ max 0;
9
(  )
4q − p2 p2 − q
• r ≥ max 0;
6p

Chứng minh:

• p2 ≥ 3q

BĐT tương đương với:


a2 + b2 + c2 ≥ ab + bc + ca

• p3 ≥ 27r

BĐT đúng theo AM - GM ba biến.

• q 2 ≥ 3pr

BĐT tương đương với:

(ab)2 + (bc)2 + (ca)2 ≥ abc (a + b + c)

GV: Nguyễn Tuấn Anh 1. Một số phương pháp chứng minh BĐT thông dụng
Trang 66 CHƯƠNG 2. BỔ SUNG KIẾN THỨC

• pq ≥ 9r

BĐT đúng theo AM - GM ba biến.

• 2p3 + 9r ≥ 7pq

BĐT tương đương với:

2 a3 + b3 + c3 ≥ a2 b + b2 c + c2 a + ab2 + bc2 + ca2


  

BĐT này đúng theo BĐT AM - GM ba biến.

• p2 q + 3pr ≥ 4q 2

BĐT tương đương với:

a3 b + b3 c + c3 a + ab3 + bc3 + ca3 ≥ 2 a2 b2 + b2 c2 + c2 a2




BĐT này đúng theo AM - GM ba biến.

• p4 + 3q 2 ≥ 4p2 q

BĐT tương đương với

a4 + b4 + c4 + a2 b2 + b2 c2 + c2 a2 ≥ 2abc (a + b + c)

BĐT này đúng theo BĐT AM - GM.

• p4 + 4q 2 + 6pr ≥ 5p2 q

BĐT tương đương:

a2 (a − b) (a − c) + b2 (b − a) (b − c) + c2 (c − a) (c − b) ≥ 0

BĐT là BĐT Schur bậc 4 nên hiển nhiên đúng.

• p3 + 9r ≥ 4pq

BĐT trên tương đương với:

a (a − b) (a − c) + b (b − a) (b − c) + c (c − a) (c − b) ≥ 0

BĐT là BĐT Schur bậc 3 nên hiển nhiên đúng.

• q 3 + 9r2 ≥ 4pqr

BĐT trên tương đương với:

(x + y + z)3 + 9xyz ≥ 4 (x + y + z) (xy + yz + xz)

Trong đó x = ab, y = bc, z = ca. BĐT thu được hiển nhiên đúng vì nó là BĐT Schur
bậc 3.

1. Một số phương pháp chứng minh BĐT thông dụng GV: Nguyễn Tuấn Anh
CHƯƠNG 2. BỔ SUNG KIẾN THỨC Trang 67

• p3 r + q 3 ≥ 6pqr

BĐT tương đương với:

pr p2 − 3q + q q 2 − 3pr ≥ 0
 

BĐT này hiển nhiên đúng theo chứng minh trên.


( )
2
p 4q − p
• r ≥ max 0;
9

p 4q − p2
Ta cần chứng minh: r ≥ . BĐT này tương đương với:
9
a (a − b) (a − c) + b (b − c) (b − a) + c (c − a) (c − b) ≥ 0

BĐT này hiển nhiên đúng vì đây là BĐT Schur bậc 3.


(  2 )
24q − p p −q
• r ≥ max 0;
6p
 
4q − p2 p2 − q
Ta cần chứng minh: r ≥ . BĐT này tương đương với:
6p
a2 (a − b) (a − c) + b2 (b − a) (b − c) + c2 (c − a) (c − b) ≥ 0

BĐT này hiển nhiên đúng vì đây là BĐT Schur bậc 4.

Ta thấy trong quá trình chứng minh các đánh giá đơn giản trên ta dùng rất nhiều BĐT
Schur. Đây là công cụ hữu hiệu khi kết hợp với kỹ thuật phân tích pqr trong chứng minh
BĐT.2

BĐT Schur

Với mỗi số thực dương a, b, c, k . BĐT sau luôn đúng:

ak (a − b) (a − c) + bk (b − a) (b − c) + ck (c − a) (c − b) ≥ 0

Chứng minh:
Do tính đối xứng của BĐT nên ta giả sử a ≥ b ≥ c. Khi đó:

ak (a − b) (a − c) ≥ bk (a − b) (b − c)

Vì vậy:
ak (a − b) (a − c) + bk (b − a) (b − c) + ck (c − a) (c − b)

≥ ck (c − a) (c − b) ≥ 0
2
Bạn đọc có thể tham khảo bài viết " BĐT Schur và ứng dụng - Trần Xuân Đáng - Toán học tuổi trẻ "

GV: Nguyễn Tuấn Anh 1. Một số phương pháp chứng minh BĐT thông dụng
Trang 68 CHƯƠNG 2. BỔ SUNG KIẾN THỨC

Ví dụ minh họa

Ví dụ 8.

Với các số thực dương x, y, z. Chứng minh rằng:

x4 + y 4 + z 4 ≥ xyz(x + y + z)

LỜI GIẢI. Để đơn giản ta chuẩn hóa p = x + y + z = 1 và đặt q = xy + yz + zx, r = xyz .


Khi đó BĐT trên tương đương với:

1 − 4q + 2q 2 + 3r ≥ 0

Ta lại có:
7q 2
9r + 2 ≥ 7q ⇔ 3r ≥ −
3 3
Do đó ta chỉ cần chứng minh:
7q 2 1
1 − 4q + 2q 2 + − ≥ 0 ⇔ (3q − 1) (2q − 1) ≥ 0
3 3 3
1
BĐT trên luôn đúng vì q ≤ . 
3
Ví dụ 9 (Iran - 1996).

Với các số thực dương x, y, z. Chứng minh rằng:


 
1 1 1 9
(xy + yz + zx) 2 + 2 + 2 ≥
(x + y) (y + z) (z + x) 4

LỜI GIẢI. Ta chú ý rằng:


" #2
X X
(x + y)2 (y + z)2 = (x + y) (y + z) − 4 (x + y + z) (x + y) (y + z) (z + x)
cyc cyc

Do đó BĐT đã cho tương đương:


2
2 !
p +q − 4p (pq − r) 9
q 2

(pq − r) 4

⇔ 4p4 q − 17p2 q 2 + 4q 3 + 34pqr − 9r2 ≥ 0

⇔ pq p3 + 9r − 4pqr + q p4 + 4q 2 + 6pr − 5p2 q + r (pq − 9r)


 

Theo các đánh giá trên bài toán được chứng minh. 
Một khó khăn khi dùng phương pháp pqr đó là khối lượng tính toán khá nhiều. Tuy
nhiên nếu thành thạo thì đây chỉ là kỹ năng cơ bản giống như kỹ năng cho phương pháp

1. Một số phương pháp chứng minh BĐT thông dụng GV: Nguyễn Tuấn Anh
CHƯƠNG 2. BỔ SUNG KIẾN THỨC Trang 69

SOS. Tiếp đến ta đến với hai ví dụ để ta thấy tính hiệu quả cũng như vẻ đẹp khi ta kết
hợp pqr và BĐT Schur.

Ví dụ 10 (BĐT schur và phương pháp biến đổi p, q, r - Võ Thành Văn).

Với các số thực dương x, y, z có tổng bằng 1. Chứng minh rằng:


1 1 1 247
+ + + 2xyz ≥
x+y y+z z+x 54

LỜI GIẢI. Ta chú ý x + y + z = 1. BĐT đã cho tương đương:


1 1 1 xyz 247
+ + +2 ≥
x+y y+z z+x x+y+z 54

Theo BĐT Schur ta được:


xyz 1  1
≥ 4 (xy + yz + zx) − (x + y + z)2 = [4 (xy + yz + zx) − 1]
x+y+z 9 9

Nên ta cần chứng minh:


1 + q 2 (4q − 1) 247
+ − ≥0
q−r 9 54

⇔ 48q 2 + 259r + 54 ≥ 48qr + 205q

 1
2 146
⇔ 48 q − + r (16 − 48q) + 243r − 173q + ≥0
3 3

Lại theo BĐT Schur ta lại có:



p 4q − p2 4q − 1
r≥ =
9 9

Do đó ta chỉ cần chứng minh:


1 2 146
 
48 q − + r (16 − 48q) + 27 (4q − 1) − 173q + ≥0
3 3

 1
2 65
⇔ 48 q − + 16r (1 − 3q) + (1 − 3q) ≥ 0
3 3
1
BĐT trên là hiển nhiên vì q ≤ 
3
Ví dụ 11 (Trần Nam Dũng).

Với các số thực dương x, y, z. Chứng minh rằng:

2 a2 + b2 + c2 + abc + 8 ≥ 5 (a + b + c)


GV: Nguyễn Tuấn Anh 1. Một số phương pháp chứng minh BĐT thông dụng
Trang 70 CHƯƠNG 2. BỔ SUNG KIẾN THỨC

LỜI GIẢI. Ta có:


9abc
4 a2 + b2 + c2 + 2abc + 16 ≥ 4 a2 + b2 + c2 +
 
+ 15
a+b+c

≥ 4 p2 − 2q + 4q − p2 + 15


Do vậy ta cần chứng minh:

4 p2 − 2q + 4q − p2 + 15 ≥ 10p


⇔ 3p2 − 10p + 15 − 4q ≥ 0

p2
Mà q ≤ do vậy ta cần có:
3
4p2 5
3p2 − 10p + 15 − ≥ 0 ⇔ (p − 3)2 ≥ 0
3 3
Bài toán được chứng minh. Đẳng thức xảy ra khi và chỉ khi a = b = c = 1. 
Ta đã thấy rõ tính linh hoạt khi ta thay ba biến x, y, z thành ba biến mới p, q, r có mối
quan hệ mật thiết với nhau, cũng như vẻ đẹp của bài chứng minh khi kết hợp với BĐT
Schur. Hơn nữa với những bài khó hơn ta không đơn thuần là biến đổi và dùng Schur
mà còn kết hợp với chia trường hợp để xử lý bài toán - Đó là cả một kỹ thuật, bài viết
sẽ không trình bày ở đây mời bạn đọc xem trong tài liệu tham khảo số 9.

1.4 Phương pháp dồn biến

Trong chứng minh BĐT ta thường gặp các BĐT có dạng: f (x, y, z) ≥ 0 để đánh giá
cho vế trái ta thường tìm một chặn dưới cho nó bằng cách áp dụng các BĐT phụ, hàm
số. . . Một trong số đó là tìm chặn dưới ứng với các biến “ít lệch” hơn (suy nghĩ này rất
tự nhiên vì hầu hết các đẳng thức xảy ra trong chứng minh BĐT là khi các biến bằng
nhau). Tùy vào mỗi bài mỗi giả thiết ta có một số cách khác nhau như:
√ √
x + y x + y 
f (x, y, z) ≥ f ; ; z ; f (x, y, z) ≥ f ( xy; xy; z)
2 2
r r !
x2 + y 2 x2 + y 2
f (x, y, z) ≥ f ; ;z
2 2

Khi đó thay vì chứng minh ta chuyển về chứng minh các chặn dưới đó lớn hơn hoặc
bằng 0 với số biến ít hơn (vì vậy phương pháp có tên là dồn biến).
Tài liệu sẽ điểm qua 3 ví dụ đại diện cho ba cách dồn biến thông dụng trong chứng
minh BĐT.

1. Một số phương pháp chứng minh BĐT thông dụng GV: Nguyễn Tuấn Anh
CHƯƠNG 2. BỔ SUNG KIẾN THỨC Trang 71

Ví dụ 12 (BĐT Nesbitt).

Với các số thực dương x, y, z. Chứng minh rằng:


x y z 3
+ + ≥
y+z z+x x+y 2

LỜI GIẢI. Do BĐT có dạng tổng nên ta chọn cách dồn biến:
x + y x + y 
f (x, y, z) ≥ f ; ;z
2 2

Xét
x y z 3
f (x, y, z) = + + −
y+z z+x x+y 2
Ta hy vọng có đánh giá sau luôn đúng:
x + y x + y  x y 2 (x + y)
f (x, y, z) − f ; ;z = + − ≥0
2 2 y + z z + x 2z + x + y

Thật vậy:
x y (x + y)2 2 (x + y)
+ ≥ ≥
y+z z+x 2xy + xz + yz 2z + x + y
Do vậy thay vì chứng minh bài toán với 3 biến x, y, z giờ ta chỉ chứng minh BĐT với 2
biến t, z :
2t z 3
+ ≥
t + z 2t 2

(t − z)2
⇔ ≥0
2 (t + z) t

Nhận xét: Nếu ta kết hợp chuẩn hóa thì ta chỉ cần chứng minh BĐT với 1 biến khi đó
công việc trở nên dễ dàng hơn.

Ví dụ 13 (MOSP-2001).

Với các số thực dương x, y, z thỏa mãn xyz = 1. Chứng minh rằng:

(x + y)(y + z)(z + x) ≥ 4(x + y + z − 1)

LỜI GIẢI. BĐT có điều kiện tích ba biến là hằng số nên ta sẽ chọn cách dồn biến:
√ √
f (x, y, z) ≥ f ( xy, xy, z)

Xét
f (x, y, z) = (x + y)(y + z)(z + x) − 4(x + y + z − 1)

GV: Nguyễn Tuấn Anh 1. Một số phương pháp chứng minh BĐT thông dụng
Trang 72 CHƯƠNG 2. BỔ SUNG KIẾN THỨC

Ta cần đánh giá sau đây là đúng:


√ √ 
f (x, y, z) − f xy, xy, z

√  √  √ 
= xy x + y − 2 xy + z x2 + y 2 + xz + yz − 2xy − 2z xy − 4 x + y − 2 xy

√ √ 2
h √ √ 2
i √ √ 2
= xy x − y + z (x − y)2 + z x − y −4 x− y

√ √ 2 2
h √ √ 2
i
= x− y z + xy + z x + y − 4

√ √ 2  √ 
= x− y (z + x)(z + y) + 2z xy − 4

Ta lại có:
p √
(z + x)(z + y) ≥ 4 z 2 xy = 4 z

vế trái của BĐT trên sẽ lớn hơn hoặc bằng 4 nếu z ≥ 1. Điều này hoàn toàn có được
bằng cách sắp lại thứ tự các biến trong BĐT (vì BĐT là đối xứng).
Tóm lại ta chỉ cần chứng minh:

2t(t + z)2 − 4 (2t + z − 1) ≥ 0; (t = xy)

 1
2  1

⇔ 2t t + − 4 2t + −1 ≥0
t2 t2

(t − 1)2 t4 + 2t3 + 1 − t2

⇔ ≥0
t3

BĐT cuối luôn đúng vì t ≤ 1. 


Nhận xét:Phương pháp dồn biến sẽ trở nên hiệu quả hơn nếu ta biết áp dụng kết hợp
với các kỹ thuật khác: Sắp thứ tự biến, chuẩn hoá, đạo hàm,...
Bây giờ ta lại xét lại một ví dụ cũ bên trên nhưng với cách nhìn mới hơn.

Ví dụ 14 (All-Russian Olympiad 2002).

Với ba số thực dương a, b, c có tổng bằng 3. Chứng minh rằng:


√ √ √
a + b + c ≥ ab + bc + ca
√ √ √
LỜI GIẢI. Để thuận tiện trong phép chứng minh ta sẽ đặt a = x; b = y; c = z . Khi
đó giả thiết bài toán sẽ là:
x2 + y 2 + z 2 = 3

1. Một số phương pháp chứng minh BĐT thông dụng GV: Nguyễn Tuấn Anh
CHƯƠNG 2. BỔ SUNG KIẾN THỨC Trang 73

Nên ta chọn cách dồn biến:


r r !
x2 + y 2 x2 + y 2
f (x, y, z) ≥ f , ,z
2 2

với f (x, y, z) = x + y + z − x2 y 2 − y 2 z 2 − z 2 x2
Ta hy vọng
r r ! r ! " 2 #
x2 + y 2 x2 + y 2 x2 + y 2 x2 + y 2

f (x, y, z) − f , ,z = x+y−2 − x2 y 2 −
2 2 2 2

r ! 2
x2 + y 2 x2 − y 2

= x+y−2 +
2 2

 
2
−(x − y)2 x2 − y 2

 
= r +

x2 + y 2 2
x+y+2
2
 
 (x + y)2 1
= (x − y)2 

− r ≥0
 4 x2 + y 2
x+y+2
2

Đánh giá trên không phải lúc nào cũng đúng, do đó ta cần sắp lại thứ tự các biến và
hy vọng khi đó nó sẽ đúng.

Không mất tính tổng quát, ta giả sử x ≥ y ≥ z . Khi đó z ≤ 1; x + y ≥ 2 và:

(x + y)2 1 1 1
− r ≥ − √ >0
4 x2 + y 2 2 2+ 2
x+y+2
2

Vậy ta chỉ cần chứng minh:


r r ! r
x2 + y 2 x2 + y 2 p x2 + y 2
3 − 2t2 − t4 − 2t2 3 − 2t 2

f , ,z = 2t + ≥ 0; vớit = ≥1
2 2 2

Mà p
3 − 2t2 − t4 − 2t2 3 − 2t2

2t +


p 2 2 t2 − 1
= 3t4 − 6t2 + 2t + 3 − 2t2 = 3 t2 − 1 + 2 (t − 1) + √ ≥0
3 − 2t2 + 1
Bài toán được chứng minh. 

GV: Nguyễn Tuấn Anh 1. Một số phương pháp chứng minh BĐT thông dụng
Trang 74 CHƯƠNG 2. BỔ SUNG KIẾN THỨC

Trên là ba cách dồn biến thông dụng. Đương nhiên tùy mỗi bài toán khác nhau ta sẽ
chọn cách dồn biến khác nhau 3

Để hiểu hơn về phương pháp, mời bạn đọc thử sức với các bài toán sau:

Bài 2.10 (PTNK - Năm 2016).

Tìm số nguyên dương k nhỏ nhất sao cho BĐT sau:

xk y k z k x3 + y 3 + z 3 ≤ 3


đúng với mọi số thực dương x, y, z thỏa mãn x + y + z = 3.

Bài 2.11 (Đề chọn đội tuyển chuyên Nguyễn Du - Đăk Lăk lần 1 - 2016).

Với ba số thực dương a, b, c thỏa mãn abc = 1. Chứng minh rằng:


1 1 1
+ + ≤1
2+a 2+b 2+c

Ngoài ra bạn đọc hãy xem ví dụ sau đây để có một hướng nhìn khác về cách dồn biến
- không đơn thuần là các phép biến đổi tương đương.

Ví dụ 15 (IMO-1984).

Với ba số thực không âm x, y, z có tổng bằng 3. Chứng minh rằng:


7
0 ≤ xy + yz + xz − 2xyz ≤
27

LỜI GIẢI.
x+y
Ý tưởng: Giả sử ta cần đánh giá f (x, y, z) ≥ f (t; t; z) ; t = khi đó ta sẽ khảo sát
2
hàm số:
g(ε) = f (t + ε; t − ε; z)

với ε ≥ 0. Ta chứng minh đây là hàm tăng (bằng công cụ đạo hàm) vì vậy:
x − y 
f (x, y, z) = g ≥ g(0) = f (t; t; z)
2
Qua trở lại bài toán, ta chứng minh:
7
xy + yz + xz − 2xyz ≤
27
Xét
7
f (x, y, z) = xy + yz + xz − 2xyz −
27
1 x+y 1
với giả sử x ≥ y ≥ z . Với mỗi (x, y, z) cố định ta đặt ≤t= ≤
3 2 2
3
Bạn đọc nên xem quyển " Sáng tạo BĐT - Phạm Kim Hùng " để thấy được những cách dồn biến rất đặc
biệt.

1. Một số phương pháp chứng minh BĐT thông dụng GV: Nguyễn Tuấn Anh
CHƯƠNG 2. BỔ SUNG KIẾN THỨC Trang 75

• Xét hàm số:


7
g(ε) = f (t+ε, t−ε, z) = (t + ε) (t − ε)+2t (1 − 2t)−2 (t + ε) (t − ε) (1 − 2t)− ; (ε ≥ 0)
27

g 0 (ε) = −2ε + 4ε (1 − 2t) = ε (2 − 8t) ≤ 0

Vì vậy:
g(ε) ≤ g(0)

• Từ trên ta được:
x − y 
g ≤ g(0)
2

7 7
⇔ xy + yz + xz − 2xyz − ≤ t2 + 2t (1 − 2t) − 2t2 (1 − 2t) −
27 27

7 1
t2 + 2t (1 − 2t) − 2t2 (1 − 2t) − = (3t − 1)2 (12t − 7) ≤ 0
27 27
Bài toán được chứng minh. 

1.5 Phương pháp SOS

SOS là viết tắt của Sum of Square. Ý tưởng phương pháp là đưa bài toán chứng minh
BĐT về phân tích thành tổng các đại lượng bình phương. Ví dụ:
1
a3 + b3 + c3 − 3abc = (a + b + c) (a − b)2 + (b − c)2 + (c − a)2 ≥ 0
 
2
Vậy là ta chứng minh được BĐT AM - GM ba biến.

Ví dụ 16 (Moldova MO 2006).

Với a, b, c là độ dài ba cạnh của tam giác. Chứng minh rằng:


 
2 b c  a 
a − 1 + b2 − 1 + c2 −1 ≥0
c a b

LỜI GIẢI. BĐT trên tương đương với:


a3 b2 + b3 c2 + c3 a2 ≥ abc a2 + b2 + c2


⇔ a2 (a − b + c) (b − c)2 + b2 (a + b − c) (a − c)2 + c2 (b + c − a) (a − b)2 ≥ 0


Nhận xét:Lời giải trên rất đẹp tuy nhiên có 2 vấn đề phát sinh:

GV: Nguyễn Tuấn Anh 1. Một số phương pháp chứng minh BĐT thông dụng
Trang 76 CHƯƠNG 2. BỔ SUNG KIẾN THỨC

• Để biến đổi thu được BĐT cuối không phải là chuyện dễ dàng, vậy có một cách
thức nào cho việc phân tích này?

• Nếu các phần

Sa = a2 (a − b + c) ; Sb = b2 (a + b − c) ; Sc = c2 (b + c − a)

trong một bài toán khác chúng không đảm bảo là không âm vậy có chứng minh
được BĐT hay không?

Chính hai vấn đề trên là động lực để phương pháp SOS xuất hiện.

Các phân tích cơ bản

Một số phân tích sau sẽ giúp chúng ta rút ngắn thời gian khi sử dụng SOS cũng như có
định hướng cho các phép biến đổi

• a2 + b2 − 2ab = (a − b)2

a b (a − b)2
• + −2=
b a ab
• a3 + b3 − ab(a + b) = (a + b)(a − b)2
1
• a2 + b2 + c2 − ab − bc − ca = (a − b)2 + (b − c)2 + (c − a)2

2
1 1
• a2 + b2 + c2 − (a + b + c)2 = (a − b)2 + (b − c)2 + (c − a)2

3 3
• (a + b)(b + c)(c + a) − 8abc = a(b − c)2 + b(c − a)2 + c(a − b)2
1
• a3 + b3 + c3 − 3abc = (a + b + c) (a − b)2 + (b − c)2 + (c − a)2
 
2
p (a − b)2
• 2(a2 + b2 ) − (a + b) = p
2(a2 + b2 ) + (a + b)

a b c 3 X (a − b)2
• + + − =
b+c c+a a+b 2 2(a + c)(b + c)
Sym

7a + b + c a + 7b + c a + b + 7c
• (a + b + c)3 − 27abc = (b − c)2 + (c − a)2 + (a − b)2
2 2 2

Định lý SOS

Xét biểu thức


S = f (a, b, c) = Sa (b − c)2 + Sb (a − c)2 + Sc (a − b)2

trong đó Sa ; Sb ; Sc là các hàm số theo a, b, c

1. Một số phương pháp chứng minh BĐT thông dụng GV: Nguyễn Tuấn Anh
CHƯƠNG 2. BỔ SUNG KIẾN THỨC Trang 77

1. Nếu Sa ; Sb ; Sc ≥ 0 thì S ≥ 0.

2. Với a ≥ b ≥ c và:

 Sb ≥ 0


• bS + S ≥ 0 thì S ≥ 0.
a


 S +S ≥0
b c

Mở rộng:
(
Sa ≥ Sb ≥ Sc
+ Nếu thì S ≥ 0.
Sb + Sc ≥ 0
(
Sa ≤ Sb ≤ Sc
+ Nếu thì S ≥ 0.
Sb + Sa ≥ 0

 Sa ; Sc ≥ 0


• aS + 2S ≥ 0 thì S ≥ 0.
b


 S + 2S ≥ 0
c b
(
Sb ; Sc ≥ 0
• thì S ≥ 0.
a2 Sb + b2 Sa ≥ 0
(
Sa + Sb + Sc ≥ 0
3. Nếu thì S ≥ 0.
Sa Sb + Sb Sc + Sc Sa ≥ 0

Chứng minh:4

1. Nếu Sa ; Sb ; Sc ≥ 0 thì S ≥ 0. (hiển nhiên)

2. Với a ≥ b ≥ c và:

 Sb ≥ 0


• Ta có: Sb + Sa ≥ 0


 S +S ≥0
b c

Vì a ≥ b ≥ c ⇒ (a − c)2 ≥ (a − b)2 + (b − c)2 nên

S = Sa (b − c)2 +Sb (a − c)2 +Sc (a − b)2 ≥ (Sa + Sb ) (b − c)2 +(Sc + Sb ) (a − b)2 ≥ 0



 Sa ; Sc ≥ 0


• Ta có: S + 2S ≥ 0
a b


 S + 2S ≥ 0
c b

4
Khi làm một bài thi mà sử dụng phương pháp SOS, học sinh phải kèm phép chứng minh vào bài giải tránh
bị mất điểm.

GV: Nguyễn Tuấn Anh 1. Một số phương pháp chứng minh BĐT thông dụng
Trang 78 CHƯƠNG 2. BỔ SUNG KIẾN THỨC

Nếu Sb ≥ 0 kết hợp với Sa ; Sc ≥ 0 ta được điều phải chứng minh. Ở đây ta chỉ
xét với Sb ≤ 0.
Vì (a − c)2 ≤ 2(a − b)2 + 2(b − c)2 nên:

S = Sa (b − c)2 +Sb (a − c)2 +Sc (a − b)2 ≥ (Sa + 2Sb ) (b − c)2 +(Sc + 2Sb ) (a − b)2 ≥ 0

(
Sb ; Sc ≥ 0
• Ta có:
a2 Sb + b2 Sa ≥ 0
a−c a
Vì a ≥ b ≥ c nên ≥ khi đó:
b−c b
  a − c 2    a 2 
2 2 2 2
Sa (b − c) + Sb (a − c) = (b − c) Sa + Sb . ≥ (b − c) Sa + Sb .
b−c b

( 
Sb ; Sc ≥ 0  a 2 
2
Vì vậy thì S ≥ (b − c) Sa + Sb . + Sc (a − b)2 ≥ 0
a2 S 2
b + b Sa ≥0 b

(
Sa + Sb + Sc ≥ 0
3. Ta có:
Sa Sb + Sb Sc + Sc Sa ≥ 0
(
Sa < 0
Ta chứng minh khi các trường hợp khác tương tự.
Sb + Sc ≥ 0

Ta có:

(Sb + Sc ) Sa (b − c)2 + Sb (a − c)2 + Sc (a − b)2


 

= (Sb + Sc )2 a2 − 2 (Sb + Sc ) (bSc + cSb ) a + (Sb + Sc ) c2 Sb + b2 Sc + Sa (b − c)2


 

= [(Sb + Sc ) a − (bSc + cSb )]2 + (Sa Sb + Sb Sc + Sc Sa ) (b − c)2 ≥ 0

Ví dụ minh họa

Ví dụ 17 (BĐT Schur).

Với a, b, c là ba số thực không âm. Chứng minh rằng:

a3 + b3 + c3 + 3abc ≥ ab(a + b) + bc(b + c) + ca(c + a)

LỜI GIẢI. Không mất tính tổng quát ta giả sử a ≥ b ≥ c

1. Một số phương pháp chứng minh BĐT thông dụng GV: Nguyễn Tuấn Anh
CHƯƠNG 2. BỔ SUNG KIẾN THỨC Trang 79

BĐT trên tương đương:

⇔ a3 + b3 + c3 − 3abc ≥ ab(a + b) + bc(b + c) + ca(c + a) − 6abc

1
⇔ (a + b + c) (a − b)2 + (b − c)2 + (c − a)2 ≥ a(b − c)2 + b(c − a)2 + c(a − b)2
 
2

−a + b + c a−b+c a+b−c
⇔ (b − c)2 + (c − a)2 + (a − b)2 ≥ 0
2 2 2

Khi đó ta thấy:

a−b+c

 Sb = ≥0



 2



a − b + c −a + b + c

Sb + Sa = + =c≥0

 2 2




 Sb + Sc = a − b + c + a + b − c = a ≥ 0



2 2

Do đó theo định lý SOS ta được điều phải chứng minh. 

Ví dụ 18.

Với a, b, c là ba số thực dương. Chứng minh rằng:


P 2
a X bc 5
2 ≥
P +2
ab (b + c) 2

LỜI GIẢI. Không mất tính tổng quát ta giả sử a ≥ b ≥ c.


BĐT tương đương với:

a2
P
X bc 1

P −1+2 − ≥0
ab (b + c)2 4

1P
(a − b)2 X  (b − c)2 
2
⇔ P +2 ≥0
ab 4(b + c)2

 
X 1 1  (a − b)2 ≥ 0
⇔  P − 2
2 ab 2(a + b)

GV: Nguyễn Tuấn Anh 1. Một số phương pháp chứng minh BĐT thông dụng
Trang 80 CHƯƠNG 2. BỔ SUNG KIẾN THỨC

Vậy nên:
1 1 1 1
Sa = − ; Sb = − ;
2 (ab + bc + ac) 2(b + c)2 2 (ab + bc + ac) 2(c + a)2

1 1
Sc = −
2 (ab + bc + ac) 2(a + b)2

Ta thấy rằng 0 ≤ Sb ≤ Sc do đó theo định lý SOS ta cần b2 Sa + a2 Sb ≥ 0


Thật vậy,
a2 + b 2 b2 a2
b2 Sa + a2 Sb = − −
2 (ab + bc + ac) 2(b + c)2 2(c + a)2

a3 c + b3 c − a2 bc − ab2 c + a3 b + b3 a − 2a2 b2 AM −GM


= ≥ 0
2 (ab + bc + ac) (b + c)2 (c + a)2

Bài toán được chứng minh. 


Bài viết sẽ không liệt kê ra các ví dụ áp dụng định lý SOS nữa (vì như vậy sẽ quá dài)
mà để bạn đọc tự trải nghiệm nó 5 . Đôi khi ý tưởng của phương pháp cũng mang một
giá trị quý không kém gì phương pháp, ví dụ cuối cùng sau minh chứng cho điều đó.

Ví dụ 19.

Với ba số thực dương a, b, c. Chứng minh rằng:


a b c a b c 3
+ + ≤ + + −
b+c c+a a+b b c a 2

LỜI GIẢI. Không mất tính tổng quát, ta giả sử c là số nhỏ nhất trong ba số a, b, c.
Ta có:
 
a b c a b c 3
+ + ≤ + + −
b+c c+a a+b b c a 2

 
2a 2b 2c a b c
⇔ + + −3≤2 + + −3
b+c c+a a+b b c a

(a − b)2 (a − b)2 (a − c) (b − c)
 
a + b + 2c
⇔ + (a − c) (b − c) ≤ 2 +
(b + c) (c + a) (b + c) (c + a) (a + b) ab ac

   
2 1 2 a + b + 2c
⇔ − (a − b)2 + − (a − c) (b − c) ≥ 0
ab (b + c) (c + a) ac (b + c) (c + a) (a + b)

Vế trái là biểu thức của những số hạng không âm do đó bài toán được chứng minh. 
5
Hãy chọn một BĐT đối xứng ba biến và cố gắng phân tích để sử dụng định lý SOS.

1. Một số phương pháp chứng minh BĐT thông dụng GV: Nguyễn Tuấn Anh
CHƯƠNG 2. BỔ SUNG KIẾN THỨC Trang 81

Phương phap SOS không chỉ là công cụ mạnh, mà ý tưởng của nó còn len lỏi vào rất
nhiều bài chứng minh BĐT. Kết lại phần này mời bạn đọc hãy đến với các bài toán sau:

Bài 2.12 (Đề thi chọn HSG lớp 12 - Hà Nội - Năm 2016).

Với ba số thực dương a, b, c thỏa mãn ab + bc + ac + 2abc = 1. Tìm giá trị nhỏ nhất của biểu
thức sau:
1 1 1
P = + + − 2(a + b + c)
a b c

Bài 2.13 (Đề kiểm tra đội tuyển toán chuyên Bảo Lộc lần 3-2016).

Với ba số thực a, b, c ∈ (0; 1). Chứng minh rằng:

a − a2 b − b2 c − c2 ≥ (a − bc) (b − ca) (c − ab)


  

Bài 2.14 (Đề thi chọn đội tuyển THPT chuyên Đại học Vinh - Năm 2016).

Tìm tất cả các số thực k sao cho bất đẳng thức sau đúng với mọi số thực a, b, c

(a + b + c)2
+ k max (a − b)2 , (b − c)2 , (c − a)2 ≤ a2 + b2 + c2

ab + bc + ca ≤
3

GV: Nguyễn Tuấn Anh 1. Một số phương pháp chứng minh BĐT thông dụng
Trang 82 CHƯƠNG 2. BỔ SUNG KIẾN THỨC

2 Một số kỹ thuật, BĐT phụ đặc biệt trong chứng minh


BĐT

2.1 Một số phép đổi biến trong chứng minh BĐT

Trong chứng minh bất đẳng thức, việc đổi biến qua các phép đặt ẩn phụ thường xuyên
xuất hiện, nó cho phép người làm nhìn nhận bài toán với một góc độ khác hơn, đôi lúc
đó là chìa khóa để giải bài toán, đôi lúc là tiền đề để ta nhìn nhận lại bài toán, chọn lọc
cho nó một phương pháp giải hợp lý hơn mà nếu ngay từ đầu sẽ khó nhận ra. Phương
pháp biến đổi pqr là một minh chứng cho điều đó, tuy nhiên vẫn còn rất nhiều cách đổi
biến khác nếu chưa từng gặp qua thì sẽ rất khó khăn trong việc liên kết các giả thiết.
Vì tính không mẫu mực đó, bài viết sẽ điểm qua các cách đổi biến, với nhiều góc độ
nhất có thể để từ đó bạn đọc có được nhiều lời giải thú vị cho cùng một bài toán.
a. Một số phép đổi biến cơ bản

Các phép đổi biến sơ cấp: Tịnh tiến, co giãn, nghịch đảo...

Ý tưởng: Ta sẽ sử dụng các phép đổi biến


 
1 1 1
(x, y, z) → (x + k, y + k, z + k) ; (x, y, z) → , , ; ...
x y z

để biến đổi bài toán, qua đó có một cách nhìn khác cho bài toán.

Ví dụ 20.
Với x, y, z là các số thực dương có tích bằng 8. Chứng minh rằng
1 1 1
+ 2 + 2 ≥1
x2 −x+1 y −y+1 z −z+1

LỜI GIẢI. Từ hình dáng của bài toán làm ta liên tưởng đến một bổ đề rất chặt của
Võ Quốc Bá Cẩn - Vasile Cirtoaje: Với ba số thực dương có tích bằng 1 thì BĐT sau
luôn đúng:
1 1 1
+ + ≥1
x2 + x + 1 y 2 + y + 1 z 2 + z + 1
Dễ dàng đoán được điểm rơi của BĐT là x = y = z = 2 nên để đưa về bổ đề, một ý
tưởng là ta tịnh tiến các biến x, y, z qua một phép biến đổi: (x, y, z) → (a + 1; b + 1; c + 1)
và BĐT cần chứng minh là:
1 1 1
+ 2 + 2 ≥1
a2 +a+1 b +b+1 c +c+1
Giống với dạng của bổ đề. Bài toán sẽ được giải quyết nếu ta giải quyết được hai vấn
đề sau:

2. Một số kỹ thuật, BĐT phụ đặc biệt trong chứng minh BĐT GV: Nguyễn Tuấn Anh
CHƯƠNG 2. BỔ SUNG KIẾN THỨC Trang 83

• a, b, c chưa phải là những số dương: Ta thấy nếu x, y, z có số nào đó không lớn 1 thì
a, b, c tương ứng khi đó sẽ không dương và ta không dùng được bổ đề. Tuy nhiên
nếu tình huống đó xảy ra thì BĐT ban đầu lại hiển nhiên đúng. Hay nói cách khác
ta chỉ cần chứng minh trong trường hợp x, y, z > 1 tức a, b, c > 0.

• Thế nhưng khi đó abc ≤ 1. Muốn dùng được bổ đề ta cần có abc = 1, ta lại sử dụng
ý tưởng tịnh tiến biến. Cụ thể, tồn tại k ≥ 0 sao cho: (a + k)bc = 1 theo bổ đề thì:
1 1 1
2
+ + 2 ≥1
(a + k) + (a + k) + 1 b2 +b+1 c +c+1

Vì cách chọn của k nên ta có:


1 1 1 1 1 1
+ 2 + 2 ≥ 2
+ 2 + 2 ≥1
a2 +a+1 b +b+1 c +c+1 (a + k) + (a + k) + 1 b + b + 1 c + c + 1

Vậy bài toán được chứng minh. 


Trên là ví dụ mà ta đổi biến dựa vào biểu thức cần chứng minh. Tiếp theo là hai ví dụ
mà ý tưởng đổi biến dựa hoàn toàn vào điều kiện của bài toán. Đây cũng là công dụng
của phép đổi biến mà người làm nên lưu ý, nhất là với các bài toán có điều kiện khá
“lạ”.

Ví dụ 21 (Iran 2002).

Với x, y, z là các số thực dương thỏa mãn x2 + y 2 + z 2 + xyz = 4. Chứng minh rằng

x+y+z ≤3

LỜI GIẢI. Ta dễ dàng đoán điểm rơi của BĐT là x = y = z = 1, suy nghĩ tự nhiên (giả
sử là ta chưa biết cách khai thác điều kiện mà bài toán cho) ta sẽ tịnh tiến biến: Đặt
x = a + 1, y = b + 1, z = c + 1 khi đó điều kiện bài toán cho sẽ là:

(a + 1)2 + (b + 1)2 + (c + 1)2 + (a + 1) (b + 1) (c + 1) = 4


1 1 2
⇔ (a + b + c)2 + 3 (a + b + c) +

a + b2 + c2 + abc = 0 (∗)
2 2

Giả sử ngược lại, ta có a + b + c > 0 thì điều gì sẽ xảy ra? Ta chú ý −1 < a, b, c < 1 nên:
1 2  3√ 3
a + b2 + c 2 ≥ a2 b2 c2 > abc
2 2

Nhưng như vậy là trái với đẳng thức khai thác bên trên. Bài toán được chứng minh. 

GV: Nguyễn Tuấn Anh 2. Một số kỹ thuật, BĐT phụ đặc biệt trong chứng minh BĐT
Trang 84 CHƯƠNG 2. BỔ SUNG KIẾN THỨC

Ví dụ 22 (Chọn đội tuyển Thanh Hóa – Vòng 2, năm 2015 -2016).

1 1 1
Với x, y, z là các số thực dương thỏa mãn x + y + z = + + . Chứng minh rằng
x y z
1 1 1 3
2 + 2 + 2 ≤
(2xy + yz + zx) (2yz + zx + xy) (2zx + xy + yz) 16x y 2 z 2
2

 
1 1 1
LỜI GIẢI. Từ giả thiết, với (x, y, z) thỏa điều kiện thì , , cũng thỏa điều kiện.
x y z
Ta
 có một
 ý tưởng là thay vì chứng minh bài toán với (x, y, z) ta sẽ chứng minh với
1 1 1
, , . BĐT cần chứng minh là:
x y z

1 1 1 3
2
+ 2
+ 2

(2x + y + z) (2y + z + x) (2z + x + y) 16

Theo BĐT AM - GM ta có:


 
1 1 11 1 1 1
2
+ 2
+ 2
≤ + +
(2x + y + z) (2y + z + x) (2z + x + y) 4 (x + y) (x + z) (y + z) (y + x) (z + x) (z + y)

Ta cần chứng minh:

1 1 1 3
+ + ≤
(x + y) (x + z) (y + z) (y + x) (z + x) (z + y) 4

⇔ 3 (x + y) (y + z) (z + x) ≥ 8 (x + y + z)

Lại theo BĐT AM – GM ta được:

9 (x + y) (y + z) (z + x) ≥ 8 (x + y + z) (xy + yz + zx)

Nên ta chỉ cần chứng minh: xy + yz + zx ≥ 3 . Mà điều này là hiển nhiên vì

1 1 1
x+y+z = + +
x y z


Nhận xét: Qua ba ví dụ trên, ta thấy phép đổi biến cho phép người làm tiến gần tới
một kết quả quen thuộc nào đó hoặc chí ít là đưa đến một ý tưởng mới cho bài toán.
Hơn nữa bằng cách phối hợp các phép biến đổi này người làm toán có thể tạo ra các
bài toán “lạ” mà bản chất của nó đã được che giấu đi, thậm chí là thêm bớt vào đó để
làm bài toán “lạ” này khó hơn. Đây là mảnh đất hứa hẹn nhiều sáng tạo, người viết mời
bạn đọc khai phá.

2. Một số kỹ thuật, BĐT phụ đặc biệt trong chứng minh BĐT GV: Nguyễn Tuấn Anh
CHƯƠNG 2. BỔ SUNG KIẾN THỨC Trang 85

Đổi biến với giả thiết tích các biến là hằng số

Ý tưởng: Với giả thiết tích các biến là hằng số k 3 (trong bài viết sẽ đề cập đến ba biến,
trường hợp nhiều biến hơn ta làm tương tự) ta có thể thực hiện các phép biến đổi sau:
(Phần chứng minh vì sao có phép biến đổi đó dành lại cho bạn đọc)
   2 2 2
  
ka kb kc ka kb kc kbc kca kab
(x, y, z) → , , ; (x, y, z) → , , ; (x, y, z) → , , ...
b c a bc ca ab a2 b 2 c 2
Vấn đề đặt ra là sử dụng phép biến đổi nào là hợp lý. Ta có thể dựa vào một số yếu tố
sau để lựa chọn phép biến đổi: Bậc của tử và mẫu thức, BĐT dạng đối xứng hay hoán
vị,... Để thấy được hiệu quả của phép đổi biến này cũng như đặt như thế nào là hợp
lý ta đến với bài toán sau, một bài toán chặt và có nhiều ứng dụng trong chứng minh
BĐT.

Ví dụ 23 (Võ Quốc Bá Cẩn - Vasile Cirtoaje).


Với x, y, z là các số thực dương có tích bằng 1. Chứng minh rằng
1 1 1
+ 2 + 2 ≥1
x2 +x+1 y +y+1 z +z+1
 
a b c
LỜI GIẢI. Ta thử sử dụng phép biến đổi (x, y, z) → , , . Khi đó BĐT trở thành:
b c a
b2 c2 a2
+ + ≥1
a2 + ab + b2 b2 + bc + c2 c2 + ca + a2
Hình dáng bài toán bấy giờ làm ta liên tưởng đến BĐT Cauchy dạng phân thức. Tuy
nhiên nếu áp dụng trực tiếp:
b2 c2 a2 (a + b + c)2
+ + ≥
a2 + ab + b2 b2 + bc + c2 c2 + ca + a2 2 (a2 + b2 + c2 ) + ab + bc + ca
Và ta cần có
(a + b + c)2
≥1
2 (a2 + b2 + c2 ) + ab + bc + ca
nhưng điều này là sai. Điều đó có nghĩa là:

• BĐT thu được sau phép đổi biến không đơn thuần là sử dụng trực tiếp Cauchy
như trên.

• Hoặc là phép đổi biến của ta là chưa hợp lý. Ta thử với phép đổi biến sau:
 
bc ca ab
(x, y, z) → , ,
a2 b 2 c 2
và bài toán cần chứng minh là:
a4 b4 c4
+ + ≥1
b2 c2 + bca2 + a4 c2 a2 + cab2 + b4 b2 a2 + bac2 + c4
BĐT trên được giải quyết đơn giản bằng BĐT Cauchy.

GV: Nguyễn Tuấn Anh 2. Một số kỹ thuật, BĐT phụ đặc biệt trong chứng minh BĐT
Trang 86 CHƯƠNG 2. BỔ SUNG KIẾN THỨC

Bài toán được giải quyết. 


Nhận xét:

• Ta thấy nếu thực hiện phép đổi biến thích hợp bài toán sẽ hiện rõ bản chất được
giấu đi. Để hiểu hơn bạn đọc hãy thử với phép đổi biến
 2 2 2
a b c
(x, y, z) → , ,
bc ca ab

sẽ hiểu tại sao ta lại chọn phép đổi biến bên trên.

• Trong lời giải, với phép đổi biến


 
a b c
(x, y, z) → , ,
b c a

vẫn có thể giải quyết được bài toán nhưng phải tinh tế hơn trong khâu thêm bớt
mới sử dụng BĐT Cauchy sau đó.

• Với kết quả này ta có thể chứng minh trực tiếp ví dụ 1 qua đánh giá sau:

1 1
≥  4  2
x2 −x+1 x x
+ +1
2 2

Đổi biến liên quan đến lượng giác

Ý tưởng: Với các bài toán BĐT có điều kiện đặc biệt, chứa căn, dạng phân thức... nhưng
chúng có đặc điểm chung là có hình dáng “tựa” với một công thức lượng giác nào đó.
Một ý tưởng nên làm khi đó là đổi biến về giá trị lượng giác, với mục tiêu vận dụng
được các công thức đặc biệt của lượng giác.

Ví dụ 24.

Với 2018 là các số thực dương a1 , a2 , ..., a2018 thỏa mãn


1 1 1
2
+ 2
+ ... + =1
1 + a1 1 + a2 1 + a22018

Chứng minh rằng


a1 a2 ...a2018 ≥ 20171009
1
LỜI GIẢI. Từ giả thiết làm ta liên tưởng đến công thức lượng giác 2
= cos2 x ,
1 + tan x
π
rất tự nhiên ta sẽ đặt ai = tan xi , 0 < xi < , i = 1, 2, ..., 2018 khi đó giả thiết bài toán là:
2

cos2 x1 + cos2 x2 + ... + cos2 x2018 = 1

2. Một số kỹ thuật, BĐT phụ đặc biệt trong chứng minh BĐT GV: Nguyễn Tuấn Anh
CHƯƠNG 2. BỔ SUNG KIẾN THỨC Trang 87

và ta cần chứng minh:

tan x1 . tan x2 ... tan x2018 ≥ 20171009 ⇔ sin x1 . sin x2 ... sin x2018 ≥ 20171009 cos x1 . cos x2 ... cos x2018

BĐT trên được chứng minh đơn giản bằng đánh giá sau: (tương tự cho các đánh giá
còn lại)
p
sin2 x1 = 1 − cos2 x1 = cos2 x2 + ... + cos2 x2018 ≥ 2017
2018
cos2 x2 .cos2 x3 .cos2 x2018

Và bài toán được chứng minh. 


Vấn đề đặt ra là biểu thức như thế nào thì ta nên lượng giác hóa và lượng giác hóa như
thế nào là hợp lý. Ta có một số cách thường dùng như sau:
Tiếp theo ta đến với một phép đổi biến được sử dụng với giả thiết x, y, z là độ dài ba
cạnh của tam giác.

Biểu thức đại số Công thức LG liên tưởng Cách LG hóa


"
√ p
2 x = sin t
2
1−x 1 − sin x = |cos x|
x = cos t
1
1 + x2 1 + tan2 x = x = tan t
cos2 x
2x 2 tan x
tan 2x = x = tan t
1 − x2 1 − tan2 x
4x3 − 3x cos 3x = 4cos3 x − 3 cos x x = cos t
..... ..... .....

Phép biến đổi Ravi

Ý tưởng của phép biến đổi dựa vào định lý sau:


Điều kiện cần và đủ để x, y, z là độ dài ba cạnh của tam giác là tồn tại ba số thực dương
a, b, c sao cho: x = a + b, y = b + c, z = c + a.
Định lý trên cho phép ta chuyển một bài toán BĐT hình học về BĐT đại số và ngược
lại.

Ví dụ 25 (T7/426).

Với x, y, z là độ dài ba cạnh của tam giác. Chứng minh rằng:



p 3 3xyz
(y + z − x) (x + z − y) (x + y − z) ≤ √
(x + y + z) x + y + z

LỜI GIẢI. Theo định lý trên tồn tại a, b, c > 0 sao cho x = a + b, y = b + c, z = c + a. Khi
đó BĐT trở thành:
p √
8 (a + b + c) abc (a + b + c) ≤ 3 3 (a + b) (b + c) (c + a)

GV: Nguyễn Tuấn Anh 2. Một số kỹ thuật, BĐT phụ đặc biệt trong chứng minh BĐT
Trang 88 CHƯƠNG 2. BỔ SUNG KIẾN THỨC

Sử dụng BĐT
9 (a + b) (b + c) (c + a) ≥ 8 (a + b + c) (ab + bc + ca)

BĐT được chứng minh. 

Ví dụ 26 (T11/421).

Với x, y, z là độ dài ba cạnh của tam giác. Gọi S, p lần lượt là diện tích và nửa chu vi của
tam giác đó. Chứng minh rằng:
1 1 1 9
2 + 2 + 2 ≥
x2 (p − x) y 2 (p − y) z 2 (p − z) 4S 2

LỜI GIẢI. Sử dụng phép biến đổi Ravi tồn tại a, b, c > 0 sao cho x = a + b, y = b + c, z =
c + a. Khi đó BĐT trở thành:
1 1 1 9
2
+ 2
+ 2

(ac + bc) (ba + ca) (cb + ab) 4abc (a + b + c)

Quan sát kỹ thì đây là bài toán khá nỗi tiếng Iran Olympiads 1996. 
Thường thì ta có xu hướng chuyển một BĐT hình học về BĐT đại số vì với BĐT đại
số có nhiều công cụ xử lý hơn. Tuy nhiên, nếu làm ngược lại phép đổi biến Ravi có phát
huy được tác dụng hay không? Mời bạn đọc đến với ví dụ tiếp theo.

Ví dụ 27 (Leonard Giugiuc).

Với ba số thực dương a, b, c. Chứng minh rằng:


r r r r
a+b b+c c+a (a + b) (b + c) (c + a)
+ + ≥3
c a b 4abc

LỜI GIẢI. Vì a, b, c là các số thực dương nên nếu ta đặt x = a + b, y = b + c, z = c + a


thì x, y, z là độ dài ba cạnh của một tam giác. BĐT trở thành:
r r r r
x y z R
+ + ≤3
p−x p−y p−z r

  v
1 A B C u 1
⇔r sin + sin + sin ≤ 3u
A B C 2 2 2 t A B C
sin . sin . sin 4 sin . sin . sin
2 2 2 2 2 2

A B C 3
⇔ sin + sin + sin ≤
2 2 2 2
BĐT cuối cùng khá quen thuộc có nhiều cách giải mà đơn giản nhất là sử dụng tính
chất của tam thức bậc 2:
A B C 3 A+B A−B A+B 1
sin + sin + sin ≤ ⇔ −2sin2 + 2 cos sin − ≤0
2 2 2 2 4 2 4 2

2. Một số kỹ thuật, BĐT phụ đặc biệt trong chứng minh BĐT GV: Nguyễn Tuấn Anh
CHƯƠNG 2. BỔ SUNG KIẾN THỨC Trang 89

Vế trái của BĐT cuối thu được là một tam thức bậc hai có biệt thức là không dương
nên bài toán được chứng minh. 
Cuối cùng, để thuận lợi trong việc sử dụng phép biến đổi Ravi ta chú ý một số biến đổi
sau: Với x = BC, y = CA, z = AB là độ dài ba cạnh của ∆ABC khi đó tồn tại a, b, c > 0
sao cho x = a + b, y = b + c, z = c + a thì:
x+z−y y+x−z z+y−x
• a= ;b = ;c =
2 2 2
• p là nửa chu vi của tam giác: p = a + b + c.
p p
• Diện tích của tam giác: S = p (p − x) (p − y) (p − z) = abc (a + b + c)
p
S abc (a + b + c)
• Bán kính đường tròn nội tiếp r của tam giác: r = =
p a+b+c
xyz (a + b) (b + c) (c + a)
• Bán kính đường tròn ngoại tiếp R của tam giác: R = = p
4S 4 abc (a + b + c)
r r
A (p − y) (p − z) ab
• sin = =
2 yz (b + c) (c + a)
r r
A p (p − x) c (a + b + c)
• cos = =
2 yz (b + c) (c + a)
r r
A (p − y) (p − z) ab
• tan = =
2 p (p − x) c (a + b + c)
b. Một số phép đổi biến nâng cao

Đổi biến với giả thiết x + y + z + 2 = xyz, (x, y, z > 0)

Với x, y, z thỏa điều kiện trên ta được:


1 1 1
+ + =1
x+1 y+1 z+1
1 1 1
Nên nếu ta đặt a = ,b = ,c = thì a + b + c = 1 và ta được:
x+1 y+1 z+1
b+c c+a a+b
x= ,y = ,z =
a b c
Vậy với x, y, z thỏa điều kiện x + y + z + 2 = xyz thì tồn tại a, b, c > 0 sao cho:
b+c c+a a+b
x= ,y = ,z =
a b c
1 1 1 1
Hệ quả: xy + yz + zx + 2xyz = 1, (x, y, z > 0) Ta có: + + + 2 = nên theo phép
x y z xyz
đổi biến trên thì tồn tại a, b, c > 0 sao cho:
a b c
x= ,y = ,z =
b+c c+a a+b

GV: Nguyễn Tuấn Anh 2. Một số kỹ thuật, BĐT phụ đặc biệt trong chứng minh BĐT
Trang 90 CHƯƠNG 2. BỔ SUNG KIẾN THỨC

Ví dụ 28 (Olympic KHTN năm 2016 ngày 2).

Với ba số thực dương x, y, z thỏa mãn xy + yz + zx + 2xyz = 1. Chứng minh rằng:

x (x + 1) y (y + 1) z (z + 1) 9
2 + 2 + 2 ≤
(2x + 1) (2y + 1) (2z + 1) 16

LỜI GIẢI. Nếu người làm chưa biết đến phép đổi biến này thì thật khó để liên kết
được giả thiết cho lời giải bài toán. Từ giả thiết tồn tại a, b, c > 0 sao cho
a b c
x= ,y = ,z =
b+c c+a a+b
BĐT cần chứng minh là:
a (a + b + c) b (a + b + c) c (a + b + c) 9
2
+ 2
+ 2

(2a + b + c) (a + 2b + c) (a + b + 2c) 16

Ta chuẩn hóa a + b + c = 3 và sử dụng đánh giá:


a 1 1
2
≤ (a − 1) +
(3 + a) 32 16

Bài toán được giải quyết. 


Tiếp tục ta đến với hai điều kiện khá đặc biệt mà nó có nhiều cách đổi biến.

Đổi biến với giả thiết x2 + y 2 + z 2 + 2xyz = 1, (x, y, z > 0)

Trước tiên là ý tưởng nhìn nhận giả thiết này như hệ quả của giả thiết đầu mà ta đã
có cách đổi biến. Ta có:
x y z 1
x2 + y 2 + z 2 + 2xyz = 1 ⇔ + + +2=
yz zx xy xyz

Theo cách đổi biến đầu tiên thì tồn tại a, b, c > 0 sao cho:
x b+c y c+a z a+b
= , = , =
yz a xz b xy c

r r r
bc ca ab
⇔x= ,y = ,z =
(a + b) (a + c) (b + c) (b + a) (c + a) (c + b)

Tiếp theo là ý tưởng đổi biến về lượng giác. Với x, y, z thỏa điều kiện thì tồn tại 0 <
π
A, B < sao cho: x = cos A, y = cos B . Thay vào điều kiện và ta giải phương trình bậc 2
2
theo z ta được:
π
 
z = cos C; 0 < C < ; A + B + C = π
2
Hay nói cách khác là tồn tại A, B, C là ba góc của một tam giác nhọn sao cho:

x = cos A, y = cos B, z = cos C

2. Một số kỹ thuật, BĐT phụ đặc biệt trong chứng minh BĐT GV: Nguyễn Tuấn Anh
CHƯƠNG 2. BỔ SUNG KIẾN THỨC Trang 91

Ý tưởng của phép đổi biến trên dựa vào đẳng thức:

cos2 A + cos2 B + cos2 C + 2 cos A cos B cos C = 1

nó cho phép ta khai thác triệt để giả thiết nhưng có nhược điểm là liên quan đến hình học
nên hạn chế một số đánh giá nhất định. Rất tự nhiên, ta suy nghĩ đến phép thế Ravi, giả
sử k, m, n là ba cạnh của tam giác khi đó. Vì tam giác là tam giác nhọn nên k 2 , m2 , n2 cũng
là ba cạnh của một tam giác, nên tồn tại a, b, c > 0 sao cho k 2 = a+b, m2 = b+c, n2 = c+a
và ta được:
k 2 + n2 − m2 a
x = cos A = =p
2kn (a + b) (a + c)
b c
Tương tự ta được: y = p ,z = p
(b + c) (b + a) (c + a) (c + b)
Hệ quả: x2 + y2 + z2 + xyz = 4, (x, y, z > 0) Ta có:
 x 2  y 2  z 2 x y  z 
2 2 2
x + y + z + xyz = 4 ⇔ + + +2 =1
2 2 2 2 2 2
Do vậy, tồn tại a, b, c > 0 sao cho:
r r r
bc ca ab
x=2 ,y = 2 ,z = 2
(a + b) (a + c) (b + c) (b + a) (c + a) (c + b)

Theo phép đổi biến ý tưởng lượng giác thì tồn tại A, B, C là ba góc của một tam giác
nhọn sao cho:
x = 2 cos A, y = 2 cos B, z = 2 cos C

Sử dụng phép thế Ravi, tồn tại a, b, c > 0 sao cho:


2a 2b 2c
x= p ,y = p ,z = p
(a + b) (a + c) (b + c) (b + a) (c + a) (c + b)

Trước khi bước vào các ví dụ bạn đọc hãy giải lại ví dụ 2 bằng cách đổi biến trên để
thấy được công dụng của nó.

Ví dụ 29 (USA MO 2001).

Với ba số thực không âm x, y, z thỏa mãn x2 + y 2 + z 2 + xyz = 4. Chứng minh rằng:

0 ≤ xy + yz + zx − xyz ≤ 2

LỜI GIẢI. BĐT vế trái khá đơn giản ta sẽ giải quyết BĐT vế phải bằng phép đổi biến
và đương nhiên ta chỉ cần thực hiện chứng minh với x, y, z > 0.
Hướng 1: Đổi biến theo hướng đại số. Từ giả thiết tồn tại a, b, c > 0 sao cho:
2a 2b 2c
x= p ,y = p ,z = p
(a + b) (a + c) (b + c) (b + a) (c + a) (c + b)

GV: Nguyễn Tuấn Anh 2. Một số kỹ thuật, BĐT phụ đặc biệt trong chứng minh BĐT
Trang 92 CHƯƠNG 2. BỔ SUNG KIẾN THỨC

BĐT cần chứng minh khi đó là:


p p p
2ab (b + c) (a + c) + 2bc (c + a) (b + a) + 2ca (a + b) (c + b) − 4abc ≤ (a + b) (b + c) (c + a)

Trái ngược với vẻ “cồng kềnh” bên ngoài, BĐT đơn giản được chứng minh bằng đánh
giá sau:
p p
2ab (b + c) (a + c) = 2 ab (b + c) ab (a + c) ≤ ab (b + c) + ab (a + c)

tương tự cho ba số hạng còn lại và cộng theo vế ta được điều phải chứng minh.
Hướng 2: Đổi biến lượng giác. Theo chứng minh trên tồn tại A, B, C là ba góc của một
tam giác nhọn sao cho:

x = 2 cos A, y = 2 cos B, z = 2 cos C

Và BĐT cần chứng minh là:

4 cos A cos B + 4 cos B cos C + 4 cos C cos A − 8 cos A cos B cos C ≤ 2

Ta có:

4 cos A cos B = 2 sin 2A. cot A. sin 2B. cot B ≤ sin 2A. cot B + sin 2B. cot A

tương tự cho ba số hạng còn lại và ta được:

4 cos A cos B + 4 cos B cos C + 4 cos C cos A

≤ cot A (sin 2C + sin 2B) + cot B (sin 2A + sin 2C) + cot C (sin 2B + sin 2A)

= 2 cot A. sin (B + C) cos (B − C) + 2 cot B. sin (C + A) cos (C − A)


+2 cot C. sin (A + B) cos (A − B)

= −2 cos (B + C) cos (B − C) − 2 cos (C + A) cos (C − A) − 2 cos (A + B) cos (A − B)

= −2 cos (2A) − 2 cos (2B) − 2 cos (2C)


= 6 − 4 cos2 A + cos2 B + cos2 C = 6 − (4 − 8 cos A cos B cos C) = 2 + 8 cos A cos B cos C

Do vậy 4 cos A cos B + 4 cos B cos C + 4 cos C cos A − 8 cos A cos B cos C ≤ 2 
Nhận xét: Bài giải trên ngoài mục tiêu thể hiện công dụng của phép đổi biến dựa vào
điều kiện, nó còn góp một phần nào đó cho lời giải thích lý do vì sao với một BĐT liên
quan yếu tố hình học người làm luôn tìm cách chuyển nó về một bài BĐT đại số.

2. Một số kỹ thuật, BĐT phụ đặc biệt trong chứng minh BĐT GV: Nguyễn Tuấn Anh
CHƯƠNG 2. BỔ SUNG KIẾN THỨC Trang 93

Đổi biến với giả thiết xy + yz + zx = 1, (x, y, z > 0)


A B
Với x, y, z thỏa điều kiện trên thì tồn tại 0 < A, B < π sao cho x = tan , y = tan .
2 2
Thay vào điều kiện ta được:
1 − xy C
z= = tan ; (0 < C < π; A + B + C = π)
x+y 2

Hay nói cách khác là tồn tại A, B, C là ba góc của một tam giác sao cho
A B C
x = tan , y = tan , z = tan
2 2 2

Cách đặt trên hơi thiếu tự nhiên, thật ra ý tưởng của nó xuất phát từ đẳng thức:
A B B C C A
tan tan + tan tan + tan tan = 1
2 2 2 2 2 2

Sử dụng phép biến đổi Ravi thì từ điều kiện sẽ tồn tại a, b, c > 0 sao cho:
r r r
ab bc ca
x= ,y = ,z =
c (a + b + c) a (a + b + c) b (a + b + c)

Ví dụ 30.

Với ba số thực không âm x, y, z thỏa mãn xy + yz + zx = 1. Chứng minh rằng:



x y z 3 3
+ + ≤
1 + x2 1 + y 2 1 + z 2 4

LỜI GIẢI. Ta sẽ giải quyết bài toán theo hai hướng đổi biến như trên.
Hướng 1: Đổi biến lượng giác. Từ giả thiết tồn tại A, B, C là ba góc của tam giác sao
cho:
A B C
x = tan , y = tan , z = tan
2 2 2
Và điều ta cần chứng minh bấy giờ là:

3 3
sin A + sin B + sin C ≤
2

BĐT này có thể chứng minh đơn giản như sau:


π π
π + A B+C +A+B+C √
sin + sin A + sin B + sin C ≤ 2 sin 3 + 2 sin ≤ 4 sin 3 =2 3
3 2 2 4

Bài toán được chứng minh.


Hướng 2: Đổi biến theo hướng đại số. Từ giả thiết tồn tại a, b, c > 0 sao cho:
r r r
ab bc ca
x= ,y = ,z =
c (a + b + c) a (a + b + c) b (a + b + c)

GV: Nguyễn Tuấn Anh 2. Một số kỹ thuật, BĐT phụ đặc biệt trong chứng minh BĐT
Trang 94 CHƯƠNG 2. BỔ SUNG KIẾN THỨC

Và ta cần chứng minh:


  √
p 1 1 1 3 3
abc (a + b + c) + + ≤
(a + b) (a + c) (b + c) (b + a) (c + a) (c + b) 4

  √
p 2 (a + b + c) 3 3
⇔ abc (a + b + c) ≤
(a + b) (b + c) (c + a) 4

Để đơn giản ta chuẩn hóa a + b + c = 3 và ta cần có:



8 abc ≤ (a + b) (b + c) (c + a)

BĐT này đúng theo BĐT AM – GM, bài toán được chứng minh. 

2.2 Một số BĐT phụ thường dùng trong chứng minh BĐT
1 1 1
a. Bổ đề + 2 + 2 ≥ 1.6
a2 +a+1 b +b+1 c +c+1

Bổ đề (Võ Quốc Bá Cẩn - Vasile Cirtoaje)

Với ba số thực dương a, b, c có tích bằng 1 thì


1 1 1
+ 2 + 2 ≥1
a2 +a+1 b +b+1 c +c+1

LỜI GIẢI. Vì abc = 1 nên tồn tại x, y, z > 0 sao cho


yz zx xy
a= ; b = ; c =
x2 y2 z2

Khi đó BĐT trở thành:


x4 y4 z4
+ + ≥1
y 2 z 2 + yzx2 + x4 z 2 x2 + zxy 2 + y 4 x2 y 2 + xyz 2 + z 4

Theo BĐT Cauchy ta có:


x4 y4 z4
+ +
y 2 z 2 + yzx2 + x4 z 2 x2 + zxy 2 + y 4 x2 y 2 + xyz 2 + z 4

2
x2 + y 2 + z 2
≥ 4
x + y 4 + z 4 + x2 y 2 + y 2 z 2 + z 2 x2 + xyz (x + y + z)
6
Đây là bài viết "Ứng dụng một bổ đề kết hợp với đạo hàm để chứng minh BĐT" bài viết sau khi viết xong,
người viết mới phát hiện nó trùng với bài viết trong quyển "Vẻ đẹp BĐT trong các kỳ thi Olympic - Trần
Phương, Võ Quốc Bá Cẩn, Trần Quốc Anh". Nói như vậy có nghĩa trùng về mặt ý tưởng mong muốn sử dụng
bổ đề để chứng minh BĐT, còn việc vận hành bổ đề như thế nào thì đây là hai bài viết khác nhau. Bạn đọc có
thể tìm đọc bài viết trong quyển sách trên để thấy được điều đó, và quan trọng hơn hết là thấy được tính hiệu
quả của của bổ đề.

2. Một số kỹ thuật, BĐT phụ đặc biệt trong chứng minh BĐT GV: Nguyễn Tuấn Anh
CHƯƠNG 2. BỔ SUNG KIẾN THỨC Trang 95

Vậy ta chỉ cần chứng minh:


2 2 2 2
≥ x4 + y 4 + z 4 + x2 y 2 + y 2 z 2 + z 2 x2 + xyz (x + y + z)

x +y +z

⇔ x2 y 2 + y 2 z 2 + z 2 x2 ≥ xyz (x + y + z)

1 1 1
⇔ (xy − yz)2 + (yz − zx)2 + (zx − xy)2 ≥ 0
2 2 2
Đẳng thức xảy ra khi và chỉ khi a = b = c = 1. 

Bổ đề kết hợp với đạo hàm

Ví dụ 31 (Lê Hữu Điền Khuê).


Với ba số thực dương a, b, c có tích bằng 1. Chứng minh rằng:
1 1 1
2 + 2 + ≥1
3a2 + (a − 1) 2
3b + (b − 1) 3c + (c − 1)2
2

Phân tích: Quá giống về giả thiết và kết luận so với bổ đề nhưng không may ở chổ
1 1
2

3a2 + (a − 1) a2 +a+1

BĐT này không phải lúc nào cũng đúng. Vậy ta sẽ điều chỉnh lại một tí, ta sẽ đăt câu
hỏi tại sao phải là a2 + a + 1? bổ đề yêu cầu là phải có dạng a2m + am + 1 là được, vậy
thì a4 + a2 + 1 được hay a6 + a3 + 1...
Hóa ra vấn đề là ta cần có một đánh giá
1 1
2

3a2 + (a − 1) a2m + am + 1

đúng với mọi a dương, đồng nghĩa để chứng minh bài toán ta chỉ việc tìm m. Ta sẽ xem
1 1
f (a) = 2

3a2 + (a − 1) a2m + am + 1

là một hàm số theo biến a và nhiệm vụ cuối cùng là tìm m sao cho hàm số đạt GTNN
là 0 khi a = 1.
Vẫn còn quá phức tạp, ta đang mò mẫm để đi chứ không phải giải một bài hẳn hoi nên
chưa cần đảm bảo cần và đủ. Ta sẽ giảm nhẹ lại, cực trị tại a = 1 thì sao? (vì có một
tham số cần 1 phương trình là giải được) hy vọng đó sẽ là GTNN.

−8a + 2 2ma2m−1 + mam−1


f 0 (a) = +
2 2 (a2m + am + 1)
2
3a2 + (a − 1)

−2 m
⇒ f 0 (1) = 0 ⇔ + =0⇔m=2
3 3

GV: Nguyễn Tuấn Anh 2. Một số kỹ thuật, BĐT phụ đặc biệt trong chứng minh BĐT
Trang 96 CHƯƠNG 2. BỔ SUNG KIẾN THỨC

Vậy ta chỉ cần chứng tỏ


1 1

3a2 + (a − 1)2 a4 + a2 + 1

Thật vậy, BĐT trên tương đương:

a(a − 1)2 (a + 2) ≥ 0

LỜI GIẢI. Ta có đánh giá sau (tương tự cho b, c):


1 1
2

3a2 + (a − 1) a4 + a2 + 1

Vì vậy:
1 1 1
2
+ 2
+
3a2 + (a − 1) 3b2 + (b − 1) 3c2 + (c − 1)2

1 1 1
≥ + 4 + 4
a4 + a + 1 b + b + 1 c + c2 + 1
2 2

theo bổ đề trên ta được điều phải chứng minh. Đẳng thức xảy ra khi và chỉ khi a = b =
c = 1. 
Nhận xét:

• Tìm ra tham số m để có đánh giá trên chỉ là điều kiện cần chứ chưa phải là đủ,
vậy nên cần kiểm tra lại mà đơn giản nhất là biến đổi tương đương.

• Cách tìm m bằng đạo hàm thật sự không mới, tuy nhiên sự kết hợp của bổ đề và
phương pháp này thật sự sẽ cho một cách nhìn mới hoàn toàn.

Một số ví dụ áp dụng

Ví dụ 32 (Đề thi chọn đội tuyển Anh dự thi IMO 2005).

Với ba số thực dương a, b, c có tích bằng 1. Chứng minh rằng:


a+3 b+3 c+3
2 + 2 + ≥3
(a + 1) (b + 1) (c + 1)2

Phân tích: Ta sẽ tìm m để có đánh giá

a+3 1
2

(a + 1) a2m + am + 1

3
Thực hiện như trên ta được m =
4

2. Một số kỹ thuật, BĐT phụ đặc biệt trong chứng minh BĐT GV: Nguyễn Tuấn Anh
CHƯƠNG 2. BỔ SUNG KIẾN THỨC Trang 97

LỜI GIẢI. Ta có:


a+3 3

(a + 1) 2 3 3
a2 + a4 + 1

x3 .(x − 1)2 . x5 + 2x4 − x2 + x + 3



⇔ 2
≥0
(x4 + 1) (x6 + x3 + 1)
1
với x = a 4 , BĐT hiển nhiên đúng. Vậy nên:
a+3 b+3 c+3
2
+ 2
+
(a + 1) (b + 1) (c + 1)2

3 3 3
≥ + +
3 3 3 3 3 3
a2 + a4 + 1 b2 + b4 + 1 c2 + c4 + 1
Theo bổ đề trên ta được điều phải chứng minh.
Đẳng thức xảy ra khi và chỉ a = b = c = 1. 

Ví dụ 33 (Đề thi chọn đội tuyển Việt Nam dự thi IMO 2005).

Với ba số thực dương a, b, c có tích bằng 1. Chứng minh rằng:


1 1 1 3
3 + 3 + 3 ≥
(a + 1) (b + 1) (c + 1) 8

Phân tích: Ta tìm m để có đánh giá:


8 3
3

(a + 1) a2m + am + 1
3
và tìm được m = .
2
LỜI GIẢI. Ta có:
8 3

(a + 1)3 3
a3 + a 2 + 1

(x − 1)2 5x4 + 10x3 + 6a2 + 10a + 5



⇔ 3
(x2 + 1) (x6 + x3 + 1)

với x = a.
Đánh giá tương tự cho b, c kết hợp với bổ đề ta được điều phải chứng minh. Đẳng thức
xảy ra khi và chỉ khi a = b = c = 1. 
Kết thúc ở đây hoặc tiếp tục đưa ra hàng loạt ví dụ tương tự chắc chắn sẽ làm bạn đọc
cảm thấy đơn điệu vì chưa thấy hiệu quả. Tiếp đến ta sẽ đến với một số bài toán mà
dạng ban đầu khó mà nghĩ đến bổ đề.

GV: Nguyễn Tuấn Anh 2. Một số kỹ thuật, BĐT phụ đặc biệt trong chứng minh BĐT
Trang 98 CHƯƠNG 2. BỔ SUNG KIẾN THỨC

Ví dụ 34 (Vasile Cirtoaje - Discrete Inequalities Vol 3).

Với ba số thực dương a, b, c có tích bằng 1. Chứng minh rằng:


1 1 1

3
+√
3 2
+ √
3 2
≥1
a2 + 25a + 1 b + 25b + 1 c + 25c + 1

Phân tích: Ta vẫn chọn kiểu đánh giá:

1 1
√ ≥
3
a2 + 25a + 1 a2m + am + 1

2
và chọn được m = . Lý do vì sao ta mạnh dạn chọn cách đánh giá này xin được giải
3
thích sau.
LỜI GIẢI. Ta có:
1 1
√ ≥
3
a2 + 25a + 1 2 1
a3 + a3 + 1

√ √ 2 √ √
 
3
3 a( a − 1)
3 3 2
a +4 a+1
3

⇔ √ 3 ≥0
3 √
a2 + 3
a+1 (a2 + 35a + 1)

Đánh giá tương tự cho b, c kết hợp với bổ đề ta được điều phải chứng minh. Đẳng thức
xảy ra khi và chỉ khi a = b = c = 1. 

Ví dụ 35 (MEMO, 2012).

Với ba số thực dương a, b, c có tích bằng 1. Chứng minh rằng:


√ √ √
16a2 + 9 + 16b2 + 9 + 16c2 + 9 ≥ 4 (a + b + c) + 3

Phân tích: Ta tìm m để có đánh giá:

p 3
16a2 + 9 − 4a ≥
a2m + am + 1

4
như trên ta tìm được m = . Vấn đề còn lại là biến đổi tương đương để chứng minh
5
đánh giá vừa chọn là đúng.

2. Một số kỹ thuật, BĐT phụ đặc biệt trong chứng minh BĐT GV: Nguyễn Tuấn Anh
CHƯƠNG 2. BỔ SUNG KIẾN THỨC Trang 99

LỜI GIẢI. Để thuận lợi ta đặt a = x5 ta có:


p 3
16a2 + 9 − 4a ≥
8 4
a5 + a5 + 1

9 3
⇔√ ≥
16x10 + 9 + 4x5 x8 + x4 + 1

2
⇔ 3x8 + 3x4 + 3 − 4x5 ≥ 16x10 + 9

⇔ 3x4 (x − 1)2 3x10 + 6x9 + 9x8 + 4c7 + 5x6 + 6x5 + 7x4 + 2x2 + 4x + 6 ≥ 0


Vậy đánh giá trên luôn đúng tức:


p p p
16a2 + 9 − 4a + 16b2 + 9 − 4b + 16c2 + 9 − 4c

3 3 3
≥ + +
8 4 8 4 8 4
a5 + a5 + 1 a5 + a5 + 1 a5 + a5 + 1

Áp dụng bổ đề trên ta được điều phải chứng minh. Đẳng thức xảy ra khi và chỉ khi
a = b = c = 1.
Nhận xét: Bước chứng minh đánh giá tương đối nặng về kỹ năng dù không khó lắm.
Thầy giáo Võ Quốc Bá Cẩn đã khắc phục nó bằng cách thực hiện 2 bước đánh giá phụ
có thể tóm tắt như sau:
p 2a + 3 3
16a2 + 9 − 4a ≥ ≥
2a2 + 2a + 1 8 4
a5 + a5 + 1

Tiếp đến ta bắt đầu với các ví dụ mà giả thiết khác đi dần dần.

Ví dụ 36 (Azerbaijan NMO 2015).


1
Với ba số thực dương a, b, c thỏa mãn abc = . Chứng minh rằng:
8
15
a2 + b 2 + c 2 + a2 b 2 + b 2 c 2 + c 2 a2 ≥
16

Phân tích: Để có thể vận dụng bổ đề ta thay đổi giả thiết lại bằng cách đặt
a b c
x = ;y = ;z =
2 2 2
Khi đó xyz = 1 và BĐT được viết lại là:
 
2 2 2
 1 1 1
4 x +y +z + 2
+ 2+ 2 ≥ 15
x y z

GV: Nguyễn Tuấn Anh 2. Một số kỹ thuật, BĐT phụ đặc biệt trong chứng minh BĐT
Trang 100 CHƯƠNG 2. BỔ SUNG KIẾN THỨC

đến đây ý tưởng bài toán đã khá rõ ràng là sử dụng BĐT AM - GM, nếu vậy bài giải
chỉ dừng lại ở đó. Ta sẽ tìm một lời giải dựa trên bổ đề trên nếu làm được chắc chắn
đấy là một lời giải thú vị đẹp mắt, tức ta cần tìm m cho đánh giá:
1 15
4x2 + 2
≥ 2m
x x + xm + 1
−6
và nhận được m = .
5
LỜI GIẢI.
Ta có:
1 15
4x2 + ≥
x2 −12 −6
x 5 +x 5 +1
thật vậy,
1 15
4x2 + ≥
x2 −12 −6
x 5 +x 5 +1

1 15t12
⇔ 4t10 + ≥
t10 1 + t6 + t12

⇔ 4t20 + 1 1 + t6 + t12 ≥ 15t22


 

Với t5 = a, BĐT cuối đúng theo AM - GM.


Đánh giá tương tự cho các biến b, c kết hợp với bổ đề ta được điều phải chứng minh.
1
Đẳng thức xảy ra khi và chỉ a = b = c = . 
2
Nhận xét: Qua bài giải ta thấy bổ đề có thể chứng minh được cả BĐT cổ điển AM -
GM (đương nhiên đang nói là ba biến).

Ví dụ 37 (AM - GM 3 biến).

Với ba số thực dương a, b, c. Chứng minh rằng:


a+b+c √
3
≥ abc
3

Phân tích: Trước hết ta chuẩn hóa abc = 1 (để có thể vận dụng bổ đề) khi đó BĐT
cần chứng minh là:
a+b+c≥3

như trên ta sẽ tìm một đánh giá


3
a≥
a2m + am + 1

và tìm được m = −1. Việc hoàn thành bài giải dành lại cho bạn đọc.

2. Một số kỹ thuật, BĐT phụ đặc biệt trong chứng minh BĐT GV: Nguyễn Tuấn Anh
CHƯƠNG 2. BỔ SUNG KIẾN THỨC Trang 101

Nhận xét: Có vẻ như ta đã đem công cụ quá mạnh để xử lý một bài toán đơn giản,
tuy nhiên ví dụ này là mấu chốt của vấn đề để giải thích lý do tại sao ta mạnh dạn
chọn Bổ đề làm cấu hình để đánh giá:
• Bổ đề được chứng minh bằng BĐT Cauchy dạng phân thức là hệ quả của BĐT
Cauchy, mà BĐT Cauchy thì xuất phát từ đẳng thức Lagrange nên ta không hề
trùng lấp khi dùng Bổ đề để chứng minh cho AM - GM ba biến như trên.

• Ta phải thừa nhận BĐT AM - GM là BĐT cổ điển được sử dụng nhiều (chính
xác là rất nhiều) trong chứng minh BĐT, ấy vậy mà bộ đôi trên lại có thể chứng
minh được AM - GM. Điều đó cho ta lòng tin bộ đôi này có thể giải quyết được
rất nhiều dạng toán chứ không phải hạn chế ở những dạng giống Bổ đề.
Câu hỏi đặt ra là một bài ngẫu nhiên có áp dụng bộ đôi này được không? Câu trả lời
thật khó nói nó tùy thuộc vào mỗi người giải để thấy điều đó ta sẽ vào các bài toán sau:

Khai thác thêm bộ đôi phương pháp

Ví dụ 38 (Vasile Cı̂rtoaje, 2012).


Với ba số thực dương a, b, c có tích bằng 1. Chứng minh rằng:
√ √ √
25a2 + 144 + 25b2 + 144 + 25c2 + 144 ≤ 5 (a + b + c) + 24

Phân tích: Ta sẽ tìm m cho đánh giá


p 3
5a + 9 − 25a2 + 144 ≥
a2m + am + 1
nhưng thật không may là không có m nào thỏa mãn lý do là khi a tiến về 0 thì vế phải
là một số âm. Gấp giấy lại và kết luận bộ đôi phương pháp và bổ đề không dùng được
nữa rồi, nếu bạn đọc suy nghĩ vậy thì thật là tiếc. Ta để ý một chút, lý do làm vế phải
âm là do số 144 trong căn vì vậy số 9 đằng trước không đủ để trị! Vậy tại sao phải là 9
thay số khác được không? bao nhiêu là đủ?
Với phân tích vậy ta tiến hành giải luôn, chắc chắn bạn đọc sẽ chọn số 12 thay cho số 9
như sau:
p 12
5a + 12 − 25a2 + 144 ≥
a2m + am + 1
−10
và tìm được m = . Việc còn lại là chứng minh đánh giá như sau:
13
p 12
5a + 12 − 25a2 + 144 ≥
−20 −10

20 10 7
2a 13 14+ a 13
26
+1 
⇔ 5t + 10t − 12t + 10 ≥ t 25t + 144
⇔ 20 5t30 − 6t27 + 10t20 − 12t17 + 10t10 − 12t7 + 5 ≥ 0


GV: Nguyễn Tuấn Anh 2. Một số kỹ thuật, BĐT phụ đặc biệt trong chứng minh BĐT
Trang 102 CHƯƠNG 2. BỔ SUNG KIẾN THỨC

với t13 = a. Có thể dùng hàm số để chứng BĐT trên nhưng có vẻ phức tạp, ở đây ta sẽ
dùng AM - GM với cách ghép cặp cũng khá thú vị:

30 12 27
 5t + t ≥ 6t


9t20 + 3t8 ≥ 12t17

 8t10 + 4t ≥ 12t7

Vậy ta chỉ cần chứng minh:

t20 − t12 + 2t10 − 3t8 − 4t + 5 ≥ 0


⇔ 6t20 + 12t10 + 30 ≥ 6t12 + 18t8 + 24t

ta lại ghép cặp dùng AM - GM:



20 12
 6t + 4 ≥ 10t


12t10 + 2t12 + 4 ≥ 18t8

 2t12 + 22 ≥ 24t

Vậy đánh giá trên được chứng minh. Hoàn thành bài giải dành cho bạn đọc.
Nhận xét: Chắc chắn bạn đọc chưa thỏa mãn với bài giải trên, đúng vậy ví dụ này
được chọn không phải để phô diễn kỹ thuật tính toán. Một câu hỏi đặt ra là với bài
toán khác thì số thêm vào là bao nhiêu? Chắc chắn một điều trực giác sẽ không đứng
về phía chúng ta nữa.
Ta sẽ điều chỉnh cách đánh giá một tí như là một cách để lý giải cho con số 12
p 3 + 3k
5a + 9 + k − 25a2 + 144 ≥
a2m+ am + 1

Để đảm bảo vế phải không âm ta phải chọn k ≥ 3, khi đó áp dụng phương pháp tìm m
như trên ta được:
−40 −10
−1=k ≥3⇔ ≤m<0
13m 13
−10
Vậy bài giải trên may mắn chọn m = tương ứng với k = 3. Qua đó nếu ta chọn k
13
là số khác lớn hơn 3 vẫn có được một đánh giá tuy nhiên việc kiểm chứng đánh giá có
đúng hoàn toàn hay không, khá là khó vì m là số khá xấu. Hơn nữa k chỉ nên chọn gần
3 (biên của điều kiện) nếu xa ra chắc chắn đánh giá đó sẽ sai.
Để thấy được việc chọn k không được máy móc như thế nào ta cần một ví dụ nữa nhưng
trước hết ta xét ví dụ sau:

2. Một số kỹ thuật, BĐT phụ đặc biệt trong chứng minh BĐT GV: Nguyễn Tuấn Anh
CHƯƠNG 2. BỔ SUNG KIẾN THỨC Trang 103

Ví dụ 39 (Bosnia and Herzegovina TST 2015 day 1 problem 1).

Với ba số thực dương a, b, c thỏa mãn a + b + c ≤ 3. Tìm giá trị nhỏ nhất của
a+1 b+1 c+1
+ +
a (a + 2) b (b + 2) c (c + 2)

Phân tích: Dễ dàng dự đoán GTNN là 2 và dễ nhất là sử dụng phương pháp tiếp tuyến
để làm. Vậy là tốt khi thi cử thế nhưng ta đang tìm tòi với bộ đôi phương pháp và bổ
đề nên ta thử tìm một lời giải thú vị dựa trên chúng.
Ta sẽ tìm m cho đánh giá:
a+1 2
≥ 2m
a (a + 2) a + am + 1
5
và tìm được m = . Tuy nhiên ta phải khóe léo một tí mới dùng được bổ đề vì giả thiết
6
được suy ra là abc ≤ 1. Vậy nên, với a, b, c thỏa giả thiết ta chọn một số a0 ≥ a sao cho
a0 bc = 1 khi đó:

a+1 b+1 c+1 a0 + 1 b+1 c+1


+ + ≥ 0 0 + +
a (a + 2) b (b + 2) c (c + 2) a (a + 2) b (b + 2) c (c + 2)

Kết hợp đánh giá trên cho ba biến a0 , b, c ta hoàn thành bài toán.
Nhận xét: Ta được một kết quả chặt hơn làm khó được phương pháp tiếp tuyến là:
Với ba số thực dương a, b, c có tích bằng 1. Chứng minh rằng
a+1 b+1 c+1
+ + ≥2
a (a + 2) b (b + 2) c (c + 2)

Quay lại việc chọn k ta xét ví dụ sau:

Ví dụ 40 (Tuymaada Olympiad 2014).

1 1 1
Với ba số thực dương a, b, c thỏa mãn + + = 3. Chứng minh rằng:
a b c
1 1 1 3
√ +√ +√ ≤√
a3 +1 b3+1 3
c +1 2

LỜI GIẢI. Ta có: r


3 2 3
− ≥
a3
 
2 +1 3

2 a−3 + a 2 + 1

√ 2  √ 
⇔ a3 − 1 a3 + 4 a3 + 1 ≥ 0

GV: Nguyễn Tuấn Anh 2. Một số kỹ thuật, BĐT phụ đặc biệt trong chứng minh BĐT
Trang 104 CHƯƠNG 2. BỔ SUNG KIẾN THỨC

Vậy với abc ≥ 1 thỏa giả thiết ta luôn chọn được a0 ≤ a sao cho a0 bc = 1 khi đó:
r r r r
3 2 3 2 3 2 3 2
− + − + − + −
2 a3 +1 2 b3 +1 2 b3 +1 2 c3 +1

r r r
3 2 3 2 3 2
≥ − 03
+ − 3
+ −
2 a +1 2 b +1 2 c3 +1

3 3 3 3
≥ +  +  ≥
−3

3 3 2
0 − −
2 a0−3 + a 2 + 1 2 b−3 + b 2 + 1 2 c−3 + c 2 + 1


Nhận xét:
1 1 1
• Bài toán có thể làm mạnh hơn bằng cách thay giả thiết + + = 3 bằng abc = 1.
a b c
• Để có được đánh giá như trên ta đã kết hợp cả 2 kỹ năng của 2 ví dụ phân tích
rất kỹ bên trên tức chọn đồng thời k, m cho đánh giá:
r
2 3k
1+k− ≥
a3 + 1 a2m + am + 1
√ 1 3
với k ≥ 2 − 1 và để đơn giản ta đã chọn k = tương ứng m = − . Bạn đọc hãy
2 2
thử tìm với k = 1 sẽ thấy không có m mặc dù k thỏa mãn điều kiện, điều đó chứng
tỏ chọn k phải gần biên của điều kiện mới có hy vọng đánh giá chặt.

Tiếp tục ta sẽ điểm qua một số bài toán để thấy tính hiệu quả của Bộ đôi trên:

Các ví dụ nâng cao

Ví dụ 41 (JBMO TST - Macedonia 2013).

Với ba số thực dương a, b, c thỏa mãn abc = 1. Chứng minh rằng:


1 √ √ √  1 1 1
a + b + c + + + ≥ 3
2 1+a 1+b 1+c
1√ 1 3
HƯỚNG DẪN. Sử dụng đánh giá: a+ ≥ 0 =1
2 1+a a + a0 + 1
(do đó giả thiết abc = 1 là thừa chưa dùng.) 

Ví dụ 42 (Zdravko Cvetkovski).

Với ba số thực dương a, b, c thỏa mãn abc = 1. Chứng minh rằng:


a b c
+ + ≤1
a2 + 2 b 2 + 2 c 2 + 2

2. Một số kỹ thuật, BĐT phụ đặc biệt trong chứng minh BĐT GV: Nguyễn Tuấn Anh
CHƯƠNG 2. BỔ SUNG KIẾN THỨC Trang 105

1 a 1 1
HƯỚNG DẪN. Sử dụng đánh giá: − 2 + ≥ 2

3 a +2 9 3 (a + a + 1)
Mở rộng:
Với k = 1, 2, 3, 4, 5 và a, b, c là các số thực dương có tích bằng 1 thì BĐT sau luôn đúng:
a b c 3
+ 2 + 2 ≤
a2 +k b +k c +k k+1

Ví dụ 43 (Võ Quốc Bá Cẩn).

Với ba số thực dương a, b, c thỏa mãn abc = 1. Chứng minh rằng:


 5  5  5
a b c 3
+ + ≤
a3 + 1 b3 + 1 c3 + 1 25
1 a
 5 1 3
HƯỚNG DẪN. Sử dụng đánh giá: 5 − 3 + ≥ 6 −10 
2 a +1 26 2 (a + a−5 + 1)
Ví dụ 44 (Crux Mathematicorum - Mihaly Bencze).

Với ba số thực dương a, b, c thỏa mãn abc = 1. Chứng minh rằng:


a b c 1 1 1
+ 2 + 2 ≤ + +
a2 +a+1 b +b+1 c +c+1 a+2 b+2 c+2
1 a 1 1
HƯỚNG DẪN. Sử dụng đánh giá: − 2 + ≥ 2

a+2 a +a+1 9 3 (a + a + 1)
Cuối cùng ta sẽ đến với các bài toán từ giả thiết đến dạng toán đều khác hoàn toàn với
bổ đề, thế nhưng khi ta sử bộ đôi bổ đề và phương pháp trong bài vào thì được kết quả
hoàn toàn bất ngờ (xin nhắc lại niềm tin để ta thực hiện bước đánh giá này đó là bổ đề
rất chặt và khi khi kết hợp với phương pháp trên có thể chứng minh được BĐT AM -
GM).

Ví dụ 45 (Vasile Cirtoaje, GM - B, 9 - 10, 1992).

Với ba số thực dương a, b, c. Chứng minh rằng:


r r r
2a 2b 2c
+ + ≤3
a+b b+c c+a

HƯỚNG DẪN. Ta chứng minh bài toán tương đương với giả thiết xyz = 1:
r r r
2 2 2
+ + ≤3
x+1 y+1 z+1

Sử dụng đánh giá: r


2 1 3
1− + ≥
−1
 
x+1 2
−1
2 x + x 2 + 1

GV: Nguyễn Tuấn Anh 2. Một số kỹ thuật, BĐT phụ đặc biệt trong chứng minh BĐT
Trang 106 CHƯƠNG 2. BỔ SUNG KIẾN THỨC

Ví dụ 46 (IMO - 2001, Problem 2).

Với ba số thực dương a, b, c. Chứng minh rằng:


a b c
√ +√ +√ ≥1
a2 + 8bc b2 + 8ca c2 + 8ab

HƯỚNG DẪN. Chuẩn hóa abc = 1 ta cần chứng minh:


a b c
r +r +r ≥1
8 8 8
a2 + b2 + c2 +
a b c

Sử dụng đánh giá:


a 1 1 1
− + ≥
−8 −4
r
8 3 3
a2 + a 3 +a 3 +1
a


Ví dụ 47 (Jedaihan - Trích ”BĐT và những lời giải hay”).

Với ba số thực dương a, b, c. Chứng minh rằng:


s s s
a4 + 2b2 c2 b4 + 2c2 a2 c4 + 2a2 b2
+ + ≥a+b+c
a2 + 2bc b2 + 2ca c2 + 2ab

HƯỚNG DẪN. Chuẩn hóa abc = 1 ta cần chứng minh:


r r r
a6 + 2 b6 + 2 c6 + 2
+ + ≥a+b+c
a4 + 2a b4 + 2b c4 + 2c

Sử dụng đánh giá: r


a6 + 2 3
− a + 1 ≥
a4 + 2a a2 + a + 1


Ví dụ 48 (THTT - T9/460).

Với ba số thực dương a, b, c thỏa abc ≥ 1. Chứng minh rằng:

a4 b 2 c 2 a2 b 4 c 2 a2 b 2 c 4 3
+ + ≥
bc + 1 ca + 1 ab + 1 2

HƯỚNG DẪN. Với mỗi a, b, c thỏa giả thiết ta luôn chọn được a0 ≤ a thỏa mãn a0 bc = 1.
Khi đó BĐT trở thành:
a03 b3 c3 3
0
+ + ≥
a +1 b+1 c+1 2

2. Một số kỹ thuật, BĐT phụ đặc biệt trong chứng minh BĐT GV: Nguyễn Tuấn Anh
CHƯƠNG 2. BỔ SUNG KIẾN THỨC Trang 107

Sử dụng đánh giá:


b3 1 1 3
− + ≥
−5
 
b+1 2 2
−5
2 b + b 2 + 1 

Ví dụ 49 (THTT - T8/454).

Với ba số thực dương a, b, c. Chứng minh rằng:


 2  2  2  
a b c 5 a b c
+ + +3≥ + +
a+b b+c c+a 2 a+b b+c c+a

HƯỚNG DẪN. Ta chứng minh bài toán tương đương với giả thiết xyz = 1 sau:
 1 2  2  1 2  
1 5 1 1 1
+ + +3≥ + +
x+1 y+1 z+1 2 x+1 y+1 z+1

Sử dụng đánh giá:


 1 2 5 1 3
+1− + ≥
−3 −3
 
x+1 2 (x + 1) 2
2 x 2 + x 4 + 1

Ví dụ 50 (China Northern Mathematical Olympiad 2006).

Với ba số thực dương a, b, c có tổng bằng 3. Chứng minh rằng:

a2 + 9 b2 + 9 c2 + 9
+ + ≤5
2a2 + (b + c)2 2b2 + (c + a) 2c2 + (a + b)

LỜI GIẢI. Bài toán tương đương với:


5 a2 + 9 5 b2 + 9 5 c2 + 9
− + − + − ≥0
3 2a2 + (3 − a)2 3 2b2 + (3 − b) 3 2c2 + (3 − c)

Với mỗi a, b, c thỏa giả thiết, không mất tính tổng quát ta xem a = min {a, b, c}. Khi đó
tồn tại a0 thỏa a ≤ a0 ≤ 1 sao cho a0 bc = 1. Khi đó:
5 a2 + 9 5 b2 + 9 5 c2 + 9
− + − + −
3 2a2 + (3 − a)2 3 2b2 + (3 − b)2 3 2c2 + (3 − c)2

5 a02 + 9 5 b2 + 9 5 c2 + 9
≥ − + − + −
3 2a02 + (3 − a0 )2 3 2b2 + (3 − b)2 3 2c2 + (3 − c)2

GV: Nguyễn Tuấn Anh 2. Một số kỹ thuật, BĐT phụ đặc biệt trong chứng minh BĐT
Trang 108 CHƯƠNG 2. BỔ SUNG KIẾN THỨC

Đến đây ta chỉ cần sử dụng đánh giá sau:


5 b2 + 9 1 1
− 2
+ ≥
3 2b + (3 − b)
2 21 7 (b + b7 + 1)
14

Ví dụ 51 (Đề chọn đội tuyển Tỉnh Nghệ An năm 2015 - 2016).

Với ba số thực dương a, b, c. Chứng minh rằng:


r r r
a b c a b c
+ + + + + ≥3
b+c c+a a+b 2b + 2c 2c + 2a 2a + 2b

Phân tích: Có thể tiếp cận bài toán bằng Cauchy (tương tự như chứng minh cho BĐT
Nesbitt). Ở đây ta có sẽ kết hợp với bổ đề để cho ra một lời giải mới. Trước tiên ta
chuẩn hóa a + b + c = 3 (không chuẩn hóa abc = 1 vì khi đó BĐT không tách biến). Khi
đó BĐT cần chứng minh là:
r r r
a b c a b c
+ + + + + ≥3
3−a 3−b 3−c 6 − 2a 6 − 2b 6 − 2c

Đến đây có 2 vấn đề phát sinh:

• Giả thiết a + b + c = 3 không dùng được Bổ đề.

• Nếu làm như các bài trên ta sẽ đánh giá a0 ≥ a với hy vọng a0 bc = 1 và:
r
a0 a0
r
a a
+ ≥ +
3−a 6 − 2a 3 − a0 6 − 2a0
r
a a
Tuy nhiên hàm số f (a) = + là hàm tăng vì vậy đánh giá này không
3−a 6 − 2a
dùng được.

Loay hoay mãi cuối bài viết mới nhận ra một điều rất đơn giản là Bổ đề không chỉ
đúng với abc = 1 mà đúng với cả a + b + c = 3, ab + bc + ca = 3. Vậy thì 2 khó khăn trên
và cả những khó khăn phải lách đi bằng hàm tăng giảm đều được giải quyết một cách
nhẹ nhàng.
LỜI GIẢI. Ta sử dụng đánh giá:
r
a a 3
+ ≥
3−a 6 − 2a −9 −9
a 4 +a 8 +1

Tương tự với b, c. Kết hợp với Bổ đề (đương nhiên đang nói trong trường hợp a+b+c = 3)
ta được điều phải chứng minh. 

2. Một số kỹ thuật, BĐT phụ đặc biệt trong chứng minh BĐT GV: Nguyễn Tuấn Anh
CHƯƠNG 2. BỔ SUNG KIẾN THỨC Trang 109

Bộ đôi Bổ đề trên rất hiệu quả với các bài toán tách biến với giả thiết abc là hằng số,
không những vậy với cả những bài không tách biến ta có thể chuẩn hóa để đưa về dạng
tách biến cũng có thể áp dụng được. Đặc biệt với một số bài có giả thiết tổng a + b + c
hoặc ab + bc + ca là hằng số nếu khéo léo vẫn có thể áp dụng được hiển nhiên đây không
phải là vạn năng nên sẽ có những bài không áp dụng được đơn cử như bài toán đơn
giản sau:
Romania 2005
Với ba số thực dương a, b, c có tổng bằng 3. Chứng minh rằng:
1 1 1
2
+ 2
+ 2
≥ a2 + b 2 + c 2
a b c

Không thể dùng được kỹ thuật trong bài lý do:

• Nếu làm mạnh giả thiết lên là abc = 1 thì BĐT trên không còn đúng nữa.

• Nếu dùng Bổ đề với giả thiết a + b + c = 3, 7 thì không tìm được m.

Cuối cùng mời bạn đọc lướt qua các bài toán sau để hiểu rõ hơn về tính hiệu quả của
bổ đề khi được kết hợp với công cụ đạo hàm.

Bài tập đề nghị

Bài 2.15 (Algebraic Inequalities).

Với ba số thực dương a, b, c. Chứng minh rằng:

a2 b2 c2
+ + ≥1
a2 + ab + b2 b2 + bc + c2 c2 + ca + a2

HƯỚNG DẪN. Kết quả trực tiếp của Bổ đề. 

Bài 2.16 (Vũ Đình Quý).

Với ba số thực dương a, b, c có tích bằng 1. Chứng minh rằng:


1 1 1
+ 2 + 2 ≤3
a2 −a+1 b −b+1 c −c+1
1 1 3
HƯỚNG DẪN. Sử dụng đánh giá: 1 − + ≥ 
a2 −a+1 2 2 (a + a−2 + 1)
−4

7
Xin nhắc lại khi đó Bổ đề vẫn còn đúng

GV: Nguyễn Tuấn Anh 2. Một số kỹ thuật, BĐT phụ đặc biệt trong chứng minh BĐT
Trang 110 CHƯƠNG 2. BỔ SUNG KIẾN THỨC

Bài 2.17 (Trần Quốc Luật, Võ Quốc Bá Cẩn).


Với ba số thực dương a, b, c có tích bằng 1. Chứng minh rằng:
a b c
+ 3 + 3 ≤1
2a3 + 1 2b + 1 2c + 1
1 a 1 1
HƯỚNG DẪN. Sử dụng đánh giá: − 3 + ≥ 
3 2a + 1 6 2 (a−4 + a−2 + 1)
Bài 2.18 (Discrete vol 3).
Với ba số thực dương a, b, c thỏa mãn abc = 1. Chứng minh rằng:
√ √ √
a2 − a + 1 + b2 − b + 1 + c 2 − c + 1 ≥ a + b + c
p 1 3
HƯỚNG DẪN. Sử dụng đánh giá: a2 − a + 1 − a + ≥ 2

2 2 (a + a + 1)
Bài 2.19 (Đề chọn đội tuyển Tỉnh Đăk Lăk lần 1 năm 2016 - 2017).
Với ba số thực dương a, b, c thỏa mãn abc = 1. Chứng minh rằng:
1 1 1
+ + ≤1
2+a 2+b 2+c
1 1 1 1
HƯỚNG DẪN. Sử dụng đánh giá: − + ≥ 
−4 −2
 
3 2+a 6
2 a 3 + a 3 + 1
r r r
x y x+y 8
b. Bổ đề + ≥2 .
y+z z+x x + y + 2z

Bổ đề (Võ Quốc Bá Cẩn - Trần Quốc Anh)

Với x, y, z là các số thực dương và z là số nhỏ nhất. Chứng minh rằng:


r r r
x y x+y
+ ≥2
y+z z+x x + y + 2z

LỜI GIẢI. Thật vậy, theo BĐT Holder ta có:


r r 2
x y
x2 (y + z) + y 2 (z + x) ≥ (x + y)3

+
y+z z+x

Vì z là số nhỏ nhất trong ba số nên ta được:


x2 (y + z) + y 2 (z + x) = xy (x + y − 2z) + z(x + y)2

(x + y)2 2 (x + y)2
≤ (x + y − 2z) + z(x + y) = (x + y + 2z)
4 4
8
Bạn đọc có thể tham khảo thêm "Ứng dụng một bổ đề hay trong chứng minh BĐT - Ngô Trung Hiếu, Cao
Minh Quang - THTT 469"

2. Một số kỹ thuật, BĐT phụ đặc biệt trong chứng minh BĐT GV: Nguyễn Tuấn Anh
CHƯƠNG 2. BỔ SUNG KIẾN THỨC Trang 111

Vậy nên ta có:


s s
3
4(x + y)3
r r r
x y (x + y) x+y
+ ≥ ≥ 2
=2
y+z z+x x (y + z) + y 2 (z + x)
2
(x + y) (x + y + 2z) x + y + 2z

Một số ví dụ áp dụng bổ đề

Ví dụ 52 (BĐT Nesbitt).

Với x, y, z là các số thực dương. Chứng minh rằng:


x y z 3
+ + ≥
y+z z+x x+y 2

LỜI GIẢI. Không mất tính tổng quát ta giả sử z là số nhỏ nhất trong các số x, y, z .
Khi đó:
r r r r
x y x+y x y xy 4 (x + y)
+ ≥2 ⇔ + +2 ≥
y+z z+x x + y + 2z y+z z+x (x + z) (y + z) x + y + 2z

Mà r
(x + y) xy

x + y + 2z (x + z) (y + z)

⇔ (x + y)2 (y + z) (z + x) ≥ xy(x + y + 2z)2

⇔ z 2 (x − y)2 + z (x + y) (x − y)2 ≥ 0
Do vậy 9
x y 2 (x + y)
+ ≥
y+z z+x x + y + 2z
Vậy nên ta cần chứng minh:
x + y 
2 (x + y) z 3 2 z 3
+ ≥ ⇔ x + yz + ≥
x + y + 2z x + y 2 +2 x+y 2
z
z
Để đơn giản ta đặt t = , và ta cần chứng minh:
x+y
2
t 3 (2t − 1)2
+t≥ ⇔ ≥0
1 2 2 (2t + 1)
+2
t
9
BĐT này có có thể chứng minh trực tiếp bằng biến đổi tương đương:
2
x y 2 (x + y) (x − y) (x + y + z)
+ ≥ ⇔ ≥0
y+z z+x x + y + 2z (x + z) (y + z) (x + y + 2z)

GV: Nguyễn Tuấn Anh 2. Một số kỹ thuật, BĐT phụ đặc biệt trong chứng minh BĐT
Trang 112 CHƯƠNG 2. BỔ SUNG KIẾN THỨC

BĐT cuối là hiển nhiên nên bài toán được chứng minh. 

Ví dụ 53 (Trích trong lời giải đề Tỉnh Bà Rịa - Vũng Tàu).

Với x, y, z là các số thực dương. Chứng minh rằng:



r r r r
x y z x y z
+ + ≥ 2+2 . .
y+z z+x x+y y+z z+x x+y

LỜI GIẢI. Không mất tính tổng quát ta giả sử z là số nhỏ nhất trong ba số x, y, z . Khi
đó, ta có: r r r
x y x+y
+ ≥2
y+z z+x x + y + 2z
Ta lại có:
xy 1 1 (x + y)2
= ≤ =
(y + z) (z + x) z (x + y + z)
+1
4z (x + y + z)
+ 1 (x + y + 2z)2
xy (x + y)2
Bài toán được chứng minh nếu ta chứng minh được:
s
√ (x + y)2
r r
x+y z z
2 + ≥ 2+2 .
x + y + 2z x+y x + y (x + y + 2z)2

v v

r
u 1 z u z 1
⇔ 2u + ≥ 2 + 2u
ux + y .
t 2z x+y 2z
2
1+ t
1+
x+y x+y
z
Để dễ dàng, ta đặt t = . BĐT trên được viết lại là:
x+y
r
√ √
r
1 t
2 + t≥ 2+2
1 + 2t (2t + 1)2

r

r
4 t 4t t
⇔ +t+4 ≥2+ 2
+4 2
1 + 2t 1 + 2t (2t + 1) (2t + 1)2


(2t − 1) 2t2 − t − 2
r r
2t t 4t (1 − 2t)
⇔ 2
≥4 2
−4 = r
1 + 2t
r 
(2t + 1) (2t + 1) 2 2t t
(2t + 1) +
(2t + 1)2 1 + 2t

vậy nên ta cần có:


 4t
2 + t − 2t2 ≥ r r
2t t
2
+
1 + 2t
r r(2t + 1)
2t t 
⇔ + 2 + t − 2t2 ≥ 4t
(2t + 1)2 1 + 2t

2. Một số kỹ thuật, BĐT phụ đặc biệt trong chứng minh BĐT GV: Nguyễn Tuấn Anh
CHƯƠNG 2. BỔ SUNG KIẾN THỨC Trang 113

1
vì t ≤ nên
2 s r r r
2t t t t
≥ ; ≥
(2t + 1)2 2 1 + 2t 2

Do đó ta cần có:
r
t 1
 
2 + t − 2t2 ≥ 2t ⇔ t t − (t − 2) 2t2 + 3t + 2 ≥ 0
 
2 2

BĐT trên đúng. Nên bài toán được chứng minh 


Để thấy được nhiều hơn, và nắm rõ hơn ứng dụng của bổ đề, mời bạn đọc hãy làm thử
với các bài toán sau:

Bài tập đề nghị

Bài 2.20 (VMO 1996).

Với x, y, z là ba số thực không âm và không có hai số nào đồng thời bằng 0. Chứng minh
rằng: r r r r
x y z xyz
+ + ≥2 1+
y+z z+x x+y (x + y) (y + z) (z + x)

HƯỚNG DẪN. Sử dụng trực tiếp bổ đề. 

Bài 2.21 (VMO 1996).

Với x, y, z là ba số thực không âm thỏa mãn xy + yz + zx + xyz = 4. Chứng minh rằng:

x + y + z ≥ xy + yz + zx

HƯỚNG DẪN. Sử dụng phép đổi biến (xem trong mục đổi biến) trước khi sử dụng bổ
đề. 

Bài 2.22 (MOSP 2002).

Với x, y, z là ba số thực dương. Chứng minh rằng:


s 2 s 2 s 2
3 2a 3 2b 3 2c
+ + ≥3
b+c c+a a+b

HƯỚNG DẪN. Sử dụng đánh giá:


 s  32 " !#2
 2 r 2
r r
1  3 2x 2y  ≥ 1 2x 2y 2 (x + y)

3
+ + ≥
2 y+z z+x 2 y+z z+x x + y + 2z

GV: Nguyễn Tuấn Anh 2. Một số kỹ thuật, BĐT phụ đặc biệt trong chứng minh BĐT
Trang 114 CHƯƠNG 2. BỔ SUNG KIẾN THỨC

2.3 Bất đẳng thức Gerretsen


Trong một tam giác với p, R, r lần lượt là nửa chu vi, bán kính đường tròn ngoại tiếp và bán
kính đường tròn nội tiếp. Chứng minh rằng:

p2 ≤ 4R2 + 3r2 + 4Rr

LỜI GIẢI. BĐT cần chứng minh tương đương với:


1 2 2
 r2
p − r − 4rR − 1 ≤
4R2 2R2

A B C
⇔ cos A cos B cos C ≤ 8sin2 sin2 sin2
2 2 2

• Nếu ∆ABC là tam không nhọn, BĐT trên là hiển nhiên.

• Nếu ∆ABC là tam giác nhọn, ta chia hai vế của BĐT cho sin A sin B sin C và ta
cần chứng minh:
A B C
cot A cot B cot C ≤ tan tan tan
2 2 2
A
Ta sẽ chứng minh cot A cot B tan2 , và tương tự cho các đánh giá còn lại. Khi đó
2
BĐT sẽ được chứng minh.
Thật vậy, đánh giá trên tương đương với:
cos (A + B) + cos (A − B) 1 − cos C 1 + cos (A + B)
≤ =
cos (A − B) − cos (A + B) 1 + cos C 1 − cos (A + B)

⇔ 2 cos (A + B) ≤ 2 cos (A + B) cos (A − B)

⇔ cos C ≥ cos C cos (A − B)

BĐT cuối là hiển nhiên, nên bài toán được chứng minh.

—————– HẾT —————–

2. Một số kỹ thuật, BĐT phụ đặc biệt trong chứng minh BĐT GV: Nguyễn Tuấn Anh
ĐÍNH CHÍNH TÀI LIỆU

Tài liệu "BẤT ĐẲNG THỨC QUA CÁC ĐỀ CHỌN ĐỘI TUYỂN NĂM
HỌC 2016 - 2017" viết cho năm học 2016 − 2017 có một số sai sót. Trong mục này sẽ
đính chính lại với bạn đọc về những sai sót đó. Bản cập nhật bạn đọc có thể tải lại tại:
https://goo.gl/kRyx98

Sai sót do đánh máy

• Trang 59: Ví dụ 2 , đề đánh sai là "Tìm GTNN của P = a2 + b2 + c4 ". Chỉnh lại:
P = a2 + b2 + c3 . Và khi đó lời giải có điều chỉnh lại đôi chút.

• Trang 71: Đánh máy là "...chuẩn hoán...". Chỉnh lại : "...chuẩn hóa..."

• Trang 78: Đánh máy là "...mà đề bạn đọc tự...". Chỉnh lại: "...mà để bạn đọc tự..."

Lời giải sai

• Trang 22: Bài 16, lời giải ở cách 1 là lời giải sai.

115
Trang 116 CHƯƠNG 2. BỔ SUNG KIẾN THỨC

2. Một số kỹ thuật, BĐT phụ đặc biệt trong chứng minh BĐT GV: Nguyễn Tuấn Anh
Tài liệu tham khảo

[1] http://diendantoanhoc.net/

[2] http://artofproblemsolving.com/

[3] http://forum.mathscope.org/

[4] https://olympictoanhoc.blogspot.com/

[5] Algebraic inequalities old and new methods - Vasile Cirtoaje

[6] Sáng tạo BĐT - Phạm Kim Hùng

[7] Bất đẳng thức suy luận và khám phá - Phạm Văn Thuận, Lê Vĩ.

[8] Sử dụng AM - GM để chứng minh BĐT - Võ Quốc Bá Cẩn, Trần Quốc Anh.

[9] Bất đẳng thức schur và phương pháp biến đổi p, q, r - Võ Thành Văn

[10] Bất đẳng thức qua các đề chọn đội tuyển năm học 2016 - 2017 - Nguyễn Tuấn Anh

117

You might also like